◆ちょっとした疑問はここに書いてね8(はぁと)◆

このエントリーをはてなブックマークに追加
1ご冗談でしょう?名無しさん
         )
   , ―――'
 γ∞γ~  \     ____________
 人w/ 从从) )   /いちいちスレッド立てないで
  ヽ | | l  l |〃 <  ちょっとした疑問はここに書いてね。
  `wハ~ ーノ)    \              Part 8
   / \`「        ̄ ̄ ̄ ̄ ̄ ̄ ̄ ̄ ̄ ̄ ̄ ̄
前スレ
Part7:http://cheese.2ch.net/test/read.cgi/sci/1011199123/
宿題を聞くときは、どこまでやってみてどこが
分からないのかを書くこと。丸投げはダメだからね。

★過去スレ、関連スレはここ>>2よ。
★それから、書き込む前に>>3の注意事項を読んでね。
★数式の書き方(参考)はこちら>>4
2ご冗談でしょう?名無しさん:02/03/01 13:06 ID:moTzBBiL
3ご冗談でしょう?名無しさん:02/03/01 13:07 ID:moTzBBiL
書き込む際の注意

1.)
板の性格上、UNIX端末からの閲覧も多いと推察されます。
機種依存文字(ローマ数字、丸数字等)は避けて下さい。

2.)
以下のような質問に物理板住人は飽き飽きしているので、たぶん無視されます。
しないで下さい。
「相対性理論は間違っています」「量子力学は間違っています」
「宇宙論は間違っています」「シュレディンガーの猫は変です」
「永久機関を作りました」

3.)
宿題を聞くときは、どこまでやってみてどこが分からないのかを書くこと。
丸投げはダメよ。

4.)
厨房を放置できない人も厨房です。
4ご冗談でしょう?名無しさん:02/03/01 13:07 ID:moTzBBiL
【掲示板での数学記号の書き方例】
●スカラー:a,b,c,...,z, A,B,C,...,Z, α,β,γ,...,ω, Α,Β,Γ,...,Ω, ...
●ベクトル:x=[x[1],x[2],...], |x>, x↑, vector(x) (← 混同しない場合はスカラーと同じ記号でいい.通常は(成分を横で書いても)縦ベクトルとして扱う.)
●行列(1成分表示):A[i,j], I[i,j]=δ_(ij)
●行列(全成分表示):A=[[A[1,1],A[2,1],...],[A[1,2],A[2,2],...],...]=[a1,a2,a3,...], I=[[1,0,0,...],[0,1,0,...],[0,0,1,...],...] (← ここでは列ごとに表示(縦ベクトルを横に並べる).行ごとに表示しても構わないが,統一して使わないと混同するので注意.)
●関数:f(x), f[x]
●数列:a(n), a[n], a_n
●足し算:a+b
●引き算:a-b
●掛け算:a*b, ab (← 通常"*"を使い,"x"は使わない.)
●割り算・分数:a/b, a/(b+c), a/(bc) (← 通常"/"を使い,"÷"は使わない.)
●複号:a±b=a士b, a干b (← "±"は「きごう」で変換可.他に漢字の"士""干"なども利用できる.)
●平方根:√(a+b)=(a+b)^(1/2) (← "√"は「るーと」で変換可.)
●指数・指数関数:a^b, x^(n+1), exp(x+y)=e^(x+y) (← "^"を使う."exp"はeの指数.)
●対数・対数関数:log_{a}(b), log(x/2)=log_{10}(x/2), ln(x/2)=log_{e}(x/2) (← 底を省略する場合,"log"は常用対数,"ln"は自然対数.)
●三角比・三角関数:sin(a), cos(x+y), tan(x/2)
●内積・外積・スカラー3重積:a・b=(a,b), axb=[a,b], a・(bxc)=(axb)・c=[a,b,c]=det([a,b,c])
●行列式・トレース:|A|=det(A), tr(A)
●絶対値:|x|
●ガウス記号:[x] (← 関数の変数表示などと混同しないように注意.)
●共役複素数:z~
●階乗:n!=n*(n-1)*(n-2)*...*2*1, n!!=n*(n-2)*(n-4)*...
●順列・組合せ:P[n,k]=nPk, C[n.k]=nCk, Π[n,k]=nΠk, H[n,k]=nHk (← "Π"は「ぱい」で変換可.)
●微分・偏微分:y', dy/dx, ∂y/∂x (← "∂"は「きごう」で変換可.)
●ベクトル微分:∇f=grad(f), ∇・A=div(A),∇xA=rot(A), (∇^2)f=Δf (← "∇"は「きごう」,"Δ"は「でるた」で変換可.)
●積分:∫[0,1]f(x)dx=F(x)|_[x=0,1], ∫[y=0,x]f(x,y)dy, ∬_[D]f(x,y)dxdy, 点[C]f(r)dl (← "∫"は「いんてぐらる」,"∬"は「きごう」で変換可.)
●数列和・数列積:Σ[k=1,n]a(k), Π[k=1,n]a(k) (← "Σ"は「しぐま」,"Π"は「ぱい」で変換可.)
●極限:lim[x→∞]f(x) (← "∞"は「むげんだい」で変換可.)
●図形:"△"は「さんかく」,"∠"は「かく」,"⊥"は「すいちょく」で変換可.

●その他
・関数等の変数表示や式の括弧は,括弧()だけでなく[]{}を適当に組み合わせると見やすい場合がある.
・ギリシャ文字はその読み方で変換可.
・上記のほとんどの数学記号や上記以外の数学記号"⇒∀≠≧≒∈±≡∩∽"などは「きごう」で順次変換できる.
5かおりん祭り ◆IidAAeuI :02/03/01 13:54 ID:???
\(^▽^)/新スレおめでとうございま−す♪
6ご冗談でしょう?名無しさん:02/03/01 16:21 ID:???
かおりん祭りは「ちゅどーん」と変わらんな。
お前はパブロフの犬か?
だから知性があるとは思わなかったとか言われるんだよ。

何が「新スレおめでとうございま−す♪」だ。おめでてーな
新スレならどんなクソスレでもおめでてーのかと小一時間問い詰めたい
7ご冗談でしょう?名無しさん:02/03/01 16:31 ID:???
目に付くように上げておきますよ。
8ご冗談でしょう?名無しさん:02/03/01 16:31 ID:???
といいつつ下げてしまった。
9ご冗談でしょう?名無しさん:02/03/01 17:28 ID:???
>>6
4.)
厨房を放置できない人も厨房です。
10ご冗談でしょう?名無しさん:02/03/01 17:58 ID:???
>>9
厨房を放置できない厨房を放置できない人も厨房です。
11ご冗談でしょう?名無しさん:02/03/01 18:16 ID:???
>>10
厨房を+>>10
12ご冗談でしょう?名無しさん:02/03/01 20:24 ID:???
for(;;){>>11();}
13ご冗談でしょう?名無しさん:02/03/01 20:26 ID:2dQEDRnW
こんにちは。
ミントメールというのをご存知ですか?
これはアメリカで運営されていて、登録者は企業の広告メールを1通受け取るごとに
月に10ドルもらえるというサービスです。
メールを受け取るだけで良いので英語が読める必要はありません。
もちろん登録料・維持費は一切かからないためノーリスクです。
興味のある方は以下の手順に沿って登録してみてください。

↓まずはこのサイトを開いてください。
http://www.mintmail.com/?m=2237270

ここから(sign-up)をクリックします〜!!そしたら加入画面が出ます。
下記の説明通り入力すればいいです。
(*がついている部分のみ正確に入力します。)
- First Name*: 姓 → TANAKA
- Last Name*: 名 → TAROU
- Company Name: 書かなくていいです
- Street Address*: 市区郡より →MINATOKU TORANOMON 1-2-3 SUMAIRUMANSYON 101
(住所は正確に入力して下さい。この住所に小切手を送ります)
- City*: 都市名 → TOUKYOUTO, JAPAN
- State*: そのまま
- Zip*: 郵便番号 → 000-0000
- Country*: 国 → JAPANを選択
- Phone*:電話番号 → 国家番号(日本):81 + 地域番号の前0を除いた電話番号
03-1111-1111 → 81-3-1111-1111,090-1111-1111 → 81-90-1111-1111
- Fax:書かなくてもいいです。
- E-mail*: メールアドレス(全てがメールで処理されるから正確に書いてください)
- Confirm E-mail*: メールアドレスもう一回入力
- Year of birth*: 出生年 → 1970
- Gender*: 性別 → Male(男性), Femaie(女性)
- Password*: 暗証番号 (6文字以上)
- Confirm Password: 暗証番号確認, 上記と同じ
- how do you want to receive commissions that you earn?プレゼント選択
*gift certificates(double$$) プレゼント券(2倍) *cash現金
- do you want to be notified when your referrals sing up?*あなたの会員が登録された場合お知らせしますか?
 yes を選 択
- 興味ある分野10個まで選択します。(10個以上だった場合、エラーになるから10個まで)

- Submitをクリックすれば画面に thank youというメッセージと一緒にあなたのID番号と
 暗証番号が出ます。そして5分以内にあなたのメールアドレスに加入完了のメールが届きます。
14理学部:02/03/01 21:22 ID:xH0havxT
久々にこの板にキタ……

gnuplotでwith pointsよりさらに大きい点でプロットする方法ってあります?
あのままEPSで吐いてTeXに貼り付けてOHPに印刷すると、結構見づらいのです。
15ご冗談でしょう?名無しさん:02/03/01 21:48 ID:???
マジックでぐりぐり。
16ご冗談でしょう?名無しさん:02/03/01 22:27 ID:???
>>14
online helpを見ると、点の型やサイズを指定できるようですが?
17sage:02/03/02 01:02 ID:xnWKkQ8w
「光速度不変」や「光の速度は超えられない」
などはどのように証明されたのですか。
1817:02/03/02 01:03 ID:???
sageミスりました。鬱
19ご冗談でしょう?名無しさん:02/03/02 10:03 ID:???
質問スレなんだからむしろ上げろ。
>>17
実験事実
20ご冗談でしょう?名無しさん:02/03/02 11:46 ID:???
質問です。
任意の半径の球状に分布した電荷を考え、その電荷分布領域の内外の電界を求めろ
っていう問題で、体積電荷密度が場所にかかわらず一様な時の解法はなんとか
分かるんですが、体積電荷密度が球の中心からの距離rの関数になったときの
解法がわかりません。
この場合のガウスの定理での体積積分項の扱い方教えてくださいです。
21ご冗談でしょう?名無しさん:02/03/02 12:42 ID:???
ガウスの定理そのままだと思うが。
22ご冗談でしょう?名無しさん:02/03/02 15:49 ID:1CtEOzld
鏡は左右逆には映り、上下逆に映らないのはなぜだろうか?
湯川秀樹にも解けなかった問題らしいが。
23ご冗談でしょう?名無しさん:02/03/02 16:01 ID:???
鏡は前後が逆になるの。
左右逆にはなってません。
24ご冗談でしょう?名無しさん:02/03/02 16:03 ID:1CtEOzld
前後が逆になったら、近くの物が遠くに見え、
遠くの物が近くに見えると思うのですが?
25ご冗談でしょう?名無しさん:02/03/02 16:08 ID:???
対称面をよく考えろ。
26ご冗談でしょう?名無しさん:02/03/02 16:41 ID:???
>>24
頻出すぎて説明するのだるいんだが、
例えば鉛筆を鏡に映してみろ。
水平に寝かせた時と垂直に立てた時で何か違いがあるか?
鏡の中の鉛筆と手元の鉛筆の、グリップとペン先はちゃんと対応してるだろ?
だがペン先を鏡のほうに向けた時だけは違う。
鏡の像と実物では、ペン先の向いてるいる方向は全く逆になっている。
おまえは勝手に鏡の中に映った自分の右手と左手で鏡の中の
左右を勝手に判断しているだけだ。それが間違いの元。
27ご冗談でしょう?名無しさん:02/03/02 19:39 ID:D1LSFpUQ
君らに物理なんかわかるのかね?
28 ◆NQyNIOsM :02/03/02 20:05 ID:???
>>27
わかりませんが、何か?
29ご冗談でしょう?名無しさん:02/03/02 20:09 ID:???
>>27
教えてください、先生
30ご冗談でしょう?名無しさん:02/03/02 20:52 ID:???
物理学からみて遺伝子って何モノだと思いますか?
31名無しさんα:02/03/02 21:47 ID:AXWegVan
宇宙工学関係と言ったら、どういう大学があるでしょうか…?
進路で悩んでます、本とか見てもよく分からないので…。
32ご冗談でしょう?名無しさん:02/03/02 22:14 ID:ee1QQ5mb
>>31

 関東圏では、東大、東海大、日大。あと機械工学系でも宇宙システムの
基礎技術を研究しているところもある。都科技大にもあったかな?
 その他、名大、京大、九大、大府大に航空宇宙があったと思ったが、
京大は基礎技術中心だと思う。北大、東北大、阪大の機械系でも基礎技術
の研究はやっていると思う。
33ご冗談でしょう?名無しさん:02/03/02 22:14 ID:epxdEa8X
>>31
そういうときはGoogleで探せ。
http://www.jsme.or.jp/sed/database/uni_data.html
34ご冗談でしょう?名無しさん:02/03/02 22:16 ID:???
>>30
でかい分子
35 :02/03/02 22:26 ID:Uw+gtyIy
最小の距離の単位ってなんだったっけ・・・?

36ご冗談でしょう?名無しさん:02/03/02 22:30 ID:???
SI単位系だと距離の単位はmだけですが何か?
あとはどの接頭辞がついてるかだけ。
37ご冗談でしょう?名無しさん:02/03/02 22:37 ID:ee1QQ5mb
32だけど、33のリンク見たら、あの人があそこの教授ってびっくり
マーク連発した。
ほんまかいな? っていう研究室もあると思うから、31君は、検索して
航空板で質問してみるのがいいと思う。
3835:02/03/02 22:39 ID:Uw+gtyIy
あっ 思い出したプランク長だ
39ご冗談でしょう?名無しさん:02/03/02 22:50 ID:???
>>38
それは単位とは言わない。
40名無しさんα:02/03/03 00:43 ID:GCn7s9X7
>>32 >>33
どうもです、探してミマス
41DQNでしょう?立花さん:02/03/03 01:41 ID:dCQ6esF5
特殊相対性理論の
E=MC*C
ってどうやって導いたんですか?
通俗的な解説書では説明がありませ〜ん。
ドキュソな私にも解かるように説明してちょ。
あと、ネゲエントロピーって表現を、物理学に通じた皆さんは
どう思っちゃいます?
42ご冗談でしょう?名無しさん:02/03/03 02:05 ID:DW6D/pDG
タキオンってなにさ?
いやいや、まじな話
43ご冗談でしょう?名無しさん:02/03/03 02:51 ID:37IVNc4N
>>36
Å(オングストローム)ってのはいつ無くなったんだっけ?
44理学部:02/03/03 03:49 ID:KUN1VDaX
遅レススマソ
>>16
あれ、大きさ変えられます?
plot "hoge" with points 3
とかで点種は変えられるようですけど、大きさ変えるのはどうするんでしょう?
45ご冗談でしょう?名無しさん:02/03/03 04:08 ID:???
>>44
online help は読みましたか?
まだでしたら直ちに help with しましょう

46理学部:02/03/03 04:35 ID:KUN1VDaX
>>45
むふー。出来ました。ありがとうございます
plot "hoge" with points ps 2
と、psでポイントサイズ指定ですね。help plotしか見てなかっただよもん
47ご冗談でしょう?名無しさん:02/03/03 10:13 ID:662ZZ7Ng
>>41

 系Sで静止している物体(質量M)が左右(±X方向)からエネルギーE/2を
もつ光を同時に吸収したとする。物体は系Sでは静止し続ける。
 系Sに対して−Y方向に一定の速度で運動する系S’から見るとき、光の吸収の
前後で物体がY方向に一定の速度Vで運動し、速度に変化はない。
 電磁気学によると、エネルギーEの光は運動量E/cをもつ。
したがって、系S’では物体はY方向に、
      (E/c)*(V/c) (光が斜めsinθ=V/cの方向から物体にあたるから)
の運動量を得たことになる。
 ところが物体の速度はVで一定だから、質量が増加したと考える以外には運動量を
保存させることはできない。質量の増加分をmとすると、
      MV+(EV/(c*c))=(M+m)V
これより、
      E=mc*c
48ご冗談でしょう?名無しさん:02/03/03 10:26 ID:662ZZ7Ng
47は、東京図書のボルン著・林訳の『アインシュタインの相対性理論』
巻末に図つきでのっていました。高校教科書レベルで十分理解できる
説明で、アインシュタインが1946年に発表したものとのことです。
49ご冗談でしょう?名無しさん:02/03/03 10:43 ID:???
アインシュタインってトンデモだったんだね。
ここにきている奴もトンデモを馬鹿にするなよ。
既存の理論を理解して自己満足に陥る奴ばっかだね、ここは。
50ご冗談でしょう?名無しさん:02/03/03 10:59 ID:???
自己満足に陥るなんて言うか?
5122:02/03/03 11:13 ID:0h6z5q19
>>26
いや、そんな愚直な答えを期待していたのではなく。
・・・板違いでした。
52ご冗談でしょう?名無しさん:02/03/03 12:00 ID:???
愚直?
何が言いたいのかさっぱりなのだが。
53ご冗談でしょう?名無しさん:02/03/03 12:59 ID:elal52Sw
>>22
昔にも書いたが、
左右というのは前後・上下が決まってはじめて決まる相対的な概念だからだ。

>>23
なぜ前後逆になってる像を見て左右逆に感じるのか、を説明しないと完全な説明にはならないよ。

54ご冗談でしょう?名無しさん:02/03/03 13:49 ID:???
最初からいきなり懇切丁寧にすべて説明する気はないヨ
55ご冗談でしょう?名無しさん:02/03/03 13:51 ID:???
右手を上げた自分の写真と、鏡を前に右手を上げて映った像を見比べれば、
確かに左右逆に感じる。
「奥行きが逆」ということに人間は鈍感なのか?
でも、水溜りに写った風景や逆さ富士などを見ると、鏡面の奥行き方向が逆
に映っているとはよくわかる。
結局、自分自身を鏡に映すから錯覚を起こすのかな?
56ご冗談でしょう?名無しさん:02/03/03 13:56 ID:GdePFioE
塾などで出された課題を解いている時に
もし分からない問題にぶつかったらどうするべきか?
放って置くべきか?それとも分かるまでずっと
考え続けるべきか?前者は、そのまま放った状態に
すると、次に同じような問題が出された時にまた
答えられない事態が考えられるだろう。
後者は考えることは重要なことであるが,もし
長時間考えても分からなかった場合、それこそ
時間の無駄になってしまう。では何が一番良い
方法なのか?やはり専門家に質問することにより
分からない問題も解けるようになり、勉強の効率も
上がる。当塾ではこの質問解答サービスをメール       
またはFAXで行っているので、ぜひご入塾を!

詳細:http://www51.tok2.com/home/ejhems/
57ご冗談でしょう?名無しさん:02/03/03 14:48 ID:???
>>49
ここにもアインシュタインほどのドンでもがくれば面白いんだけどね。
既存の理論の基礎も理解せずに自己満足に陥ってる奴ばっかだからね。
ここに来るトンデモは。
58ご冗談でしょう?名無しさん:02/03/03 16:35 ID:???
定常電流が流れると磁場が発生するけど、
観測者が導線に沿って移動すると相対的に
電荷が移動するように見えてあたかも定常
電流が流れているように見えると思うんだ
けどなんで磁場が発生しないの?ちなみに
高校生なので古典物理の知識しかありません。
59ご冗談でしょう?名無しさん:02/03/03 16:36 ID:Gc21hipp
59get
60ご冗談でしょう?名無しさん:02/03/03 16:44 ID:???
>>58
導線は自由電子だけでできてるのではない。導線全体としては電気的に
中性なんだから、自由電子と打ち消すだけの陽イオンもある。で、
マイナスの電荷を持った自由電子だけでなく、プラスの電荷を持った
陽イオンも同じ向きに同じ速度で「流れる」ので打ち消し合う。

61高校生:02/03/03 16:51 ID:???
>60
帯電してたら?
62ご冗談でしょう?名無しさん:02/03/03 16:56 ID:???
>>61
もちろん磁場ができるよ
63高校生:02/03/03 17:12 ID:???
>62
まじ?
64ご冗談でしょう?名無しさん:02/03/03 17:24 ID:???
>>63
それこそ電流による磁場そのものじゃん
65ご冗談でしょう?名無しさん:02/03/03 17:26 ID:???
>64
自由電子の平均速度が0の状態で観測者が導線に沿って動いたらってことだろ?
66ご冗談でしょう?名無しさん:02/03/03 17:29 ID:???
わざわざハンドル隠さんでいいよ
67高卒:02/03/03 17:30 ID:???
地球の磁場って帯電した地球が回るからできるんだとおもうけど、
地球と自転している観測者は地球は自転していることを認識でき
ないのに、磁場は認識できるので、やっぱ絶対速度は存在するん
でしょうかねえ。
68高校生:02/03/03 17:36 ID:???
高校生ですが、この板のレベルの低さには失望しました。
69ご冗談でしょう?名無しさん:02/03/03 18:05 ID:???
>>67
自転は認識できるよ。フーコー振り子とか
70ご冗談でしょう?名無しさん:02/03/03 18:53 ID:???
>>67
円運動は慣性系じゃないよ……なんか勘違いしてない?
71場科学生一年:02/03/03 21:32 ID:MjbjHUKO
大学院に入ることを考えていますが、今はどの程度の準備が出来ていれば良いのですか?
72ご冗談でしょう?ファイnマンkoさん:02/03/04 00:03 ID:ESmcMkxA
>>47
三行目>>系Sで静止している物体(質量M)が左右(±X方向)からエネルギーE/2を
と、
七行目>>エネルギーEの光は運動量E/cをもつ
の関連がイマイチ理解できません。
リアル厨房の私でも解かるように説明できます?
・・あと、僕ちゃんがDQNだからって弄っちゃ嫌よ。
73ご冗談です。名無しさん:02/03/04 01:35 ID:ZaaEDXKM
角速度に角光速度みたく限界はありますか?
それと太陽系が銀河中を回転する間の遠心力の影響はどう現れますか?
74ご冗談でしょう?名無しさん:02/03/04 04:09 ID:???
>>71
心の準備
75ご冗談でしょう?名無しさん:02/03/04 08:34 ID:tSkYvPOL
>>72

 質量Mの物体に、小球が左右からぶつかってきてくっつく状況と
同じです。
76ご冗談でしょう?名無しさん:02/03/04 12:05 ID:???
比熱がC〔cal/g・k〕、質量がm〔g〕の物体の温度をt〔K〕だけ変化させるのに
必要な熱量Q〔cal〕はどのように表されますか?
 
答・・・Q=mct

なんですけど mとは何ですか? cとは何ですか? tとは何ですか?
教えてください。。(時間とかそういうの)
77ご冗談でしょう?名無しさん:02/03/04 12:14 ID:???
>>76
質量と比熱と温度だって自分で書いてるやんけ
78ご冗談でしょう?名無しさん:02/03/04 12:15 ID:???
全部問題に書いてあるじゃねーか(w
79高校生:02/03/04 12:33 ID:119uP96s
高校生ですが、氏ね。
80ご冗談でしょう?名無しさん:02/03/04 13:53 ID:???
>>71
努力は怠るな
81fd:02/03/04 14:25 ID:6ZOqoCjH
Q熱リケッチアにかかってしまいました とほほ
82ご冗談でしょう?名無しさん:02/03/04 15:42 ID:???
物理主要4科目ってなんですか?
83ご冗談でしょう?名無しさん:02/03/04 20:48 ID:???
レポートをパソコンで書くとき実験書の図とかを
書きこむにはどうしたらいいですか?
84ご冗談でしょう?名無しさん:02/03/04 21:34 ID:???
>>83 フリーの xfig とか tgif で書けばよろし。
85ご冗談でしょう?名無しさん:02/03/05 02:32 ID:9zl5S1i2
質量以外の条件が同じなら超正確に測ったら重いものほど速く落ちる?
86ご冗談でしょう?名無しさん:02/03/05 02:44 ID:7YDS90Am
↑そんなことはありません
87ご冗談でしょう?名無しさん:02/03/05 04:03 ID:???
↑ほんとうですか
88ご冗談でしょう?名無しさん:02/03/05 04:13 ID:7YDS90Am
↑本当です
89ご冗談でしょう?名無しさん:02/03/05 04:20 ID:7YDS90Am
等価原理はこれまでのところ、
一兆分の一以上の高い精度で成立することが確認されています。

cf;エトベッシュ(Eotvos)の実験など
90ご冗談でしょう?名無しさん:02/03/05 11:03 ID:???
>>85-89
何に対する速度を測るかによるよ。
共通重心に対する速度か、地表に対する速度か。
91ご冗談でしょう?名無しさん:02/03/05 13:17 ID:???
空気があれば重いほうが速く落ちる
92ご冗談でしょう?名無しさん:02/03/05 13:28 ID:yUa0KlEz
>>55
文字を鏡に写す場合を考えよう。
字を紙に書いて、鏡の前に立つ。
まずは鏡に写さないで普通に文字を見る。
次に鏡に写してみる。
大多数の人は左右が逆になって見えると思うが、中には上下逆になる人もいると思う。
紙を鏡に向けるときにどう回転させたかの違いだ。
普通のひとは垂直軸を中心に回転させる。その場合は当然左右が逆になって見える。
が水平軸を中心に回転させると上下逆に見えるわけだ。
つまり左右や上下を逆にしているのは鏡ではなくて、その前段階に原因があるわけだ。

では人間を写す場合はどうか。
上の例を参考にして考えると(ちょっと飛躍するけど)、
人間が後ろを向くときに普通はまわれ右をしてバック転のような動作はしない、
ということが関係しているように思われる。

このあたりになると、もう物理とはいえないかもしれないが。
93ご冗談でしょう?名無しさん:02/03/05 13:35 ID:???
>>58
それはいい疑問だ。
それを理解するには電磁場のローレンツ変換というものを勉強する必要がある。
これは、電場や磁場を異なる慣性系から見るとどう見えるかということを説明するものだ。
高校生には少し難しいかも。

しかし58の文は意味不明だぞ。(言いたいことはわからないでもないが。)
もう一度読みなおしてみそ。
94ご冗談でしょう?名無しさん:02/03/05 13:49 ID:???
「ファインマンの2重スリット」の思考実験って何ですか?
95ご冗談でしょう?名無しさん:02/03/05 19:26 ID:T7cjD8jm
ピアノの鍵盤の叩き方で音が変わるのは実感するのだけど、物理的な説明どなたかできますか?
たとえば、一つのキーを鍵盤から離れた位置から、上から勢いをつけて叩くと割れたような汚い音になるけど、
鍵盤の上に指を乗せたまま、ぐっと押し込むと、同じ強さの音でも深みのある澄んだ音になりますよね。
結局は鍵盤がハンマーが動かして、それが弦を叩くだけだから、差なんてないようにも思えるけど、
音は確実に違いますよね。 叩く時のノイズの影響という気もしないし・・
となたか説明できるかた、よろしくお願いします。
96ご冗談でしょう?名無しさん:02/03/05 23:35 ID:oaxffMUy
宇宙はビッグバンから始まったんですよね?じゃあそれ以前は何が存在していたんですか?
97ご冗談でしょう?名無しさん:02/03/06 00:02 ID:8y7y5VS9
地球の北の果ては北極ですよね?じゃあそれより北には何があるんですか?
98ご冗談でしょう?名無しさん:02/03/06 00:24 ID:d/2gGSZg
>>97
北極から北はない。どこを向いてもすべて南。
99ご冗談でしょう?名無しさん:02/03/06 00:27 ID:vhgpKGx+
>>95
強く叩くと弦にはいろんな成分が生まれます、波長と弦の長さがきちんとfitした
ものだけが生き残りますがその間その弦と合った波長以外の音も聞こえる(ノイズ)。
丁寧に叩くと、弦の(たとえばAならAだけ)の成分しか聞こえないからキレイに聞こえる。
……だと、思う。ピアノは弾いたこと無いからしらんけど
100ご冗談でしょう?名無しさん:02/03/06 00:30 ID:???
>>98
101ご冗談でしょう?名無しさん:02/03/06 00:51 ID:rjFOIFke
ちょっと異質な質問かもしれませんが、どうしても聞きたいことがあります。

「物理」とは「物の理」であって、世界の仕組みを解き明かす学問ですよね。
「仕組み」は突き詰めていくと「こうであるとしか言いようが無い」という
「仕組み」を持たないことに行き当たり、それが「法則」などとして
受け入れられていくんだと僕は思ってるんですけど、
それはつまり、「世界で起こることは全て物理(科学)で説明できるが、物理(科学)で世界は全て説明できない」
といったことにならないんでしょうか?

来年度から生まれて始めて物理の授業を受ける未熟者の僕としては、
この質問が「てんで的外れ」なのか「学習する上での大前提」なのかも解りません。
ぜひ皆さんの意見が聞きたいのです、、、







102A.Einstein:02/03/06 01:15 ID:???
私は、神の方程式が知りたい。
103ご冗談でしょう?名無しさん:02/03/06 01:19 ID:???
物理で語るのは近似された世界であり、それ以上でもそれ以下でもない。
物理は「存在理由」を語らない。
物理は「原理」を解釈しない。

あと誰か埋めれ
104A.Eisosteiso:02/03/06 01:30 ID:???
物理を一言で言うと、実験結果を理論でこじつけるだけ。
またその逆もある。
105TSB  cj3114538-a.kasiw1.kn.home.ne.jp:02/03/06 01:35 ID:A/GHLsB8
 この板をはじめてちゃんと除いてみました
よろしく。103さん、真実を掴んでいる様で嬉しいです。
 理由は人の心ですからね、そもそも感情が。
世界は只動いているのです、
ですね。只、物理は原理を解釈します。
104については
「永遠に無秩序が続いているだけ」
「究極の理論が存在する」
「無理やり理論でこじつけていて 全てに矛盾しないだけである」
という三択の話ですね、うん。


ではさようなら。
106ご冗談でしょう?名無しさん:02/03/06 01:46 ID:???
ヘリコプターのシュミレーションしたいんですけど・・・。
プロペラのシュミレーションとか在りますか?
107ご冗談でしょう?名無しさん:02/03/06 01:51 ID:uUlmsI0+
108ご冗談でしょう?名無しさん:02/03/06 08:49 ID:5Tv5P623
手書き計算してるとき、ベクトルのrと、
ふつうのrってどう書き分けてるの?

109ご冗談でしょう?名無しさん:02/03/06 09:12 ID:???
ベクトルのrは縦線を二本
110ご冗談でしょう?名無しさん:02/03/06 10:00 ID:V8hu6vV6
NASA職員は公務員なのですか?民間なのですか?
111ご冗談でしょう?名無しさん:02/03/06 10:06 ID:???
>>105
妄想たれる坊やはもう2度と来なくていいよ。
112ご冗談でしょう@名無しさん:02/03/06 10:13 ID:hfO7cmjc
>>106

 ヘリコプター作っている企業(重工)にはあるだろうけど。
 あと、航技研や防衛庁の研究所などにも。

>>110
 どうなんでしょう。航空板で聞いたほうがいいのでは?
113ご冗談でしょう?名無しさん:02/03/06 13:41 ID:???
>>108
頭に矢印かけよ。オレは太字はキライだ。遠目に区別つけづらいから
114ご冗談でしょう?名無しさん:02/03/06 14:25 ID:WxM+iXPC
俺もそうだったが太字の方が速く書けるんでつい数ヶ月前から太字に切り換えた。
現在大学3年だが、中学生以来の習慣を捨てるのは感慨深かった。
115ご冗談でしょう?名無しさん:02/03/06 14:37 ID:???
>>114 そして院に行くとまた矢印に戻る。
116ご冗談でしょう?名無しさん:02/03/06 19:28 ID:???
「研究する人生」の掲示板を見ることができなくなりました。
あの板は、どうなったのですか。
117ご冗談でしょう?名無しさん:02/03/06 19:49 ID:mYDgL0T1
高1なのですが、どうも物理は難しいです、
教える方もなんか物理は数学や英語と比べて手抜きって感じがするし
理科の中でも化学についでNO2の位置に甘んじている物理はやはり
どこか軽んじられているのですか?
物理の勉強方法でなにかイイの有りますか?

1の顔文字であの小美人風の少女はだれなんすか?


118ご冗談でしょう?名無しさん:02/03/06 19:52 ID:???
サクラタンハァハァ
119ご冗談でしょう?名無しさん:02/03/06 23:06 ID:VLqUJyAL
>>99
ありがとうございます。
確かに、叩き方で倍音が変わるんでしょうね。
ただ疑問なのは、ピアノのハンマーは鍵盤で押した後、鍵盤を離れてフリーに飛んで弦にぶつかります。
鍵盤と弦とは直接繋がっていないんです。
基本的には、ハンマーの運動エネルギーの分だけ弦に振動を与えるから、
音の強度←→音色 は一緒にしか変わらないような・・・  でも違うんですよね。

ハンマー自身の振動が関係するかも、と思ったのですが、どうでしょう?
120 :02/03/06 23:08 ID:???
>>117
煽りですか?おめでてーな!
121おながいします:02/03/06 23:16 ID:GPMQQp4L
ある国家試験の問題です。解き方を教えてください。

2.044MeVのγ線がコンプトン散乱するとき、散乱光子のエネルギー最小値は
入射光子エネルギーの何倍か

1. 1
2. 1/2
3. 1/4
4. 1/6
5. 1/9
122おながいします:02/03/06 23:17 ID:GPMQQp4L
質量数200の原子核が4MeVのα線を放出するとき、
生成核の反跳エネルギーは何MeVか。
123あほ文系:02/03/06 23:31 ID:GPMQQp4L
解決物理野郎のみなさんこんばんわ。
質問ありますいいですか?

  ○○
 ○  ○   ワー
○ ● ○    ワー
 ○  ○
  ○○

↑こんな風にサッカーでボール囲んで肩とか組みながら
相手のゴールにつっこむことは有効ですか?
物理学的見地からのご意見をよろしくお願いします。

124おながいって?:02/03/06 23:32 ID:???
エネルギー保存則と運動量保存則使え>>121-122

125lkh:02/03/07 00:54 ID:sXAoyYKr
最強の合コンメンバー

ブス、大酒のみ女、八代亜紀似の女、レッサーパンダ、
象、小象、豚、虎、朱の盆、デブ、不惑、象、小象、豚、虎、鵜、鴉、
色黒な痩せ熊、狐目のグリコの犯人、カマドウマみたいな馬面
メガネをかけた白豚、ガンノスケ、モ娘の加護チャンの5年後
ネロヒト、クズ、電車ヲタ元ヤン、買い物帰りの主婦、ゲーハー、
おちゃらけ系、ジジババ、マサカズ、ET、セックスリーマン、ガンヲタ、
珍獣、レッドキング、キャベツ、たまねぎ、旧帝大で理系の博士号、
朝日新聞厚木支局の井石、ツルッパゲ、はげとちび、東大理学部
126ご冗談でしょう?名無しさん:02/03/07 01:01 ID:???
質問です!!

元彼(W大)がイタ電を一向に止めません。別れて一年経ちます。
もう、いいかげん止めてホスィ。この前、サークルであった時(半年振り)
挨拶されました。恨みを買うと怖そうなので一応挨拶し返しときました。
なのに、まったくもって止める気配がありません。少なくとも一週間に
5・6回は無言電話があります。寄りを戻したいなら無言電話じゃなくて
話すはずだし(メアドも携帯の番号も知っているので)彼の行動が意味わかりません。

彼の気持ちを代弁してください。ヨロシクお願いします。
127ご冗談でしょう?名無しさん:02/03/07 01:57 ID:???
128ご冗談でしょう?名無しさん:02/03/07 02:04 ID:???
i
12999:02/03/07 02:16 ID:Io6LxUPR
>>119
ピアノの弦を張るときは、ものすごく強い張力で張ると聞きます。
だから強く鍵を叩くと、弦がピアノ本体に強くbindingされているから、ピアノ本体の
方に振動が逃げる分もあるんじゃないですかね。
で、ピアノは平均律でもっともよく響くように内部の空洞が作られています(たぶん)。
だからそこからはずれた音や振動が産まれると、普段より汚く聞こえやすいんじゃない
かな。

俺はバイオリン弾きなのでこれ以上は詳しく知りません
130ご冗談でしょう?名無しさん:02/03/07 02:34 ID:EKcdFDLm
>>123
おまえはオフサイドとゆうものを知らないのか?

  ルールぐらいしっとけ。
131ご冗談でしょう?名無しさん:02/03/07 02:53 ID:???
>>130
マジレスするヴァカ
さすが物理オタク
132ご冗談でしょう?名無しさん:02/03/07 02:53 ID:???
別にネタスレじゃないので…
133ご冗談でしょう?名無しさん:02/03/07 07:38 ID:TBBGz3e4
エレガントな宇宙―超ひも理論がすべてを解明する という本についてなんですが,
非常に面白そうなのでさきほど買ってしまいました.私は物理に関しては全くの素人なんです.
で,買った後で聞くのもなんなんですが素人でもこの本は楽しめそうですか?
ぜひ読んでみた人の感想を聞かせてください
134全くの素人の楽しみかた:02/03/07 09:50 ID:???

●本を頭にのせて落とさないようにして歩けば姿勢が良くなる。

●片手で上げ下げして二の腕のダイエット。

●1.5メートル先に置いて読めば視力トレーニング。

『エレガントな宇宙―超ひも理論がすべてを解明する』
はこんなにヘルシーでスポーティー。
135ご冗談でしょう?名無しさん:02/03/07 10:02 ID:???
>>133 とても面白い本だけど、たとえ話による説明なんで
わかったようなわからないような・・・という読後感が残ると思います。
136ご冗談でしょう?名無しさん:02/03/07 10:08 ID:TBBGz3e4
>>134
舐めんなよ(゚Д゚)ゴルァ!!

>>135
マジすか!ΣΣ(゚Д゚;)
でも買っちゃったもんは仕方ないか
137天使m:02/03/07 13:48 ID:CcrIkGwo
●弾丸が胸に入れてた『エレガントな宇宙―超ひも理論がすべてを解明する』 に当たったおかげで偶然命をとりとめる。

●おせっかい焼きの幼なじみのあいつは、ああ見えて実はかなり男子に人気があり
今日もげた箱開けたら『エレガントな宇宙―超ひも理論がすべてを解明する』がドサドサーと落ちてくる 。

●ネクタイ締めず、寝ぐせ直さず、
『エレガントな宇宙―超ひも理論がすべてを解明する』を口にくわえながら駅まで猛ダッシュ 。

『エレガントな宇宙―超ひも理論がすべてを解明する』
はこんなにヘルシーでスポーティー。
138ご冗談でしょう?名無しさん:02/03/07 14:13 ID:???

物理板でさ、「全くの素人ですが」
を接頭語にするバカがいるけど、素人なら来るなと言いたくなる。
上の本だって素人が楽しめるのかぐらい自分で判断しろと言いたい。
それとも字が読めないバカのか?
139ご冗談でしょう?名無しさん:02/03/07 14:52 ID:???
全くの素人ですが何か
140ご冗談でしょう?名無しさん:02/03/07 14:54 ID:???
>>138
その一文が問題なのではなくて、
質問丸投げ傾向が強いのが問題、なんじゃないかなぁ…
141ご冗談でしょう?名無しさん:02/03/07 17:42 ID:???
>>140
禿胴

関係ないけど、このスレは質問スレの乱立を防止する為
皆様、ageでやりませんか?
142ご冗談でしょう?名無しさん:02/03/07 17:58 ID:urAOwCQy
◆ちょっとした疑問はここに書いてね7(はぁと)◆
http://cheese.2ch.net/sci/kako/1011/10111/1011199123.html
見れないんだけど、どうしてよ?
143136:02/03/07 18:53 ID:TBBGz3e4
>>138
うるせーな まだ俺は物理やってねーんだよ ボケが
お前はすっこんでろ
144ご冗談でしょう?名無しさん:02/03/07 19:13 ID:???
うわ、典型的な春厨反応。
145ご冗談でしょう?名無しさん:02/03/07 19:37 ID:???
あの、ちょっとした疑問なんですが、例えば物理板にある
★すみませんよろしくおねがいします。★
ってスレの中の82の投稿にここからリンク貼りたい時ってどうやるの?

ちょっとした疑問なんですが、教えて下さい。
146ご冗談でしょう?名無しさん:02/03/07 20:35 ID:YuGzB2OA

オレも明日は「ヘルシーでスポーティー」な本でも買って
超弦理論をエンジョイしようかな(クスクス
147ご冗談でしょう?名無しさん:02/03/07 20:41 ID:fFSI4lhI
>>145」と書きます
148ご冗談でしょう?名無しさん:02/03/07 20:41 ID:???
>>146
そういうのは2chの機能説明してるとこ行って勝手に学習してください
149え?:02/03/07 20:43 ID:???
150ご冗談でしょう?名無しさん:02/03/07 20:46 ID:???
あら?なんかちがーう・・・
>>149 http://cheese.2ch.net/test/read.cgi/sci/1014955593/149
151フフ:02/03/07 20:57 ID:???
http://cheese.2ch.net/test/read.cgi/sci/1015346709/82
わかったー!

>>147、148 ごめんなさい。もうしません。
152ご冗談でしょう?名無しさん:02/03/07 21:26 ID:oVzaIUJn
ちょっと質問、水って、真空状態で何pまで持ち上がるの?
水銀は76pだったはず・・・。急に気になって仕方ない。
153ご冗談でしょう?名無しさん:02/03/07 21:31 ID:???
>>152
水面が上がる仕組みは分かる?
ちょっと説明してみれ。
説明しようとする過程で、多分自分で理解できてくる…と思うから
154工房:02/03/07 21:34 ID:???
>>152
仮に
水銀の密度ρ=13546[kg/m^-3]
重力加速度g=9.80665[m/s^2]
の条件からH=0.76[m]押し上げられる理由を考えてみようよ。
155工房:02/03/07 21:41 ID:???
気圧P=1013[hPa]=101300[N/m^2]
を書き忘れていた
156ご冗談でしょう?名無しさん:02/03/07 21:42 ID:oVzaIUJn
即レス有難う御座います。
でもゴメンナサイ、どうか簡単に教えて頂けないでしょうか・・・。
今丁度飲んでて、貧乏学生である私の財布の中身が危ういのです。
10m30pだったとおもったんですが。
157ご冗談でしょう?名無しさん:02/03/07 21:50 ID:???
気圧と重力の釣り合いだよ。
158工房:02/03/07 21:53 ID:???
>>156
だいたい良いと思う。
財布の無事を祈ります。
(俺みたいな暇人とは違う世界を生きる人なんですね。さようなら。)
159ご冗談でしょう?名無しさん:02/03/07 21:54 ID:oVzaIUJn
じゃあ1013pでいいんですよね?
160工房:02/03/07 21:57 ID:???
>>159どうして?
161ご冗談でしょう?名無しさん:02/03/07 21:57 ID:oVzaIUJn
レスつけてくださったみなさん有難うございました。
162工房:02/03/07 22:02 ID:???
彼の財布が心配になってきた。
163まじです@名無しさん:02/03/07 22:12 ID:DlHl0q1T
すいません。
鏡像法(映像法)ってありますよね。
あの説明で、対称の位置に反対符号の電荷をおいても、
その電荷は領域外(導体の中ってことです)なので
領域内(導体外の真空中ってことです)での静電場の方程式には
関係していないって書いてあるんですけど、どうして関係しないのか
わからないんです。
どなたか教えていただけませんか。
164工房:02/03/07 22:18 ID:???
(諸先輩達の回答を期待しています)
165ご冗談でしょう?名無しさん:02/03/07 22:28 ID:???
>>163
「境界値問題」「解の一意性」をキーワードに参考書を見れ
166まじです@名無しさん:02/03/07 22:33 ID:DlHl0q1T
>>165
反対符号の電荷をおくと境界が正しいのはわかるんですよ。
でも、それ以外の部分がなんでその仮想電荷のよって影響をうけないのか
わからないんです。
167ご冗談でしょう?名無しさん:02/03/08 02:13 ID:+THQMbNL
あげ
168まんこ:02/03/08 02:22 ID:???
LEC馬場西本校に通う男です。
かわいい子がいます。やせてて髪の毛がショートで色白でおしゃれな子。
でもがたいのいいはげといつも一緒にいて、もしかしたら彼氏もちかも。。。
LECじゃなくて司○○院出身だけどね。
でもその男がまたかっこわるいんだ。絶対に合格しなさそうな鈍感な顔つき。
でもあの子のまんこをぺろぺろ舐めてきたないオチンポコをIN-OUTしてるかと思うと
悔しチン汁が溢れ出てくる。
あとは人妻でいつも自習室でガリガリやってる人。ちょっと年はいってるけど美人。
やせてる。たまにめがねをかけててそれもまた萌。
それか入っていっとう左側の一番後ろの席に座ってる子
ほとんど勉強してないけどその席がお気に入り。胸がでかい。
ど派手ネーチャンもいい。髪の毛はほとんど金髪。あまり勉強してない。年増の可能性がでかい。
そうだ馬場西で一番かわいいこを教えてやろう。
H田A子さんだ。スタイルばりよし。ツラは正統派。格好もかわいい。
たまに自習室にいるから要チェックだ。俺は処女と見ている。生理は今ちょうどのころだろう。
準ミスはT木M香さん。この子もいいぞ。気が強そうで実はMっ気ありと見た。
その友達もかわいい。かわいいっていうかプチセクシー。小悪魔系。3Pしたい。
H谷さんロリ体型萌え〜。
いつもラウンジのところで勉強会開いてるグループで
ひとつかわいいっていうか美人ばっかりのグループがある。
でもやっぱりばば西でおしゃれな背の低いMさん!好きです!大好きなんです!!
169厨房:02/03/08 05:26 ID:x8k+bCrO
速度を倍にするには4倍のエネルギーを与えないといけないんですよね?
物を落としたとき下に着く瞬間の速度は
4倍の高さから落とさないと倍にならないんですよね。
でも落下時間は倍ですよね。
衛星軌道上のロケットの推力だったら4倍の燃料がいるんですか。
同じ推力を同じ時間与えればリニアに加速しそうな気がするんですが。
170ご冗談でしょう?名無しさん:02/03/08 06:30 ID:???
>>169
速度vの物体と速度2vの物体に「同じ推力」を与えるのに
必要なエネルギーの比は幾らになるの?
171厨房:02/03/08 07:13 ID:x8k+bCrO
軌道上の速度が何に対する相対速度なのかわかんなくなっちゃったんですわ。
ランデブーする時どうやって加速制御すればいいのかわけわからんくなっちゃうんですわ。
172ご冗談でしょう?名無しさん:02/03/08 08:27 ID:???
要は何が質問なんだ?国語が苦手なオレにもわかる質問形式にしろYO
173ご冗談でしょう?名無しさん:02/03/08 08:39 ID:HRLmcq46
れご@qs
174物理学習初心者:02/03/08 15:21 ID:Ev6Sq7/u
原島 鮮 先生の名前の読み方と、主要著書とご実績をどなたか教えてください。
175ご冗談でしょう?名無しさん:02/03/08 15:24 ID:???
はらしません
(な)
176 ◆CibsEIFQ :02/03/08 16:14 ID:???
厨房質問でスマソ。

馬って何馬力くらいあるんでしょうか?
勿論、馬にも個体差があるんで、大体で構いません。
177ご冗談でしょう?名無しさん:02/03/08 17:20 ID:???
ねた?
178 ◆CibsEIFQ :02/03/08 17:34 ID:???
いや、ネタじゃないです。

1馬力は、74kgの物体を1秒間に1メートル移動させる仕事量
         ↓
1秒間に1メートルは良いとして、何故、74kgなのか?

こういうことを考えているうちに、ふと疑問に思ったのです。
それと「馬力」と言うからには「馬一頭分」に相当するパワーの方が
わかりやすくてよいのではないかと疑問に思ったのです。

「こっちの馬は8馬力やけど、あっちの馬は10馬力やねん。」って
変な会話だと思いません?(ワラ
179ご冗談でしょう?名無しさん:02/03/08 17:42 ID:???
ねたかとオモタ。
その昔馬一頭分の力を「一馬力」としたんだよ。
180ご冗談でしょう?名無しさん:02/03/08 17:43 ID:???
力じゃなくて仕事量だ。すまん。
181ご冗談でしょう?名無しさん:02/03/08 20:36 ID:???
182ご冗談でしょう?名無しさん:02/03/08 23:21 ID:zkix88HB
ジョルダンの標準形がわかりません。直接、または良書を教えてください。
ちなみに線形は佐竹を読みましたがテンソル代数で撃沈、斎藤はジョルダンの
単因子のとこで轟沈という駄目駄目な私です。っていうかこの辺りの数学ってそんなに
必要ですかねぇ?(やり方だけ覚えて済ましておこうか・・・)同様の疑問が
テンソルや群論にもあるなぁ。
183ご冗談でしょう?名無しさん:02/03/08 23:28 ID:???
放送大学の線形代数はおすすめ
184ご冗談でしょう?名無しさん:02/03/08 23:51 ID:???
>放送大学
いいかもなぁ。でも俺ん家じゃあ見られんしなぁ。そもそも放送大の講義の
レベルってどれくらいなの?阿部さんとかが出てるらしいからまともな講義
ではあるだろうけど、物理学科ってわけじゃないしねぇ。
だいたい物理数学の良書少なすぎ。アルフケンは答えないし、Vol.4でないし。
英語でやるしかないか、そっちのが安くて済むが・・・
>田崎先生
統計力学は長岡と久保で間に合ってるんで物理数学の本書いてください。
上下で800Pくらいのやつをドカンとね。その方が多分売れますよぉ。
って見てないか(w
185ご冗談でしょう?名無しさん:02/03/09 05:11 ID:???
化学反応で質量は変化するでしょうか?
例えば、2*H2 + O2 = 2*H2O + 6eV
において、(2*H2 + O2)の質量は、
(2*H2O)より、6/(光速)^2だけ、
重いでしょうか?
186ご冗談でしょう?名無しさん:02/03/09 05:50 ID:???
>>185
>2*H2 + O2 = 2*H2O + 6eV

187 ◆CibsEIFQ :02/03/09 12:38 ID:???
>>179-181

ありがとう。
これで今夜から熟睡できる。(ワラ
188ご冗談でしょう?名無しさん:02/03/09 16:06 ID:/yYZT70m
>>185

エレクトロンボルトってエネルギー(熱量)の単位では?
熱が発生したからといって、質量が変化する必要はないと思うが。
189ご冗談でしょう?名無しさん:02/03/09 16:56 ID:???
>>185
値は間違っているが、重い。
190ご冗談でしょう?名無しさん:02/03/09 16:58 ID:???
>>185
もちろん質量は変化するよ

>>188
原子核みたいな強い束縛力の働く系なら一目瞭然だろ
分子だと束縛に関わる力が電磁気力で、核力に比べて弱いから
質量の変化が小さすぎて見えない・見てもしょうがないだけ
191ご冗談でしょう?名無しさん:02/03/09 17:42 ID:dquy/4bh
球体の鏡の中にはいると何が見えますか? (18)
192ご冗談でしょう?名無しさん:02/03/09 17:46 ID:PRWBIEOT
>>190

 束縛に関わる力と化学反応の発熱って関係あるの?
193ご冗談でしょう?名無しさん:02/03/09 17:56 ID:???
>>192
へ?束縛系の質量がどう決まるか考えれば当たり前でしょ?
194ご冗談でしょう?名無しさん:02/03/09 18:11 ID:U4SsXpAL
>>192
逝ってよし。
195ご冗談でしょう?名無しさん:02/03/09 18:13 ID:???
球体の中にある物が見えます>>191
196ご冗談でしょう?名無しさん:02/03/09 18:51 ID:PRWBIEOT
>>193-194

ていうか、どうやって量るの? 6eVなら量れるのでは?
6eVでないなら、値はいくつなの?
197ご冗談でしょう?名無しさん:02/03/09 19:03 ID:???
>>196
おいおい、eVオーダーのエネルギーを質量換算したら
いくつになると思ってるんだ?陽子1つで1GeVだぞ?
198ご冗談でしょう?名無しさん:02/03/09 19:03 ID:Eosk6SFy
教えて下さい。
片方だけ穴があいている空き缶に土をいっぱいに入れて、それを地面(土)に
縦に置いたと仮定してください。
上から力を加えてズブズブとめり込ませていくときに、缶のあいている方を
上に向けたときと下に向けたときとでは、下に向けた(空き缶に入れた土が地面に接している)
ときの方が加える力が多く必要になるのはなんでですか。


199ご冗談でしょう?名無しさん:02/03/09 19:18 ID:PRWBIEOT
>>197

>値が間違っているが、重い

とあったので、元々の式の値が違うのか、6eV/(光速)^2 の値が
違うのか、聞いているだけなのです。
200ご冗談でしょう?名無しさん:02/03/09 19:30 ID:???
知らんよそんなの。自分でジュール表示のでも見つけてきて
適当に換算してくれ
201ご冗談でしょう?名無しさん:02/03/09 20:41 ID:PRWBIEOT
>>200

 化学反応式の等号って、

   H2O(気体) = H2O(液体) + Q(cal)

のような意味なんだけど、このQ(cal)の発熱をどう考えたら質量変化
に結びつくのでしょうか?
202ご冗談でしょう?名無しさん:02/03/09 21:35 ID:???
>>201
あのさ。まず君の背景知識を知りたいんだけど。
中学生相当なのか。高校生相当なのか。大学云年生程度なのか。
大学3年生以上で物理系、でこれだったらもうマジメに相手しないけど。
高校生までならしょうがないけどさ。
203ご冗談でしょう?名無しさん:02/03/09 22:14 ID:GttX0Igj
>>202

ていうか、
 H2O(300℃) = H2O(200℃) + Q(cal)
で、質量変化はあるのでしょうかという質問に等しいと思うのですが。
おかしいでしょうか?
 化学反応の前後で発光する場合は、光のエネルギーが質量に換算
されると思いますが、エンタルピー変化が質量になるとは聞いたこと
がないのです。
>>185 の6eVは、エンタルピー変化では? 
質量はエネルギーになりうるけど、エネルギーはすべて質量になりうるの?
204ご冗談でしょう?名無しさん:02/03/09 22:25 ID:???
先に出てたのは分子単位の質量を聞いてたから当然
微小ながら質量変化はあると答えたのだが。
201や203は多数の水分子の統計的な状態だよな?
それらの水分子の集合全体の質量を問うていると思っていいか?
それなら質量は変化するよ。当然。
205ご冗談でしょう?名無しさん:02/03/09 22:28 ID:???
>>203
 発光したからといって、質量変化するとはかぎらないのでは?
206ご冗談でしょう?名無しさん:02/03/09 22:34 ID:???
>>204

その質量の微小変化の値が、「化学反応エネルギー = m*c^2」 で求める
mの値に等しいかどうかということが、元々の質問の意味では? 
207ご冗談でしょう?名無しさん:02/03/09 22:46 ID:???
>>206
違うと思う理由は?
試しに核反応の式で考えてみてよ。
反応のエネルギースケールと
関与する力の種類が違うだけで本質的には同じ話だろ。
と、相手にまかせてわかってもらえるのかどうか知りたいから
202のように聞いたんだけどどうなの?
208198:02/03/09 22:58 ID:kI5bMl3H
えーと。
お忙しそうなのでよそで訊いてきます。
209ご冗談でしょう?名無しさん:02/03/09 23:29 ID:ekus3OHB
化学反応では質量は変わらないよ……
化学的な結合エネルギー(共有結合とか)が熱や光に変わるだけだよ……
>>198
逆じゃないの?
210ご冗談でしょう?名無しさん:02/03/09 23:39 ID:???
>>209
充電してある電池と、放電してしまった電池、どっちが重い?
ネジが巻いてある時計と、巻いてない時計、どっちが重い?
陽子1個と電子1個が独立してあるのと、水素原子1個、どっちが重い?

全部、同様ですが。
211ご冗談でしょう?名無しさん:02/03/09 23:46 ID:???
>>209
温度301Kの水1モルと、温度300Kの水1モル、どっちが重い?
212ご冗談でしょう?名無しさん:02/03/09 23:50 ID:???
>>209
多分中学生レベルではその答えで正しいんだけど、
高校生、あるいは大学生レベルになると
m=E/c^2 程度の質量差を無視しない場合もありえるわけで、
その場合はエネルギーが変わるということは
質量が変わることを意味する。
>>185 のように問うている場合には明らかに
その m=E/c^2 の差に注目しているわけで、
この場合には化学反応で質量が変化すると答えるべきだろう。
213御家う:02/03/09 23:52 ID:8i9p8+IO
なぜ真空中の光はあんなに速い!?電磁波の一種だが、それを加速
させるものがあるから光は目にみえるはずだ。それならすべての
物体は質量を無限にもっているのか?
214ご冗談でしょう?名無しさん:02/03/09 23:54 ID:???
>>213
分裂症ですか?
215ご冗談でしょう?名無しさん:02/03/09 23:56 ID:???
>>212
外部への熱の流出まで考えないと、質量変化を議論できないのでは?
熱の出入りがあり等温と仮定すると、静止質量の変化と議論できるけど、
化学反応による生成熱が物質に留まっている限り、系の質量変化は無いような。
>>211はそういう事をいっている様な。
21625:02/03/09 23:59 ID:KW5pKZJ/
>>209
 えーと、変わらないといいたいの?
 じゃあ重水素 2個とヘリウム4 1個、どっちが重い?

 理科年表とかに載っていると思うから調べてみたら。
217216:02/03/10 00:08 ID:SPvhfRNd
 すまん、>>210だった。

 名前の「25」はこのスレとは関係ないっす。これもすまぬ。


218ご冗談でしょう?名無しさん:02/03/10 00:22 ID:???
>>215
分子数個の質量の合計についての場合と
マクロな状態での水分子の集合全体についての場合、
別個に聞いてるようには見えないんだがな、最初の質問者は。
219物理学習初心者:02/03/10 01:21 ID:8BNjWcIB
>>175 ありがとうございました。最後に(な)をつけるのだったら、宇治田氏の奥さんだという
   松○千奈さんのほうがタイプです。
質問:大槻義彦先生の「改定新版 基礎教養 物理学」は高校で「物理」
   を選択していない初学者にとっても有益ですか?
   (私は商業高校出身で「化学」履修…。)
220ご冗談でしょう?名無しさん:02/03/10 03:46 ID:sLoHX0a4
>>216
重水素2ヶ→ヘリウム1ヶ
は化学反応じゃないよ。

高校もう一回逝っておいで。
221ご冗談でしょう?名無しさん:02/03/10 03:53 ID:???
最近物理板のレベル低下が喧伝されているが、
さもありなんと実感する。
222ご冗談でしょう?名無しさん:02/03/10 04:23 ID:???
>>220
えーと、今話してることに関しては
どちらでも同じなんだけど、同じだと分からないのかな。
223ご冗談でしょう?名無しさん:02/03/10 04:24 ID:???
>>222
真性ですか?
224ご冗談でしょう?名無しさん:02/03/10 04:25 ID:???
>>222
ファイナルアンサー?
225ご冗談でしょう?名無しさん:02/03/10 04:35 ID:???
>>207=222(だよな?)

>関与する力の種類が違うだけで本質的には同じ話だろ

関与する力とは具体的に何なのか考えてみてみろ。

まあ、この辺までは高校では習わないので
真性ならわからなくても無理はないが・・・
226ご冗談でしょう?名無しさん:02/03/10 04:37 ID:???
>>220 真性のアフォですか?
静止質量の議論で化学反応と核反応を区別してどうすんだ?
227ご冗談でしょう?名無しさん:02/03/10 04:48 ID:???
>>225
言っている物理、もしくは日本語が変。
228ご冗談でしょう?名無しさん:02/03/10 04:57 ID:???
水分子の静止質量はどうやって求めるのですか?
229ご冗談でしょう?名無しさん:02/03/10 05:50 ID:???
>>223-225
何の話をしてるのか本気で理解できてないのか?
関与する力が核力だろうが電磁気力だろうが
今の話には全く関係ないぞ。
230ご冗談でしょう?名無しさん:02/03/10 07:25 ID:???
>215
速度vで運動している物体の実質質量mは、静止質量をm0として
m=m0*{1-(v/c)^2}^0.5
となり、この物体が持つ総エネルギーは
m*c^2 ≒ m0*c^2 + m0*(v^2)/2
となる。右辺第一項は静止質量エネルギー、第二項は運動エネルギー
である。従って、化学反応後は総静止質量は減るが、その際に出た
エネルギーが全て系の運動エネルギーになれば、実質質量は変らない。
(というので良いのでは?)

231ご冗談でしょう?名無しさん:02/03/10 08:26 ID:???
熱化学方程式では、両辺の質量が等しいことが前提では?
中学でも大学でも熱化学方程式とは、そういうものでは?
そもそも、発熱量=Δm*c^2 で質量が変化するのかどうかが問題では?
熱化学方程式で、E=m*c^2 を無視できない場合とは、どういう現象?

>>230
 そもそも分子の場合、回転や振動のエネルギーがあるのでは? 
232ご冗談でしょう?名無しさん:02/03/10 08:32 ID:???
一般的に化学反応では核反応などは考えないので、
相対論による質量変化は無視。こんなの常識だろ。
233補足:02/03/10 08:34 ID:???
素粒子論では重力の影響を無視することもある。なぜかは考えてね。
234ご冗談でしょう?名無しさん:02/03/10 09:20 ID:VD5J8qa/
この宇宙に存在する原子の絶対数は常に一定である(増えないし減らない)。
あってますよね?
235ご冗談でしょう?名無しさん:02/03/10 09:35 ID:???
宇宙の中心点はどこですか?
236ご冗談でしょう?名無しさん:02/03/10 09:50 ID:mlTEMKij
冥王星の外側、太陽から約60天文単位の距離に地球の3倍の質量を持つ太陽系10番目の惑星が発見されました。
 第10惑星は、地球と同じ公転面をほぼ円軌道で公転しています。
 地球の質量を5.974×10の24乗kg、公転周期を365.2422日として、この惑星の公転周期を求めなさい。

 答:公転周期は????日 (少数部分は切り捨ててください。)
237ご冗談でしょう?名無しさん:02/03/10 09:50 ID:???
●←here!
238ご冗談でしょう?名無しさん:02/03/10 10:15 ID:???
エッ?
宇宙の中心点ってハケーンされてないんですか?
全角度の一番遠い星をそれぞれ観測して、それぞれの距離から中心を求められるんじゃないですか?
239かげお:02/03/10 10:46 ID:???
>>230
最後の3行がファイナルアンサーでいいでしょう。

>>234
核融合や核分裂では、増えたり減ったりしますが。

>>238
その方法では、地球が宇宙の中心ということになってしまいます。
240238:02/03/10 11:02 ID:???
エッ?
では地球から観測すると全ての最果ての星の距離は一定なんですか?
241216:02/03/10 11:14 ID:SPvhfRNd
>>220-232
 過去ログ見たらすでに>>212でファイナルアンサー
でてるじゃない。
 まとめれば
「運動エネルギーやポテンシャルエネルギーも
質量をもってる」
 てことになるんだよ。ってあってるよね?
相対論は詳しいとこまで理解してる自信は無いんで
間違ってたら突っ込みよろしく。

>>240
 ある一定の距離の星までしか見えないといったほうが正しい。
242ご冗談でしょう?名無しさん:02/03/10 12:44 ID:???
北極より北には何がありますか?
東の果てには何がありますか?
地球の中心はどこの国に存在しますか?
243ご冗談でしょう?名無しさん:02/03/10 13:18 ID:sl2G7JuJ
スピンが2なら粒子・反粒子間でも引力なのはなぜ?
244高校生名無しさん:02/03/10 14:13 ID:OiKNFecF
あの、1000目前で申しわけないんですが、
凸レンズって光をいってんに集めるんじゃないんでしょうか?
解説講師に通した光を凸レンズに通して、焦点距離上にあるスクリーン
に集光したところ、干渉縞ができた。幅はなんcmかという問題があるんです。
河合塾の名門の森っていう参考書の63番なんですけど。

光が一点に集まるのならば干渉縞なんてできないんじゃないですか?
教えてくださいおながいします。
245 :02/03/10 14:14 ID:pXLHUtO5
宇宙の重さはどのくらいですか?
246高校生名無しさん:02/03/10 14:18 ID:OiKNFecF
幅→明線間隔
247ご冗談でしょう?名無しさん:02/03/10 16:08 ID:Z1zCkMOc
こんにちは
論文中の数式などに出てくる
supやinfなどの記号ってどういう意味なのでしょうか?
集合関連のような気がするのですが...
248ご冗談でしょう?名無しさん:02/03/10 16:48 ID:???
回折
249ご冗談でしょう?名無しさん:02/03/10 16:50 ID:kfmblKqH
>>244
反射光と干渉してんだろ。良くある問題じゃねーか。
っていうか、その参考書の答えとか解説見りゃ出てるだろ。
250高校生名無しさん:02/03/10 19:28 ID:OiKNFecF
>>249
どの反射光ですか?
解説はもうメイセンの縞模様が出来ることを前提に話がすすめられているので
わかりませんでした
251ご冗談でしょう?名無しさん:02/03/10 20:55 ID:2c/V+UKH
>supやinfなどの記号ってどういう意味なのでしょうか?

上限と下限。詳しくは解析の本を参照のこと。

集合Aに属する全ての数が一定数Mを超えない時、Mを上界という。
最小の上界を上限という。Aの上界をsupAとかく。infは類推してね。
252訂正:02/03/10 20:56 ID:2c/V+UKH
Aの上界をsupAとかく > Aの上限をsupAとかく
253ご冗談でしょう?名無しさん:02/03/10 21:49 ID:???
細かい説明なんてせんでもsuperior,inferior
て書けば分かるんじゃないか
254 質問君:02/03/10 22:05 ID:FqtmMbqw
長さ140cm,直径6.5cm,重さ4kgの木の棒があります.
この一端に円盤を取り付けて立てて置きたいと思っています.
その為には円盤の半径をどの程度にしたら良いでしょうか?
255質問君:02/03/10 23:04 ID:FqtmMbqw
一般的に,円盤の材質は棒と同じで
A度程度の揺れまで,つまり棒と地面とが
(90-A)度となるまで倒れこまない場合について
計算式はどうなるでしょうか?

256ご冗談でしょう?名無しさん:02/03/10 23:19 ID:???
宿題を聞くときは、どこまでやってみてどこが
分からないのかを書くこと。丸投げはダメだからね。
257ご冗談でしょう?名無しさん:02/03/10 23:36 ID:???
化学反応で質量が(あまつさえ静止質量が!)減ると逝っていた輩は
はづかしくなって消えたのか?
258ご冗談でしょう?名無しさん:02/03/10 23:40 ID:???
>>257
まだ分かってないの?
259ご冗談でしょう?名無しさん:02/03/11 00:05 ID:???
>>257 アフォ発見。
260ご冗談でしょう?名無しさん:02/03/11 00:09 ID:O6EFvdHn
>>258
どうも真性のようだな。

もう一度、>>225を読み返してみろ。

そして、とりあえず、化学反応の場合と核反応の場合と、
それぞれ反応に関与する粒子を言ってみろ。
261ご冗談でしょう?名無しさん:02/03/11 00:11 ID:O6EFvdHn
念のため聞いておくが、
ゲージ粒子について何も知らなければ、
先にそう言ってくれ。
262ご冗談でしょう?名無しさん:02/03/11 00:57 ID:???
>>260,261
物理用語を振り回しても、馬鹿丸出しですが。
教えて欲しいなら正直にそういったら?
263ご冗談でしょう?名無しさん:02/03/11 01:02 ID:O6EFvdHn
>>262
ていうか、早く答えれYo!

あと、>>228にもな。
264ご冗談でしょう?名無しさん:02/03/11 01:07 ID:???
228?もうとっくに答えてますけど。
265ご冗談でしょう?名無しさん:02/03/11 01:07 ID:???
>>262
>物理用語を振り回しても、

やはり知らないと言っている様なものですな(プ
266ご冗談でしょう?名無しさん:02/03/11 01:07 ID:LHMEKMbQ
>>263
 たとえば水素分子の「静止質量」は、
(陽子の静止質量)*2+(電子の静止質量)*2じゃありません。
古典的でかまわなければ、電子は(厳密に言えば陽子も)運動して
いるから、その分の相対論的質量増加があるはずです。

 こんなとこでどう?
267ご冗談でしょう?名無しさん:02/03/11 01:09 ID:???
>>266
>相対論的質量増加があるはずです

静止質量の意味がわかっておられますか?
268ご冗談でしょう?名無しさん:02/03/11 01:11 ID:???
まあ、大抵の厨房は質問されてもまともに答えず、
ただただ、相手が疲れて(or飽きて)
この話題を早く終わらすことを願うのみですな。
269ご冗談でしょう?名無しさん:02/03/11 01:14 ID:???
あと、違うスレの方もちゃんと逃げないで答えれよ。
270ご冗談でしょう?名無しさん:02/03/11 01:16 ID:???
>>266

水素原子の静止質量 = 陽子の静止質量 + 電子の静止質量 - 結合エネルギ

結合エネルギ = m_e/2α = 13.6eV

水素分子の静止質量 = 水素原子の静止質量 * 2 - 共有結合エネルギ
271ご冗談でしょう?名無しさん:02/03/11 01:17 ID:LHMEKMbQ
>>267
 ぬぅ、レスが早い。
 つか脊椎反射で反応されても・・・。

 ちょっとは内容を読んでよ。水素分子の「静止質量」だよ?
普通はこういう言い方はしないとは思うけどさ。
272270:02/03/11 01:17 ID:???
↑修正 α→α^2
273ご冗談でしょう?名無しさん:02/03/11 01:18 ID:???
>>266
あ、そこは逆ですってば。
水素分子のほうが束縛エネルギーの分軽くなるんですよね。
274ご冗談でしょう?名無しさん:02/03/11 01:19 ID:???
>>263
陽子質量+電子質量-クーロン力による束縛エネルギー
で、それが何か?
275ご冗談でしょう?名無しさん:02/03/11 01:30 ID:???
「水」分子と言ってるのに、
何故「全員」(w が「水素」分子についてしか答えないのか。
276ご冗談でしょう?名無しさん:02/03/11 01:32 ID:???
あと、>>260にはどうして「誰も」答えないのだろうか。
277266:02/03/11 01:33 ID:LHMEKMbQ
>>270,273,273,274
 つっこみ、どもありがとさんです。
278ご冗談でしょう?名無しさん:02/03/11 01:35 ID:???
>>275
同じだよ。各粒子の質量和-結合エネルギー。
で?

>>276
>>229が読めないのか?
279ご冗談でしょう?名無しさん:02/03/11 01:39 ID:???
>>278
いや、だから、とりあえず名前を出せよ。

お前さん、昔から常にそうだな。
280ご冗談でしょう?名無しさん:02/03/11 01:44 ID:???
厨房にはゲージ粒子(γ,W,Z,g)の名前を教えない事に決定しました。
281ご冗談でしょう?名無しさん:02/03/11 01:46 ID:???
ゲージ粒子が何の関係あるの?
特殊相対論から出る質量とエネルギーの関係だけで十分な話でしょう。
で、278のレスに対する反論はございますか?
282ご冗談でしょう?名無しさん:02/03/11 01:49 ID:???
>>278
宿題の答を教えて欲しいなら、正直にそう言いなさい!
283ご冗談でしょう?名無しさん:02/03/11 01:50 ID:???
>>281 直接関係ないと思うが、279は何故か関連させて、名前を聞きたいらしい。
284ご冗談でしょう?名無しさん:02/03/11 01:52 ID:???
>>282
レス相手間違ってませんか?
>>283
宿題か何かなのかな…
285ご冗談でしょう?名無しさん:02/03/11 01:54 ID:???
>>281
じゃあその結合エネルギーを媒介してるのは何よ?

また、核子における結合エネルギーを媒介してるのは何よ?

ほんで、各ゲージ粒子に共通していない最も大きな相違点は何よ?

>>>>281
ほんで、じゃあその結合エネルギーを媒介してるのは何よ?

また、核子における結合エネルギーを媒介してるのは何よ?

ほんで、各ゲージ粒子に共通していない最も大きな相違点は何よ?

また、>>233の答えは何よ?
286ご冗談でしょう?名無しさん:02/03/11 01:55 ID:???
↑打ち込みミス
287ご冗談でしょう?名無しさん:02/03/11 01:58 ID:???
いやだから、それが最初の話と何の関係があるのかな?
質問する理由を教えてくれる?
そんな素粒子論の入門書読めば載ってるようなこと聞く理由を。
288ご冗談でしょう?名無しさん:02/03/11 01:58 ID:???
マジで宿題っぽいな。
289282:02/03/11 01:59 ID:???
>>284 すまそ! 相手間違えた。
290ご冗談でしょう?名無しさん:02/03/11 02:02 ID:???
クーロン力による結合は光子(質量0)が媒介し、
核力による結合は中間子(質量あり)が媒介するという事ではないの?
291ご冗談でしょう?名無しさん:02/03/11 02:04 ID:???
>>290
それが一体何の関係があるのか、
285以外の誰にもわからない謎なんだな・・
292ご冗談でしょう?名無しさん:02/03/11 02:05 ID:???
分子の静止質量を核子の静止質量と同様に扱って
悦に入っていたようだが・・・(プ
293ご冗談でしょう?名無しさん:02/03/11 02:06 ID:???
どうやら、宿題の答を教えて欲しいようだが…

弱い束縛系の質量 = 構成要素の質量 - 結合エネルギ

こんな事もしらんで、良く物理やってるな。
294290(=285ではないが・・・):02/03/11 02:08 ID:???
>>291
つまり、核反応の場合は中間子の質量分だけ質量欠損が起こるのに対して、
化学反応の場合は元々光子に質量がないから欠損しないんじゃないの?

漏れはそう思ってたけど。
295ご冗談でしょう?名無しさん:02/03/11 02:08 ID:???
>>285>>233
弱いからだよ。それが何か?
今はその効果の大小は問題にしてないことは
最初の質問直後のレスにも明記されてるよね。
はなからこれは原理的な問題なんだから。
296ご冗談でしょう?名無しさん:02/03/11 02:11 ID:???
>>294
反応後に実粒子の中間子が飛んでいったら、
それは別の反応じゃん。その勘違いはやばいぞ。
何か手持ちの本あったらすぐ見ておいて。
正しい知識を取り込みなおしておいた方がいい。
297ご冗談でしょう?名無しさん:02/03/11 02:11 ID:???
>>294 それは285がそう思ってるって想像でいってるんだよね?
あと、あなたの理解の過去形なんだよね?
298ご冗談でしょう?名無しさん:02/03/11 02:14 ID:???
>>292
ここは掲示板で、相手と対面してるわけじゃないんだから
間違ったら素直にそれを認めたほうが、自分のためになるよ。
プライドを捨てられるのがこういう場所のいいところなんだから。
299ご冗談でしょう?名無しさん:02/03/11 02:19 ID:???
今更だけど、煽りはなしでいこう。
300ご冗談でしょう?名無しさん:02/03/11 02:21 ID:???
>>296
>反応後に実粒子の中間子が飛んでいったら

??????
質量がエネルギーとして解放される場合の事を言っているのですが・・・?

>>297
いや、そんなに詰問されても・・・
核反応の質量欠損は中間子の分だという事は周知ですが、
化学反応において質量欠損するという話は聞いた事が無いし、
結合エネルギーは光子なのだから当たり前かなと・・・

化学反応で質量欠損するなら、
核反応で中間子に当たるものは何かなと・・・
301ご冗談でしょう?名無しさん:02/03/11 02:26 ID:???
>>300
>核反応の質量欠損は中間子の分だという事は周知ですが、

これがあなたの勘違いの元。
化学反応で質量欠損するという話を聞かないのは
実用上まったく無視できる程度の質量変化しかないからです。
結合エネルギーが質量欠損のもとです。
原子核の質量と結合エネルギーの関係とか、
素粒子や原子核の勉強はじめた時に最初に習いませんでしたか?
原子核の質量を、結合エネルギーや表面エネルギーなどの
さまざまなエネルギーの寄与からくるものとするモデルで
計算することを行いませんでしたか?
302ご冗談でしょう?名無しさん:02/03/11 02:26 ID:???
>核反応の質量欠損は中間子の分だという事は周知ですが、
どこでそんな情報を入手されたんですか?
303ご冗談でしょう?名無しさん:02/03/11 02:38 ID:???
>>300
中間子の質量分、核反応で質量が変化するのだとしたら…
重陽子の質量は陽子+中性子より100MeV以上
軽くならないと計算あいませんが…
304ご冗談でしょう?名無しさん:02/03/11 03:01 ID:???
中性子が陽子より少しだけ質量が重い理由を教えて下さい。
安直には電荷をもっている分、陽子の方が重そうに思えるのですが。
305ご冗談でしょう?名無しさん:02/03/11 03:14 ID:???
構成もとのクォークの種類が違うし
306ご冗談でしょう?名無しさん:02/03/11 03:39 ID:???
…つか、E=mc^2 が相対論からどう導出されるか知ってれば、
反応の種類によって質量欠損があったり無かったりするわけないことくらい
わかるだろうに…
化学反応ではエネルギーや運動量が保存しなくてもいいのかよ…
307ご冗談でしょう?名無しさん:02/03/11 03:40 ID:???
>>305
し、何?
308305:02/03/11 08:18 ID:???
>>307
こーせーもとのーークォークのしゅるいがちがうしーーみたいなー
309307:02/03/11 08:51 ID:???
>>308
アホかお前。>>304は「何故中性子の方が重いのか」聞いてんだろが
「何故違うのか」じゃねーんだよ。すっこんでろ
310305:02/03/11 08:56 ID:???
>>309
女子供はすっこんでろ
311ご冗談でしょう?名無しさん:02/03/11 09:35 ID:???
(うう、このスレで「終了」打ちたくなってしまう...)
312ご冗談でしょう?名無しさん:02/03/11 10:14 ID:PABAu8qU
直接物理学とは関係ないことかもしれないので恐縮ですが、
最近自宅の書棚において、新しく購入した本のページが一晩で波打ったように
曲がってしまうという奇妙な出来事が度々起きて困っています。
よくみるとハードカバーは本の表側に凸の形に少し反っているようです。
原因を3つほど推理してみました。
1.本のカバーをはずすため
2.最近は寒暖の差が激しいため
3.部屋の湿度が適切でないため
しかし、1については昔からやっていたことですが、これまではこのような
ことはありませんでした。2や3では新しい本だけがなるということが説明
できません。ちなみに私の住んでいる地方での最近の最高気温は15℃最低
気温は5℃といったところです。被害の程度はランダウ力学が最もひどく、
次いで理論電磁気学、最も少ないのが熱力入門(佐々)です。ページ数が
少〜中程度のハードカバーの本がひどいようです。相対論(佐藤勝彦、岩波)
や線形(藤原毅夫)が同様に大きく被害があることからもそういえそうです。
本自体は多少ページがくり難くはあるものの読めるので構わないのですが、
はっきりとした原因がつかめないのが気がかりです。同じような経験をお持ちの方
がおられましたらお話をお伺いしたいのですが。
313ご冗談でしょう?名無しさん:02/03/11 11:16 ID:1eLr+MEj
やっぱ湿度でないの?
実験的に冷凍庫に入れてみたら?
314ご冗談でしょう?名無しさん:02/03/11 12:47 ID:???
>313
その心は?
315あいーん。:02/03/11 13:12 ID:???
くだらないことですいません。天才は昔は頭が悪かったとよく言われますが私はそんなことは無いと思っています。それでアインシュタインのいたETH(スイス連邦工科大?)ってどれくらいのレベルの大学なのですか?
316ご冗談でしょう?名無しさん:02/03/11 13:27 ID:???
>310
まあまあ。
クォーク知らない奴にむきになるなよ。
317305:02/03/11 13:38 ID:???
>>308>>310
騙るなヨ
318 :02/03/11 13:52 ID:???
まだ私は大学一年なのですが院試について聞きたいことがあります。他大の院を受けるときに自分の位置(大学入試では偏差値)を知るにはどうすればいいのでしょうか?やりたいことができる院が複数あるときに困る気がして…。
319ご冗談でしょう?名無しさん:02/03/11 14:10 ID:???
>核反応の質量欠損は中間子の分だという事は周知ですが、
なにわけのわかんないこといっているのかと思ったら。
そーいうことだったのね。エラーが見つかって良かったね。
320307:02/03/11 16:26 ID:???
>>316
ありゃ。メール欄に書くのはさすがにわかりにくかったか
まあそれはどうでもいいけど、
中性子質量が陽子質量より大きい理由の定性的な説明がホスィ…
というか正直質量計算の具体的方法を知らない(w

321ご冗談でしょう?名無しさん:02/03/11 16:37 ID:dGZ4ybVc
>>320
素粒子の質量ちゃんと計算できる理論なんて今現在存在するの?
単なるパラメータ合わせとかでなく。
322ご冗談でしょう?名無しさん:02/03/11 16:52 ID:???
ベータ崩壊じゃだめなのか。
323ご冗談でしょう?名無しさん:02/03/11 17:24 ID:???
>>322
むしろ質量の差は、中性子が陽子にβ崩壊できる理由の様な。
どなたか、定性的な議論でもご存じないですか?
324ご冗談でしょう?名無しさん:02/03/11 19:12 ID:dfNuTnUm
宿泊代が一人1000円の旅館に3人の宿泊客が来た。
3人は宿泊代として3000円を旅館の主人に渡したが
主人は割引きしてやろうと思い使用人に500円を返してくる
ように言った。ところが使用人は500円だと3人で割り切れない
からと200円を自分のポケットに入れて3人に100円ずつ渡した。
ということは3人は1000―100で900円ずつで合計2700円
を払ったことになる。使用人の取った200円を足しても最初の3000円
には100円足りないことになる。100円はどこへ消えたのか?



ヒント:主人はいくら宿泊料を取ったか?
326ご冗談でしょう?名無しさん:02/03/11 20:42 ID:???
>>324
2700 に 200 足すなよ。轢けよ。
てゆうか物理じゃねえだろ。
327お願いします:02/03/11 21:08 ID:mJOXCQZ/
バットを折るのに最適なポイントを知りたいのです。

バット折りの状況:
・バットを水平にして、両端を固定。
・これをサッカーボールを蹴るような「蹴り」で折りたい。
・本数は2、3本。

通説:バットの一番細い所よりもやや上の部分(芯に近い方)を蹴ると良い、と言われているそうです。


板違いなんでしょうか?
「お前が考えろ!」って言われて困ってます。
お願いします。
328ご冗談でしょう?名無しさん:02/03/12 00:08 ID:???
>>327
おまえがじけーんしる!
329 :02/03/12 00:21 ID:???
たとえばシリンダー型のスペースコロニーの中で
住人が上(回転の中心)方向にジャンプした場合
元の場所に着地するのでしょうか?
あるいはそのまま中心方向へ飛んでってしまうのでしょうか?
330ご冗談でしょう?名無しさん:02/03/12 00:59 ID:vdTKf/i6
はぁ?
コロニーが回転してなきゃそうなるな
331j:02/03/12 01:41 ID:YhKWl+DR
どうしても発散が分からないのです。
微小立体を場に置いてその立体を通るフラックスの単位面積あたりの量を考えて、その体積を0に極限まで近ずけた時の量、と本には書いてあり、何となく理解は出来るんですが、しっかり確信を持ってイメージできません。
どなたか分かりやすいイメージを知っているなら、是非教えてください。
お願いします
332皆様、緊急です。:02/03/12 02:01 ID:???
http://natto.2ch.net/test/read.cgi/mass/1015750980/
遊んでる場合じゃない!

2ちゃんを政治家に潰されたくない奴はこのスレを
たくさん宣伝しろ!
皆に知らせるんだ! 他のサイトにも貼れ!
今回はマジでヤバイぞ!

映画とかのいろんなシーンから、色々まずい!
灯台で、原子力研究すると左遷される、より数倍ヤバイ

皆とりあえず見て下さい!!
333ご冗談でしょう?名無しさん:02/03/12 02:33 ID:???
>331
湧き出し
334ご冗談でしょう?名無しさん:02/03/12 02:55 ID:???
>>332

マジ?ネット規制入るわけ? とマジレスしてみる
335ご冗談でしょう?名無しさん:02/03/12 03:36 ID:???
>>329
ちゃんと計算しなくても、コロニーが非常に大きくて
ジャンプの高さもコロニーの直径に比べてはるかに小さい
という条件で考えると普通の地上の状態とまったく同じに
考えることができるから、元の位置に落ちるだろうと予想される。
計算もまあできると思うけど。
コロニーの質量ぐらいは無限大に近似していいかな。
336ご冗談でしょう?名無しさん:02/03/12 03:38 ID:???
>>305
こうゆう事がいいたかったのか?

陽子(uud) 938.27 MeV 中性子(udd) 939.56 MeV で質量差 1.29 MeV を説明したい。
構成要素の質量を見ると u: 2〜8MeV d: 5〜15MeV で、あと Mu < Md もいえる。
よって中性子の方が重い。

多分、違うと思う。
337ご冗談でしょう?名無しさん:02/03/12 04:04 ID:j8ji+AEk
自然単位と言う単位系ではc=1、h/2π=1、G=1となるようにしていますが、
h=1ではなく h/2π=1としているのは解ります。
ここで G=1としているのがわかりません。
有理単位系で電荷q1、q2の間に働く力は
(1/4π・ε0)・q1・q2/r^2
と表されます(ε0:真空の誘電率、r:電荷間の距離)。
だとすると自然単位では質量m1、m2の間に働く力は4π・G=1として
G・m1・m2/r^2=(1/4π)・m1・m2/r^2
と表されるべきだと思うのですが。
338ご冗談でしょう?名無しさん:02/03/12 04:06 ID:???
>>336
あなた、素粒子の人ですね。
339ご冗談でしょう?名無しさん:02/03/12 04:09 ID:???
>>337
G=1になんてしませんけど??
340ご冗談でしょう?名無しさん:02/03/12 05:12 ID:???
>> 338
なんで分かったの?

ともかく、陽子より中性子の方が重い定性的な理由は知りたいのだけど…
341ご冗談でしょう?名無しさん:02/03/12 10:11 ID:???
摂動論がダメダメになるような領域で定性的な説明なんて無理でわ
342ご冗談でしょう?名無しさん:02/03/12 10:40 ID:???
>>341 それは分かるんだけど、どっちの方が重い?といった問いには
サムルールとかその辺からの定性的な理解があっても良いような気もするんで、
ハドロン関係の識者の意見を仰ぎたいのら。
343ご冗談でしょう?名無しさん:02/03/12 12:11 ID:???
>>342
将来のことはわからないけど、今のところはないんじゃない? あったら
もう誰か紹介してるでしょ。陽子・中性子の質量〜1GeVに対して〜1MeVの
差(精度0.1%)が議論できるほどには理解が進んでないと思うな。
344ご冗談でしょう?名無しさん:02/03/12 12:33 ID:???
静電ポテンシャルで損する分よりQCDの相互作用のほうが強い。
今のところ定性的に言えるのはこの程度ということで満足しる!>>342
345ご冗談でしょう?名無しさん:02/03/12 12:36 ID:???
中性子の質量の一部が電荷になると陽子
と言う簡単な説明をどこかで見かけました。

こんなんでどう?
346ご冗談でしょう?名無しさん:02/03/12 13:14 ID:w8+s+JGY
やっぱビックバン説って嘘っつーん?
347ご冗談でしょう?名無しさん:02/03/12 14:01 ID:???
「π0の質量の一部が電荷になるとπ+」
が説明になってないのと同様に>>345は説明になっていない
348ご冗談でしょう?名無しさん:02/03/12 15:23 ID:???
質量が電荷になるってどういうこと?意味わかんない。
349ご冗談でしょう?名無しさん:02/03/12 19:27 ID:???
>>344
>静電ポテンシャルで損する分よりQCDの相互作用のほうが強い。

多分、そういう事なんだろうけど、何故に、中性子にはそのQCDの効果が効いて、
陽子ではそうならないかが知りたい。

>>345 ???です。
350ドキュソ:02/03/12 21:07 ID:???
レベル高すぎ、我々ドキュソは近づくことができないぞ?
351ご冗談でしょう?名無しさん:02/03/12 21:17 ID:???
磁束密度BからベクトルポテンシャルAを得る方法がわかりません.
力を貸してください.

条件としては,
磁束密度のx,y成分はゼロ.
磁束密度のz成分はある値を持つ.

教科書では
rotA=B に
divA=0 を導入することで
Aは一意に決まるとありますが,
式は確かに作れますが,計算できないです.
BからAを直接求めるような計算問題なんて載ってないし.
なんかうまい抜け道あるんでしょうか?
352ご冗談でしょう?名無しさん:02/03/12 22:55 ID:???
ただの連立微分方程式でしょう。
353ご冗談でしょう?名無しさん:02/03/12 23:50 ID:???
>>351
その条件なら、B=(0,0,Bz)と書けますよね?
それでrotとdivをバラして書いてあとはそれを解きましょう
354かげお:02/03/13 00:19 ID:???
>>329
どちらでもありません。転向力により、コロニーの回転方向にずれますから。

>>340
陽子の電荷が生む電場のエネルギーの分だけ軽い、って説明は手抜き過ぎ?
355ご冗談でしょう?名無しさん:02/03/13 07:46 ID:???
>>354
>陽子の電荷が生む電場のエネルギーの分だけ軽い、って説明は手抜き過ぎ?

むしろ、逆で、重くなるとおもうのだけど…。
356ご冗談でしょう?名無しさん:02/03/13 10:38 ID:???
>>355
に尿意。
357ご冗談でしょう?名無しさん:02/03/13 11:31 ID:???
>>349
ちゃうちゃう。陽子のほうが静電ポテンシャルで損する分を打ち消すほど
QCDの結合が強いってこと。なぜそうなるかは、低エネルギーでのQCDの
詳細を知らないといけないけど、摂動論がダメダメな領域なので、定性的な
議論など全然できないくらい難しい。
358ご冗談でしょう?名無しさん:02/03/13 13:21 ID:???
野球豚の巣窟(藁

http://ime.nu/www.baseball-lover.com
359ご冗談でしょう?名無しさん:02/03/13 14:10 ID:???
>>351
直線電流が作る磁場の求め方と同じだ
360ご冗談でしょう?名無しさん:02/03/13 15:56 ID:???
>>357
QCDから摂動論で直接出そうとするのが困難なのはよく分かるよ。
適当な有効理論で陽子より中性子が重くなる理由って議論されていないのかなぁ?

あと、ラティスの計算ならこの事をだせるのだろうか?
361ご冗談でしょう?名無しさん:02/03/13 16:27 ID:I1VCNWVQ
ボンドグラフ(bond graph)って何?
詳しい人いませんか?
362ご冗談でしょう?名無しさん:02/03/13 17:06 ID:???
陽子と中性子の質量差は、普通uとdの質量差から来ると考えられている。
uとdの質量差はQCDから出てくる量ではないので、格子を使おうが
どうしようが出せない。クォーク質量の起源の話になる。
363ご冗談でしょう?名無しさん:02/03/13 17:46 ID:???
>>361
ボンドガールならわかるのですが…
364ご冗談でしょう?名無しさん:02/03/13 17:48 ID:???
Push-pull flux quantum
これって適当な日本語訳はなんでしょう?
365>>361:02/03/13 18:33 ID:yc6mG6YS
ボンドグラフは、抵抗・コンデンサ・インダクタなどからなる電気回路、
剛体・バネ・ダンパなどから構成される力学系、流体抵抗・タンク・流体
の慣性からなる流体系が、構成要素がエネルギーの流れによって結合され
ているシステムという共通の視点で捉えられることに注目したシステムの
グラフ(線図)表現である。
366361:02/03/13 18:55 ID:I1VCNWVQ
>>365
そうそう、そのボンドグラフです。

いくつかのノードが、変な矢印で結ばれているんだけど、
意味がわかりません。

ノードと矢印にはどんな意味があるのでしょうか?

ヨロシコ
367ご冗談でしょう?名無しさん:02/03/13 19:10 ID:???
押したり引いたり量子磁束
368ご冗談でしょう?名無しさん:02/03/13 19:50 ID:???
>>362
たとえuとdのcurrent quark massが与えられたとしても、陽子と中性子の
質量差を出すのはまた別の問題で、難問でしょう。
369ご冗談でしょう?名無しさん:02/03/13 19:54 ID:???
もし中性子と陽子の質量が逆だったら、
どういう宇宙が形成されていただろうか。
370ご冗談でしょう?名無しさん:02/03/13 21:09 ID:???
>>362 情報源を教えてもらえないでしょうか? >>368 の方に説得力を感じるので。

>>369 面白い問題設定ですね。
全ての天体は中性子星かブラックホールという事になりそうですね。
大規模構造は主にダークマターで決定されるから
そっちには影響を与えないような気もします。
371369:02/03/13 21:30 ID:???
>>370
>全ての天体は中性子星かブラックホールという事になりそうですね。

それはないだろ。
372>:02/03/13 21:59 ID:TEWo7XW3
>>367
「押したり引いたり」の部分に適訳ないでしょうか?
373おしえてください:02/03/13 22:00 ID:2dS1qPlf
質問です。
すごく重たいものでも摩擦さえなければどんなに小さな力でも動かすことはできるのでしょうか?
たとえば宇宙空間に静止している一トンの車を1グラムの力ででも押せば、車はわずかでも動くのでしょうか?
374ご冗談でしょう?名無しさん:02/03/13 22:04 ID:???
f=ma
375ご冗談でしょう?名無しさん:02/03/13 22:21 ID:???
>>369
とりあえずβ崩壊できないから宇宙の元素組成が変わるだろうね。
っつーか逆β崩壊が促進されるから、原子が構成されにくくなる?

>>373
あたりまえ。
逆に、動かせないと思う理由は?
376ご冗談でしょう?名無しさん:02/03/13 22:31 ID:???
>>375
水素原子が作られないので単独中性子と
電子が代わりに大量に存在することになるけど、
重水素、ヘリウム以降は安定して存在できるな…
で、質量が集まれば中性子同士の核融合は水素より容易かな。
377ご冗談でしょう?名無しさん:02/03/14 00:26 ID:???
>>372
ドリトル先生読んだことある? あれに「Push-pull」出てくるよ。
その名訳は「オシツオサレツ」
378まんこ:02/03/14 00:30 ID:???
ちょっと待ってよww
おまえらほんとに理科系か?
中性子と陽子が逆だったら、お互いが人間によって
それぞれの名前で呼ばれるだけです。
こんなの記号論以前の話だぞ。
物理板って意外とバカの集団だったんだな。

>>369は今ごろ大笑いだろうな。
379ご冗談でしょう?名無しさん:02/03/14 00:41 ID:???
>>369
無知な>>378の煽りに乗ってみるテスト。
「陽子と中性子の違いは質量だけ」とか思ってるのかな?
380ご冗談でしょう?名無しさん:02/03/14 00:41 ID:???
問題をよく読まずに解答して不正解を食らうこと、多くないですか?>>378
381まんこ:02/03/14 00:45 ID:???
>>379
まだがんばるかw
あのくらいのレベルまで小さいと性質は質量で決まるんでしゅー。
わかりまちたか?
382ご冗談でしょう?名無しさん:02/03/14 00:45 ID:???
名前が「まんこ」だけにネタレスだろうとか言ってみるテスト。
383まんこ:02/03/14 00:46 ID:???
ちなみにこんな名前だけど放医研所属です。
384ご冗談でしょう?名無しさん:02/03/14 00:47 ID:???
>>379
チャージさえも知らないなら逝ってヨシですよ。
385ご冗談でしょう?名無しさん:02/03/14 00:49 ID:???
>>383
放医研所属ってのはあまり物理では自慢にはならないよ。
放医研でまともな物理出したことあるの?
386ご冗談でしょう?名無しさん:02/03/14 00:54 ID:???
>>368
current quark がその後どんな衣を着てvalence quarkになるか、
それらがどうやってn乃至pを作るかは、すべてQCDの問題なので、原理的には
格子QCDで答えが出せるはずです。ところがQCD相互作用はフレーバーに依らない
から、結局nとpの質量差は、電弱を考慮しない限り、始めのudの質量差に起因する
ことになります。
387WIZ:02/03/14 00:58 ID:yqMB0BBu
疑問

エーテル振動ってなに?
388かげお:02/03/14 01:08 ID:???
>>355
あれ?遠方から見ればマイナスになるような……ま、いいや。手抜き説明だし。

>>369
陽子と電子が片っ端から結合して中性子とニュートリノになり、宇宙は中性子
だらけになりそう。
389某医療系大学生:02/03/14 01:17 ID:FtZkpvPk
磁場と電場は違うんですか?
390ご冗談でしょう?名無しさん:02/03/14 01:20 ID:???
>>381
>あのくらいのレベルまで小さいと性質は質量で決まるんでしゅー。

ぶち殺してさしあげましょうか、ど腐れ。ていうか(・∀・)カエレ!
391ご冗談でしょう?名無しさん:02/03/14 01:28 ID:4vqPfd2X
http://www.watch.impress.co.jp/akiba/hotline/20000520/image/vf5d5.jpg

この画像の右側にある紫がちょっとはいった部品は何なのか教えていただけないでしょうか?
392376:02/03/14 01:31 ID:???
電子が余計だったごめん
393ご冗談でしょう?名無しさん:02/03/14 01:42 ID:o4u9D6F3
>>381
量子力学を学んでから出直して来い。ていうかお前はイラン。
394369:02/03/14 01:45 ID:???
>>379
いやあ。単に陽子が不安定に、中性子が安定な状態になったら
宇宙の元素合成の歴史はどのように書き換えられるかな?という
あくまで原子核レベルの思考実験として言ってみただけなので。
そして質量以外の違いがあるからこそこの思考実験も
おもしろくなると思うのですが。違いなかったらやる意味ないし。
あ、378みたいな頭のかわいそうな人は無視してあげてください。

>>388
でも陽子と中性子が核力で結合したら、
現実の宇宙同様陽子も安定に存在できますよね。

で両者のチャージの違いが元素合成に及ぼす影響はどの程度のものなのかな、と。

質問スレで出すネタじゃなかったかも。すいません…
395理学部:02/03/14 01:55 ID:ys2w51hx
>>391
アルミ電解コンデンサです
パスコンでしょうきっと
396ご冗談でしょう?名無しさん:02/03/14 01:59 ID:kEJd40cX
をっさんって最近見ないけど、どこへ行っちゃったの?
あの世?
397ご冗談でしょう?名無しさん:02/03/14 02:01 ID:???
>>395
ありがとうございます。
秋葉原に行って探して見ます。(売ってるかわかりませんが・・・)
398ご冗談でしょう?名無しさん:02/03/14 02:09 ID:kEJd40cX
>>397
壊れちゃって取り替えるんだとしたら、
形状は違ってもいいから、
○○μFって書いてあるのが同じヤツを探せばいいよ。
399ご冗談でしょう?名無しさん:02/03/14 02:18 ID:???
>>398
ご親切に、本当にありがとう。助かります。
(こういうことには疎いもので)
400学部2年:02/03/14 02:27 ID:VFSWn0+6
重力は相対論的質量にかかるんですか?それとも静止質量にかかるんですか?
等価原理を考えると静止質量にかかると思うんですけど、光も重力で曲げられるらしくてそこがよくわからないんですが。
401まんこ:02/03/14 02:34 ID:???
おいおいまだ気づかないの?
今研究室でみんな爆笑してるよ。
もうここにはこない。
なぜかって?っふ。聞くなよ(涙
402ご冗談でしょう?名無しさん:02/03/14 02:36 ID:???
>>400
エネルギー運動量テンソルでは?
403祭りですね?:02/03/14 02:36 ID:???
祭りの会場はここか?     腕が鳴るぜ      踊るぞヤロウども!
                   ____
       ∧         ∧ /;;;;;;;;;;;;;;;;;;;;;i\ , -``-、         , -``-、
        / ヽ        ./ .∧ \;;;;;;;;;;;;;;;;;;;/ ヽ   \      /     )
     /   `、     /   ∧  `、;;;;;;;;;;;;;;/  \    \    /    /
    /       ̄ ̄ ̄    ヽ           ヽ    ̄ ̄     /
  ( ̄ ̄ ̄ ̄ ̄祭り命 ̄ ̄ ̄ ̄) ̄祭り命 ̄ ̄ ̄)  ̄祭り命 ̄ ̄ ̄)
   / ̄ ̄ ̄ ̄ ̄ ̄ ̄ ̄ ̄ ̄ ̄.\ ̄ ̄ ̄ ̄ ̄ ̄ \  ̄ ̄ ̄ ̄ ̄ ̄ \
  /:::::::::: ヽ-=・=-′ ヽ-=・=-  /=・-   -==・-  |・=-   -=・=-  |
  ヽ:::::::::::    \___/    /  \___/   /  \___/   /
    ヽ__::::::::::::::  \/     /::::::::  \/    /:::::::  \/    /
  /\\          //\\       //\\       //\
 |   \\       //  |\\    //  |\\    //  |
 |     ̄ ̄ ̄ ̄ ̄/ '´) ̄  | ̄ ̄ ̄ ̄/ '´) ̄  | ̄ ̄ ̄ ̄/ '´) ̄  |
  \         /-‐´   |     /-‐´   |     /-‐´   |
   ヽ        丿___/     丿___/    丿___/
   |  ̄ ̄ ̄ ̄  ̄|      | ̄ ̄  ̄|      |  ̄ ̄  ̄|      |
   |______|____|____|____| ____|____|
  /         \      \    \      \     \      \
  |          |        |    |        |    |        |
  \_____/\_____/__/\_____/__/\____/
404400:02/03/14 02:49 ID:VFSWn0+6
>402
わかりません。勉強して出直してきます。
あ、一回切ったんでIDちがってるかもしれません
405391:02/03/14 03:20 ID:???
先ほどはありがとうございます。
コンデンサの表面に上から

  10
  os-con
  100
  16

と書いてあったのですがこれは何を表しているのでしょうか?
自分で調べてみた結果下記のように思ったのですが・・・

  10(???)
  os-con(製品名?)
  100(電気容量(μF)?)
  16(耐圧(V)?)

重ね重ねもうしわけありません。
406337:02/03/14 04:23 ID:YPbFapTq
>>339
自然単位を持ち出したのは勇み足だった様です。
益川敏英と言う人が重力自然単位と名付けてG=1と置いていたのですが、一般的なものだと勘違いしていました。
しかしプランクの長さ√((Gh/2π)/c^3)、プランク時間√((Gh/2π)/c^5)、プランク質量√((ch/2π)/G)は
一般的ですよね。
本題に戻しますと、いろいろな場でhではなくh/2πが使われるように、
Gよりも4πGを使った方がいいのではと言うことです。
Gが入っている式ではかなりの割でGにπが掛かっています。
407ご冗談でしょう?名無しさん:02/03/14 04:29 ID:???
かかりません。
408リーマン幾何学さん:02/03/14 06:31 ID:???
>>405
OS-CONはサンヨーの低ESRコンデンサーです。
http://www.sanyo.co.jp/compo/os-con/
低ESRコンデンサー(内部抵抗が非常に小さい?)特殊なコンデンサーです。
この手のだと、松下のSP-CAPとか海外の奴(名前忘れた)のタンタルとか
があります。
リニアテクノロジーの電源用ICとかで使うことがあり、
リニアのICのデータシートにたまに低ESRコンデンサーの説明があります。
リニアのサイト行けばPDFのデータシート落とせます。
405の予測はおおよそ当たっているかと10はESR値なのか...
ちゅうこって。。。
409リーマン幾何学さん:02/03/14 06:41 ID:???
>>405
ひょっとして391のビデオカードの
修理自分でしようとしてます?
ここらへんの部品はたぶんパスコンではないですよ。
コンデンサ取り囲んでるICはたぶん電源ICで
その下にコイル2つありますよね。
これ明らかに、電源回路でコンデンサは電源ICに付随する
部品、ここらへん使われてるコンデンサは秋葉とかで
手に入る代物じゃないですよ。特殊なもので、値も張ります。
これと全く同じコンデンサ使ってるジャンク見つけるかしないと
難しい。値が一緒だからといって変なコンデンサつけると
確実にボート逝ってしまいます。
410身体・健康板から来ました。:02/03/14 08:26 ID:MteTCp+P
身体・健康板から来ました。
磁化水って本当に効くんですか?活性水素が効くって本当ですか?
あるケミストさんという方が効かないと答えていますがどちらが正しいの?

【アルカリイオン水】奇跡の水?
http://ton.2ch.net/test/read.cgi/body/1014895802/l50
411ご冗談でしょう?名無しさん:02/03/14 08:34 ID:MscKOTHA
>>410
生物板か医歯薬看護板で聞いた方がいいと思われ。
412391:02/03/14 10:21 ID:???
>>409
ああ、そうなんですか・・・ご想像どおり自分で修理してようとしてました。
一応、探してみますが、無かったら諦めます。
ありがとうございました。
413ご冗談でしょう?名無しさん:02/03/14 11:16 ID:oEfFThkx
激論 「宇宙が出来る前の世界とは」(長文)より

>時間を視覚的にとらえて簡単にイメージしてみるとどうなるのかな?

>時間を強引に目に見えるものに置き換えてみて

>光のスピードが(見た目)速くなれば 空間が伸び 時間が縮んでるってこと?

これは正しいんでしょうか?
414368:02/03/14 11:46 ID:???
>>386
「電弱を考慮しない限り」というのは今の問題では不適当だと思いますが、
そういう説明ならおおむね納得です。
>>362がMn-MpはMd-Muに等しいという主張に読めたからクレームつけただけ
415ご冗談でしょう?名無しさん:02/03/14 12:02 ID:???
>>413
よくわからんが間違っていそう
416諸先輩方へ。:02/03/14 16:11 ID:???
なんで光速を超えるとその空間だけ時間が遅くなる(早くなる?)んですか?
わかりやす〜く簡単に教えていただきたいです。
ってか、レスるのメンドーだと思うんで、分かりやすく解説した本やらHPやらでもいいので教えてください。
417ご冗談でしょう?名無しさん:02/03/14 16:16 ID:???
誰がいつどこで光速を超えると言ったんだ?
418 ◆ZuraD6Pc :02/03/14 18:19 ID:lGAVDAd+
この春大学の物理学科に行きます。
大学に入る前のこの時期、また、入ってからは何をしておくべきですか?
「あの時これをやっておけばよかった。」
「もっと〜を頑張っとけばよかった。」
というような、生の声を聞かせていただきたいです。
419ご冗談でしょう?名無しさん:02/03/14 18:21 ID:???
>>416
そういった内容を扱うサイトを物理板ではこう言っています
http://cheese.2ch.net/test/read.cgi/sci/972668860/l50
420ご冗談でしょう?名無しさん:02/03/14 21:23 ID:2Qo4Xieb
410です。一日待ちましたがまともな回答が無いようですね。
こちらも411のように化学板に責任転嫁ですか?あれだけテレビなどのマスコミで
報道されているのに無視を決め込んで自分たちの世界に閉じこもっているなんて、
ちょっと一般常識では考えられませんね。この板の住人はオウムと大差ないと
思わざるを得ません。
私が言っている事が間違っていると思ったらさっさと健康板のスレに来て
あなたたちの正しさを証明しなさい。君達が論破されて吠え面かくのは
わかってますけどね。
421420をポア:02/03/14 21:27 ID:???
>>420
オウムですが何か
422ご冗談でしょう?名無しさん:02/03/14 21:27 ID:???
(プ
423ご冗談でしょう?名無しさん:02/03/14 21:33 ID:???
身体・健康板で論破されてやって来た420が吠え面かいているのはこのスレですか?
424ご冗談でしょう?名無しさん:02/03/14 21:52 ID:???
(マルチポストは無視しようね)
425超文系:02/03/14 23:34 ID:???
近くの山は緑なのに、遠くの山は青く見えるのはなぜですか?
426ご冗談でしょう?名無しさん:02/03/14 23:56 ID:kVe2fspz
>>425
空が青く見えるのと同じ理由じゃないかな……
427ご冗談でしょう?名無しさん:02/03/15 11:34 ID:oZS8dgcT
>>415
自分の場合、物理の事知らないから
なんとなく思っただけなんだけど、
よくよく考えると
まったく逆に思えてきたんで
ここきたんすよ
物理でいいんだよね?
428ご冗談でしょう?名無しさん:02/03/15 15:16 ID:???
>>427
>光のスピードが(見た目)速くなれば
ここでもう「何をか言わん哉」なの
光のスピードが変わらないように時空が伸縮するんだよ

429 :02/03/15 17:49 ID:EpDIMXUw
この春大学の物理学科に行きます。
大学に入る前のこの時期、また、入ってからは何をしておくべきですか?
「あの時これをやっておけばよかった。」
「もっと〜を頑張っとけばよかった。」
というような、生の声を聞かせていただきたいです。


430Who:02/03/15 19:41 ID:gQMnzWKq
>>429
そんなに気負わなくても大丈夫。
4年間コンスタントにしっかりやればよい。
いい大学生活を。
431ご冗談でしょう?名無しさん:02/03/15 21:15 ID:???
>>429
 ・「量子力学の数学的基礎」
 ・ジョン・フォン・ノイマン
 ・井上健
 ・みすず書房
 ・1988/11出版
を読め。
理解できなかったら迷わず退学手続きをしろ(w
432ご冗談でしょう?名無しさん:02/03/15 22:10 ID:oeomE7/o
物が見えるのは物に反射した光が網膜に当たるから見える。
って事は光より速いロケットに乗って地球から遠ざかりながら望遠鏡で地球を見るとだんだんと過去の地球が見えてくる?
どうなんでしょう?教えて下さい。
433ご冗談でしょう?名無しさん:02/03/15 22:57 ID:???
>>432
光より遅いもので考えようよ
たとえば、コマ撮りした写真を次々とベルトコンベアーに載せて運ばせたとする
ベルトコンベアーの回る速さより速く移動して写真を連続的に見てゆくと
逆再生されたように見える

光より速く移動して光を見たい!とか言うのは、そりゃ無理ですぜダンナ
434ご冗談でしょう?名無しさん:02/03/15 23:07 ID:???
>>433 その説明だと逆再生が可能なように受け止められるぞ
435ご冗談でしょう?名無しさん:02/03/16 06:45 ID:???
>>432
そりゃあ超えられればね。当たり前じゃん。
436ご冗談でしょう?名無しさん:02/03/16 09:51 ID:JHuHkYCn
物理板の看板ってよく変わるよね。
今回はマックスウェルとそのマックスウェル方程式、
前はパウリとパウリ行列(標準形式)、他にもシュレディンガーと
シュレディンガー方程式だったり。
次回はなんだろ?アインシュタインとアインシュタイン方程式かな?
それともボルツマンとエントロピー方程式かいな?
437ご冗談でしょう?名無しさん:02/03/16 10:37 ID:ZBVg/TSf
t'Hooftってなんて呼んでますか?
438ご冗談でしょう?名無しさん:02/03/16 11:35 ID:???
オランダ人は ェトホゥーフ(ト) って言ってた。様に聞こえた。
439ご冗談でしょう?名無しさん:02/03/16 14:30 ID:???
エーテルってなんですか?ターボとかスーパーとかあるんですか?
440DQN大阪人@豊中:02/03/16 15:58 ID:???
当方、大学新3回生です。単位のかかったレポートが終わりません。
助けてください。解決したらスレを削除します。どうかよろしくお
願いいたします。ほんとによろしくお願いします。わらをもつかみ
たい気分なんです。問題は以下のとおりです。

1.ある状態ケットΦに対してある物理量Fを観測するとはいかなること
であるか、この物理量の演算子の固有状態を用いて説明せよ。観測の
結果、その物理系はどのような状態になるかも含めて述べよ。
2.x=-aとx=aに無限に高い壁があるx軸方向の1次元系を考える。それら
の壁の間にある質量mの粒子の運動を考えよう。
(a)この粒子の従うシュレーディンガー方程式を解き、その固有状態を
求め、エネルギー固有値の一般式を示せ。さらに、エネルギーの低い
方から1、2、3、4番目の固有状態の確率分布を位置(x座標)の関数として
図示せよ。波動関数のxに関する偶奇性について述べ、その起源について
説明せよ。
(b)時刻t=0に粒子がデルタ関数δ(x)の波動関数で表される状態になって
いたとする。この状態と設問(a)でもとめた4つの固有状態との重なりを
求めよ。
(c)で求めた固有状態の波動関数を時間発展の部分も含めて示せ。
(d)設問(b)の状態が時間発展した時、時刻t(>0)にはどのような
波動関数で表されるか示せ。

以上です。なにとぞご協力お願いいたします。
441ご冗談でしょう?名無しさん:02/03/16 16:00 ID:???
>>440
>助けてください。解決したらスレを削除します。
バカモン!何考えてんだ!
442DQN大阪人@豊中@440:02/03/16 16:01 ID:???
すいません。解決したらスレを削除します、って言うのは大嘘です。
これはスレを立てようとしたら立てすぎです、って立てられなかったやつを
そのままコピーしたものですから。不愉快な思いをさせてしまい、申し
わけありません。なにとぞお願いいたします。
443カテジナ・ルース:02/03/16 16:21 ID:g1IdM6H3
>>440
量子井戸関連のレポート?
とりあえず、量子井戸 演習
で検索かければ
444kkkk:02/03/16 18:45 ID:???
何故帯電した導体同士を伝染によって繋げると同電位になるのですか?
細かく知っている方、どうか教えてください、、、
445ご冗談でしょう?名無しさん:02/03/16 19:02 ID:+Ryv/Khq
>>444
なんというか、電線で繋いでいるのにいつまでも違う電位のままの方がおかしいと思いませんか。
例えば、水位の違う水槽を管で繋ぐと、同じ水位になるでしょう。
446kkkk:02/03/16 19:28 ID:???
<445
あの、僕の考え方なんですが、「導線があることにより自由電子が電気力により
り移動し、その電子自身が作る電場は必然的に電子に対する電場を減らしていく。
何故電子が動くかというと電場があるから、つまり電子の移動が終わり、
その場の電化分布が決まってしまった時には、電子が動かない、つまり
内部の電場はゼロ、故に、導体内外の任意の空間で電場による線積分はゼロ。
したがって同電位である」というものなのですが、あっているのでしょうか?
あと、445さんの水槽の例え、非常に分かりやすかったです。ありがとうございました!!
447kkkk:02/03/16 19:29 ID:???
長くて読みにくい文を失礼しました
448ご冗談でしょう?名無しさん:02/03/16 19:33 ID:???
そうそう。それでいいよ。
449kkkk:02/03/16 19:37 ID:???
<448
ほっとしました、、、。ありがとうございます^^
450kkkk何度もごめんなさい:02/03/16 20:02 ID:???
一つの導体がもう一方の導体に囲まれている場合、内側の電荷がQ、囲んでいるほうの内側の電荷をqとすると、Q=qとなることについて質問です。
これをコンデンサーと見ると、電気容量は二つの導体の幾何学的な配置と
極板によってのみ決まるの事から、どちらかを基準にとっても
容量は同じ、この事実からQ=qとなるのでしょうか?数式ではそうなったのですが
不安です。何度も質問してすみません、、、
451kkkk何度もごめんなさい:02/03/16 20:04 ID:???
間違えました!!Q=−qでした!すみません!!
452ご冗談でしょう?名無しさん:02/03/16 20:54 ID:2SAu6XfJ
「カールルイスと亀が居ます。亀はカールルイスより少し前の所に居ます。今からこのカールルイスと亀が競走します。カールルイスが亀が始めに居た地点まで走るとその間に亀は前に進むのでカールルイスは亀を追い抜くことは出来ません。」

コレについてどう正しく説明しますか?
453ご冗談でしょう?名無しさん:02/03/16 21:33 ID:???
「カールルイスと亀が居ます。カールルイスは亀より少し前の所に居ます。今からこのカールルイスと亀が競走します。亀がカールルイスが始めに居た地点まで走るとその間にカールルイスは前に進むので亀はカールルイスを追い抜くことは出来ません。」

コレについてどう正しく説明しますか?
454sage:02/03/16 22:25 ID:???
まず、カールルイスは亀より速いのかどうかだ。
455Who:02/03/16 22:54 ID:XjflV7Om
>>452
出題者の問題の意味、意図が始めよく分からなかった。

カールルイスが亀のいた位置まで来ると亀は少し進んでいる。その後
またカールルイスが亀がいた位置まで来たら亀は少し進んでいる。
カールルイスが亀より早く走るとしても、この論理だとカールルイスは
限りなく亀に近づくがいつまで経っても追い抜けないことになる。
さて、この論理のどこがおかしいでしょうか?
と言う事か。
456ご冗談でしょう?名無しさん:02/03/16 22:59 ID:???
カールルイスが亀が始めに居た地点まで走るとその間に亀は前に進む
…はOK
のでカールルイスは亀を追い抜くことは出来ません
…何故?
457ご冗談でしょう?名無しさん:02/03/16 23:15 ID:???
ルイスが亀に追いつく 1 秒前にはまだルイスは亀に追いついていません
ルイスが亀に追いつく 0.1 秒前にはまだルイスは亀に追いついていません
ルイスが亀に追いつく 0.01 秒前にはまだルイスは亀に追いついていません
ルルイスが亀に追いつく 0.001 秒前にはまだルイスは亀に追いついていません

…ってことを示してるだけ。くだらない。
458457:02/03/16 23:25 ID:???
ルルイスって… (海底神殿か?)
鬱だ。
459あほです。:02/03/16 23:50 ID:???
実験のレポートを書いています。グラフを関数でフィットしたいのですが、
現在はエクセルしか使えません。指数関数や2次関数を組み合わせた複雑な
関数形をグラフにして表示したいのですが、どうすればよいのでしょうか
460ご冗談でしょう?名無しさん:02/03/17 00:01 ID:7O5wSNmw
>>452
古代ギリシャの詭弁「アキレスと亀」が最近では「カールルイスと亀」
になったの?
カールルイスと亀が逆だけどこれはネタなの?
461ご冗談でしょう?名無しさん:02/03/17 03:20 ID:???
文盲が多いなあ。イヤニナチウワ
462437:02/03/17 04:23 ID:???
>438
有難う御座いました.これで思う存分呼べます.
463ご冗談でしょう?名無しさん:02/03/17 05:28 ID:b8ktqmbz
>>462
トフーフトでヨシ
464ご冗談でしょう?名無しさん:02/03/17 15:45 ID:WkEO/3u2
上のバナー、rot とか div とかはローマンにしようよ。はずかしい。
465ご冗談でしょう?名無しさん:02/03/17 15:45 ID:j3aljzgO
>詳しくは覚えてないが、磁力の反発による、永久回転運動は可能だったと思うが・・・

というカキコをしたところ、

>磁力の反発による、永久回転運動は可能
物理板にお越しください。目を覚ましてあげます。

このようなレスをもらいました。
http://www.oyama-ct.ac.jp/D/kinnoken/study_result_2.html
こんなおもちゃがあるのを知っていたので書いたのですが・・・
これって物理法則に反した運動なんですか?もしくはインチキなのでしょうか?
おわかりになる方がいたら教えてください。
466465:02/03/17 15:54 ID:j3aljzgO
書いたばかりでスイマセン。レスもらって問題が解決しました。
よく読んでなかったようで、早とちりでした。お騒がせしました。
467ご冗談でしょう?名無しさん:02/03/17 16:16 ID:???
>>464
おいらは∇のほうがいいな
468ご冗談でしょう?名無しさん:02/03/17 17:31 ID:???
>>465-466
どういうレスで解決したか知らないけど、
摩擦がなけりゃなんだって永久に回り続けるよ
負荷かけたらあっという間に止まるけど
469kaze@物理1(力学):02/03/17 19:16 ID:M42B4bk4
ハシゴを壁に立てかける問題ありますよね?
その問題で、、ハシゴの重さw、長さ2Lとして、ハシゴと鉛直下向きとの角(←ハシゴと壁)をθとする。
ハシゴの下端Aと床との静止摩擦係数μとして、壁とハシゴの上端Bとの摩擦はないとする。
ハシゴがうける垂直抗力をN(A)、N(B)とし、静止摩擦力をFとする。として、
(1)ハシゴが滑らないためには、θはどのような条件か?
という問題を解いてみたのですが、tanθ≦2μとなりました。
角度とμの関係式ってちょっと妙な気がします。
もしよろしければ、答えだけでも教えて下さい。

後、これとは別に、上の問題で、
○重さWの人がハシゴの下端からχの位置の時、ハシゴがすべらないような
θの条件。
○その人がハシゴの上端まで行ってもハシゴが滑り落ちない条件
っていうのも答えのみでいいので、教えていただきたいです。
よろしくお願いします。
470ご冗談でしょう?名無しさん:02/03/17 20:07 ID:d7glZS2m
無重量間にろうそくはなぜまるくなるのかと、箱の中に物を入れて
落下させた場合になぜ無重量状態になるのかがよくわからないので
詳しく教えてもらえませんか?お願いします。
471ご冗談でしょう?名無しさん:02/03/17 20:24 ID:???
>>469
答えなんかどうでもいいから考え方を書け
472kaze@物理1(力学):02/03/17 20:32 ID:M42B4bk4
水平成分について→F=N(B)、、、1
鉛直 … →N(A)=w、、、2
Aまわりのモーメントは、
N(B)2Lcosθ=wLsinθ、、、3
ハシゴが滑り出す時は、最大静止摩擦力で、F=μN(A)であるから、
1とこれよりN(B)≦μN(A)、、、4
2と4→3して、計算すると、、tanθ≦2μとなりました。
473 ◆QM0vNwy6 :02/03/17 22:08 ID:???
原子は陽子と中性子からなる原子核と、原子核を取り巻く電子から
構成される、って高校の時習ったんですけど、原子核と電子の間の
空間には何もないのでしょうか?完全な真空なんでしょうか?

それと、金属の場合にあるという、自由電子というのは
この空間を自由に飛び回ってるんですか?
474ご冗談でしょう?名無しさん:02/03/17 22:52 ID:WkEO/3u2
>>470
ろうそくが丸くなるってのは嘘だと思うけど。火が丸くなるのは、
燃料が上に上っていかないで、芯の周りにたまるから。

箱云々は、箱の中にいる人にとっては、地面(箱の底)も
同じ速度で落ちていくんだから(ガリレイの落下の法則)、
ふわふわ浮いているような感覚になるわけ。
475ご冗談でしょう?名無しさん:02/03/17 22:54 ID:???
電気って何ですか?
476ご冗談でしょう?名無しさん:02/03/17 23:12 ID:wClnquaB
音は波で虹の色も同じ波で、
波長が違うだけだと聞いたのですが、
ほんとですか?
光は粒だとも聞いたのですが、
じゃ、虹の色の波と光の粒は違うものなのですか?
477ご冗談でしょう?名無しさん:02/03/18 00:04 ID:YV4es2kY
すっとぼけた質問で厨房ですが、
Aの二乗って、どうやって表記するのでしょうか??
A^2 でしょうか???
478ご冗談でしょう?名無しさん:02/03/18 00:33 ID:???
>>477
そうです。
479ご冗談でしょう?名無しさん:02/03/18 01:21 ID:???
>>478
^はビットごとのXORだと思うのですが。とか煽ってみるテスト。
480ご冗談でしょう?名無しさん:02/03/18 01:31 ID:???
>>476
そんなデタラメ誰に吹きこまれたの?
481ご冗談でしょう?名無しさん:02/03/18 02:46 ID:???
べき乗は ** だろ、とか言ってみる。
あとは<sup>2</sup>か。
482ご冗談でしょう?名無しさん:02/03/18 14:25 ID:FpIucLxj
縦波と横波の基本的性質として
横波:固体など、比較的密度の高い物質を伝わる
というのは超有名な話だけど、なんで光は空気中、真空中を伝わるのですか??
483ご冗談でしょう?名無しさん:02/03/18 14:32 ID:???
>>482 光の伝播の場合、空間自体が”媒質”だから
#ほら、誰か俺に突っ込め
484ご冗談でしょう?名無しさん:02/03/18 14:33 ID:G9mJLTTe
パソコンの光学マウスってどういう原理で動きを感じ取っているのか教えていただけませんか?
485ご冗談でしょう?名無しさん:02/03/18 15:20 ID:gG22oJl5
ちょっとした質問なんですが、実験で誤差無しで対象を認識、観察することは不可能、
みたいな定理を誰かが作ってましたよね?
これが本当なら何故実験屋は発狂しないんですか?
486ご冗談でしょう?名無しさん:02/03/18 15:23 ID:???
>>485
逆です。誤差があるから幸せー♪
487ご冗談でしょう?名無しさん:02/03/18 15:24 ID:???
>>485
誤差と友達だから。
488ご冗談でしょう?名無しさん:02/03/18 16:21 ID:???
誤差がなかったら発狂するのは理論屋のほうだと思われ
489ご冗談でしょう?名無しさん:02/03/18 16:25 ID:DKmYW41i
>>484
光学マウスっつってもいくつかあるんですが、昔のSUNのはマウスパッドの
格子縞を数えていました
最近のMSのアレの仕組みは知りません
490ご冗談でしょう?名無しさん:02/03/18 16:31 ID:???
>>490
マウスパッドが斜めになってると、真横に動かしたつもりでもマウスポインタは
斜めに動いちゃうのが鬱陶しくて大嫌いだった

491 ◆QM0vNwy6 :02/03/18 20:58 ID:???
473の質問したものですけど、ご意見ほしいです。
492ご冗談でしょう?名無しさん:02/03/18 21:08 ID:???
相対性理論に関する質問です

静止した物体の引力とそれが準光速で移動している時の引力は違うのでしょうか?
観測点を静止した点に置くと準光速で移動している物体の表面の引力は
大きく見えるような気がするのです
しかし移動している物体上で見ると引力は変わらないようにも見えるのです
493ご冗談でしょう?名無しさん:02/03/18 21:13 ID:???
>491
基本的には473でいいかと。
494ご冗談でしょう?名無しさん:02/03/18 21:31 ID:???
>>492
運動量の違いによって物体の重力効果は違うか、と言う質問でスか?
495ご冗談でしょう?名無しさん:02/03/18 21:37 ID:GYTiV3l8
@光は質量を持つのですか?
電子は質量を持ちますよね。
そういう意味で。

A光子は電子等と同等の粒子と扱って良いの?
つまり光子は素粒子という事で良いのでしょうか?


全然わかんないです。2つの質問誰かお願いします。
496ご冗談でしょう?名無しさん:02/03/18 21:38 ID:???
>>495
光子は質量0の素粒子です
497携帯板の90:02/03/18 21:45 ID:UdykB3Zu
携帯板からきました文系人です。こんなカキコがあったのです。


【新事実】Jホンはエリアが狭いんですか?【驚き】
http://cheese.2ch.net/test/read.cgi/phs/1016172312/

89 名前:非通知さん 投稿日:02/03/18 21:20 ID:Xq0GXlMy
まぁJは周波数が2倍な分、電波の室内、地下街への進入も2分の1な
ので入りにくいのは仕方がないね。PHSはもっと周波数高いけど基地
局が低いところにある分救えるけど


質問
周波数と進入は比例するのですか?
基地局の位置と進入も比例するのですか?
498ご冗談でしょう?名無しさん:02/03/18 21:49 ID:PeoaHPcK
>>496
質量0っていうのは、どういう意味ですか?
499ご冗談でしょう?名無しさん:02/03/18 21:59 ID:GYTiV3l8
>>496
しかし相対論だと質量=エネルギーですよね。
素粒子は全てそのように考えてると思います。
つまりエネルギーを持ってれば、質量があるという事では
ないのですか?
500ご冗談でしょう?名無しさん:02/03/18 22:00 ID:GYTiV3l8
500
501ご冗談でしょう?名無しさん:02/03/18 22:03 ID:m9lWpb8b
質量があるということが存在することだとききました。光って質量がないけど存在してないのでしょうか
502ご冗談でしょう?名無しさん:02/03/18 22:04 ID:m9lWpb8b
501のごめん 499読んでなかったです

503ご冗談でしょう?名無しさん:02/03/18 22:05 ID:???
>>498
質量の定義はご存知ですか

>>499
>質量=エネルギー
違います

>>501
違います
504ご冗談でしょう?名無しさん:02/03/18 22:06 ID:???
>>501
そんな事より
>質量があるということが存在することだ
こっちに疑問をもてよ(w
505ご冗談でしょう?名無しさん:02/03/18 22:08 ID:???
>>494
物体の重力効果ではなくてニュートンの重力定数そのものです
例えば二つの鉄球をバネで繋いでおいて
静止している時引力で1mm縮んだとして
運動している鉄球上の観測点から見ると1mmのままに見えますが
静止した観測点から見ると2mm縮んで見える(質量増大)
ように思えるのです
506ご冗談でしょう?名無しさん:02/03/18 22:20 ID:???
そんなんでいちいち定数を変化させたら
それはもう定数じゃないだろ
507ご冗談でしょう?名無しさん:02/03/18 22:26 ID:???
非相対論的な場合の方程式を
むりやり相対論的な場合にあてはめれば当然歪みは出る
508ご冗談でしょう?名無しさん:02/03/18 22:37 ID:???
で結局1mm縮んで見えるんですか
それとも2mm?
509ご冗談でしょう?名無しさん:02/03/18 23:06 ID:???
>>508
簡単に答が出せるものとは思えないんですけど。
バネ定数わからんけど重力で1mm縮めようとすると、えらく大きい鉄球になるんじゃないか?(それとも高密度か)
そんな大きなものを一般相対論の方程式でとくのかな?
俺にはできないのでsage
510ご冗談でしょう?名無しさん:02/03/18 23:10 ID:???
細かい事気にすんなよ、1mmとか2mmとか。
神経質なんじゃないの(w
511ご冗談でしょう?名無しさん:02/03/18 23:18 ID:GYTiV3l8
>>503
>>質量=エネルギー
>違います
だって、質量の事何GeVとか言うじゃない。
これはエネルギーでは無いの?
512ご冗談でしょう?名無しさん:02/03/18 23:20 ID:???
2個の巨大な鉄球がバネで連結されている構造物だよな、これ。
その運動を考えるんだろ?
>>510はこの計算がわかるの?
513ご冗談でしょう?名無しさん:02/03/18 23:31 ID:???
>>511
質量はエネルギーだけどエネルギーは必ずしも質量ではない
514ご冗談でしょう?名無しさん:02/03/18 23:31 ID:???
>>511
あのなぁ。エネルギーの一部に質量エネルギーが含まれるの。
だから断じて両者は等号で結ばれる物ではない。

よくE=mc^2という式を用いて説明が行われるが、
これは静止物体に対する式。運動する物体に対しては
E=√((mc^2)^2+(pc)^2)が正しい。pは運動量な。

相対論的質量なんて無駄なものはもう使われてないの。
515ご冗談でしょう?名無しさん:02/03/18 23:33 ID:???
>>505
ん? ローレンツ収縮のことを言っているのか?
516ご冗談でしょう?名無しさん:02/03/18 23:49 ID:???
>>515
そじゃなくて観測点によって重力定数が変わって見えるんじゃないかと
静止した観測点と動いている鉄球上の観測点と言う意味です
517ご冗談でしょう?名無しさん:02/03/18 23:49 ID:GYTiV3l8
>>513
中々教育的で・・・w
つまり光子はエネルギーだけって事ですね。
その原理はここで(簡易的にでも)説明可能ですか?
できればお願いします。

>>514
光子の場合は(mc^2)^2の項が無いから質量が0なのですか?
どうして無いんですか?
518ご冗談でしょう?名無しさん:02/03/18 23:58 ID:???
>>516
ローレンツ収縮でないなら、どういう理由で縮んで見えると思ってるの?

>>517
原理と言われても、光子はそういう性質ですとしか言いようがないのですが。
519ご冗談でしょう?名無しさん:02/03/19 00:06 ID:joHwcGWZ
>>518
そういう性質を示す計算があって(式かな?)、
何が質量を0にさせているのか?です。
だって他の素粒子の場合は(基本的に)すべて質量は
あるのでしょう? 光子の場合とは何か違いがある筈です。
その違いを教えて欲しいです。
520ご冗談でしょう?名無しさん:02/03/19 00:10 ID:???
質量は方程式にでて来るただのパラメータ
521516:02/03/19 00:15 ID:???
>>518
ですから>>505に書いてある通りバネが引力で余計縮まないかと言うことです
522ご冗談でしょう?名無しさん:02/03/19 00:33 ID:???
へえ、なんで重力だけ変化させるの?
523ご冗談でしょう?名無しさん:02/03/19 00:34 ID:joHwcGWZ
>>520
では、なぜ光子だけそのパラメータが0でなくてはならないのか?
という事です。他の素粒子は質量を持つのに。
524ご冗談でしょう?名無しさん:02/03/19 00:37 ID:???
>>523
それに答えられるのは神だけでしょう。
525:02/03/19 00:40 ID:???
神ですが何か?
526ご冗談でしょう?名無しさん:02/03/19 00:41 ID:joHwcGWZ
>>524
パラメータなんだから本来何を入れたってよい。
ただ光子の場合、何かの条件によってそれが0に拘束されるのでは?
その条件をお聞きしたいのですが・・・。
527ご冗談でしょう?名無しさん:02/03/19 00:44 ID:???
>>523 ゲージ対称性を担保するため。
質量ゼロなのは光子以外にグルーオンや重力子もある。
528ご冗談でしょう?名無しさん:02/03/19 00:45 ID:???
>>526
他の数字は皆ゼロで無いのに、どうして0だけゼロなんですか?
みたいな問いだと思われ……キミの言っていることは
529ご冗談でしょう?名無しさん:02/03/19 00:50 ID:???
>ゲージ対称性を担保するため。

つうか、単に上手く現象を再現するモデルを作る為だとおもうが。
そのモデルを分析したらゲージ対称性が成り立ったというだけのこと。
530ご冗談でしょう?名無しさん:02/03/19 00:54 ID:joHwcGWZ
>>527
要するにボソン(確か)の事ですよね。
簡潔で解り易いです。ありがとうです。
ただどうしても疑問なのは、どうしてボソンの中で光子
だけが単独で現象するのでしょう。
だって>>526だけ聞くと、
仮想光子と実光子は物理的には同じと言ってる事になりますよね。
531ご冗談でしょう?名無しさん:02/03/19 00:58 ID:joHwcGWZ
>>529
それと日本語の本でこの辺の事、詳しく書いてある書物ってありますか?
つまり光子が質量を持たない事を説明する(結果的にでも)式がでてる本。
倍風館だったかな?九後さんの本とかはどうでしょう?
532ご冗談でしょう?名無しさん:02/03/19 00:59 ID:???
はぁ?
533ご冗談でしょう?名無しさん:02/03/19 01:09 ID:???
>>530
一つの考え方として、全てのパラメータはより統一された理論によって
決定されているはずだというのがある。(多くの素粒子論屋はそう信じている。)
統一理論の考え方だと、もともと、大きな対称性があり、多くの粒子は
質量ゼロだったんだけど、対称性の破れの機構により、小さな対称性だけが残ると考える。
なぜ、光子、重力子、グルーオンだけが質量ゼロのままなのか、
すなわち、それに対応する対称性が残ったかは破れ機構の詳細による。
その詳細は現在でも主要なテーマの一つ。
ワインバーグ・サラムは電弱→電の破れの機構を解明した。
534ご冗談でしょう?名無しさん:02/03/19 01:15 ID:???
といえば聞えはいいが、実質、判らないものを
ヒッグスがらみの所に押し込めただけ。
標準模型は現象論としては素晴らしいが
対称性の破れみたいなdynamicalなところは
やっぱ謎だらけなのだよ、今でも。
535ご冗談でしょう?名無しさん:02/03/19 01:18 ID:joHwcGWZ
>>533-534
ありがとうです。
大変勉強になります。
ところで仮想光子と、実光子(実際の光ですね)は
物理的に何か違うのでしょうか?
今までの話はボソンとしての光子ですよね。
536ご冗談でしょう?名無しさん:02/03/19 01:20 ID:joHwcGWZ
大学の助手さんまでここに来てるとは感激w
まさか院生だったりして・・・。
537ご冗談でしょう?名無しさん:02/03/19 01:21 ID:???
>>534 一応、考え方としての統一論には反対してないようね。
LHCでHiggsセクタの詳細がわかればだいぶと謎は解けると思うけど。
dynamicalなところはSUSYがあると思えばGUTまで延ばせるし。
538ご冗談でしょう?名無しさん:02/03/19 01:31 ID:TpZo+a5C
初めましてこんにちわ。
私は、某大学の数学科の人間です、物理の知識は0であります。
で、問題と言うよりも、質問なのですが。
よく、「原子力発電所が事故になり放射能が漏れると、その周辺は1000年ちかく住めない」っていいますよね。
ところが、原爆が落ちた広島、及び長崎には、原爆が落ちて50年しかたってないのに人が住んでますよね。
これってなんでなんですか?
原爆の放射能は、一時的に爆発しても、長期間には残らないものなのですか?
物理初心者にも解りやすいように教えてください。
(煽りではありません)
539ご冗談でしょう?名無しさん:02/03/19 01:37 ID:???
>>538
主に撒き散らかされる放射能の量の問題だと思われ。
何回も核爆発をやると、セミパラチンスクとかビキニみたいに
人が住めないレベルになってしまう。
あと、核種の違いもあるか。
540538:02/03/19 01:41 ID:TpZo+a5C
すいませんがよく分かりません。
要するに、放射能を一気にまき散らすのが核爆弾で、授受にまき散らすのが原発事故って事ですか?
確かに原爆で人は一瞬で死にますが、原発事故で人は一瞬では死にませんね。

しかし、なぜ、核爆弾の放射能が残らないかの説明がなされてません。
541538:02/03/19 01:42 ID:TpZo+a5C
授受×
じょじょ○
542ご冗談でしょう?名無しさん:02/03/19 01:43 ID:???
>>540 核爆弾の放射能は残るが、原発事故に比べて量が少ない。
量が少なければ問題も小さい。
543538:02/03/19 01:57 ID:TpZo+a5C
>>542
なるほど、単純に量の問題だったんですね。
核爆弾は、爆発が大きい=量が多いと錯覚する。と言うことだったわけですね。
ありがとうございました。

疑問は解決しましたが、引き続き解りやすい解説、待ってます。
544ご冗談でしょう?名無しさん:02/03/19 02:16 ID:VC6qwu7C
原発事故では半減期の長いラジオアイソトープが環境に出る可能性があるが、

核兵器では、爆縮して即発中性子による臨界にしてしまうためか、
長い半減期の核種はあまり残らないと思う。

即発中性子による臨界か遅発中性子による臨界かの違い
核反応に寄与する核種の違いからくる半減期の長短の差
以上の2点による差だとおもわれます。

それと、、「原子力発電所が事故になり放射能が漏れると、その周辺は1000年ちかく住めない」
ってどこからの情報ですか?
というのは全てのフェイルセーフ機能が故障した場合のかなりの不運が重なった原発事故の場合では?
545ご冗談でしょう?名無しさん:02/03/19 02:16 ID:???
>>533
あのさ、多分その人にこんな説明してもわかんないっていうか、
ひっかかってるのは別のところなんじゃないかな?
546ご冗談でしょう?名無しさん:02/03/19 02:36 ID:???
>>535
仮想光子は質量を持った光子、とか言うと又誤解を生みそうな。
不確定性原理によって、エネルギーを真空から無理矢理引き出している状態で、
(エネルギーに応じて、観測不能な短い時間で)再び消滅します。

フェルミオンとしての光子?SUSYのこと?
547ご冗談でしょう?名無しさん:02/03/19 02:46 ID:NagrElcr
影に質量はあるんですか?
確か光にも質量があるんだっけ?(うろ覚え)
548ご冗談でしょう?名無しさん:02/03/19 02:49 ID:???
>>544
チェルノブイリクラスの事故の話と思われ。
でも、1000年ってのはどっから出てきたんだろね?
549ご冗談でしょう?名無しさん:02/03/19 05:27 ID:???
>>521
>>505を読んでも、どういう理由で余計に縮む(あるいは引力が増える)と思っているのか
わからないのですが。
550ご冗談でしょう?名無しさん:02/03/19 05:32 ID:???
>>523
なぜ電子(と陽電子)だけがそのパラメータが511keV/c^2でなくてはならないのか
という疑問は持たないのでしょうか? (w
551549:02/03/19 07:28 ID:???
もしかして光速に近い物体は質量が増えるから、F=GMm/r^2に従って
重力が増えるのでは、ということか?

F=GMm/r^2は非相対論的近似なので、光速に近い物体には当てはまりません。
光速に近付いた物体は質量が増えてブラックホールになってしまう、という
ような心配も無用。要するに>>507さんが早々と指摘している通りです。

他にも、光の質量で混乱している人もいるみたいだし、かように、「光速に近付くと
(相対論的)質量が増える」というのは混乱を生じやすいだけのまるで役立たずの
概念なのですっぱり捨て去りましょう。
552ご冗談でしょう?名無しさん:02/03/19 08:22 ID:???
>>517
>つまり光子はエネルギーだけって事ですね。

これも素人向けの解説の悪いところだが
ああいう解説では運動エネルギーのことを
(しばしば「純粋な」をつけて)エネルギーと言っていたり
(相対論的)質量をエネルギーといっていたりで困りものだ。

「質量=エネルギー」とか書いておきながらすぐその後で
「対消滅により質量は光すなわち純粋なエネルギーとなり〜」
↑こんな感じの文章を書く。これでは素人に質量=エネルギー=光
などというどういう論理関係で結ばれた等号だかさっぱりわからない、
しかし単純でインパクトがある(誤った)関係をすりこんでしまうだけである。

そうすると次は「光の質量は〜」「光って質量がゼロなのになぜエネルギーが〜」
なんていう疑問が当然浮かぶだろう。そうしてここに質問しに来る、というわけだ。

相対論的質量などという単にエネルギーをc^2で割っただけのもの
(しかも551も言っているように、非相対論的な式に静止質量の
 かわりに相対論的質量を入れたからといって正しい式にはならない、
 c=1の自然単位系ではcで割る必要すらなくエネルギーと同等のものになる)
を素人の頭に叩き込むことは非常に迷惑だ。
553ご冗談でしょう?名無しさん:02/03/19 10:33 ID:???
>>552
同感。
これからもヘンな理解のヤカラが絶えないんだろうな...鬱
554ご冗談でしょう?名無しさん:02/03/19 13:40 ID:joHwcGWZ
>>546
電磁気力を媒介するのが仮想光子だと理解してます。
例えばグルーオンなら強い力。
その仮想光子と我々が懐中電灯とかで見る光とは、物理的には
同じものかどうなのかが疑問なんです。
555ご冗談でしょう?名無しさん:02/03/19 13:55 ID:joHwcGWZ
>>552
>質量=エネルギー=光
そんな解釈はしていませんよ。

僕の解釈を整理して書けば非相対論の場合、エネルギーというと
運動エネルギーとポテンシャルです。それらとエネルギーは一応
別個で、質量エネルギーなんて概念は無い。
僕が知りたいのは、とにかく古典で言うその質量というものが何で
あるか(無論必要あれば説明が欲しいけど)では無くて、
質量が0と言うのがどういう事で、なぜそんな事になってしまうのか?
なんです。普通の感覚だと、質量が無いのに粒子(物)として扱う
というのは、どうも違和感があって、またそのような「物質」という
ものが、一体どんな性質を持つのか?って事です。
556ご冗談でしょう?名無しさん:02/03/19 13:57 ID:joHwcGWZ
>>555の修正
〜僕が知りたいのは、とにかく古典で言うその質量というものが
相対論の場合では何であるかでは無くて〜
557ご冗談でしょう?名無しさん:02/03/19 14:04 ID:joHwcGWZ
因みに
>>533-534
の方の説明は良いと思うのですが・・・。

それと
>つまり光子はエネルギーだけって事ですね。
の発言は厳密に言えば、運動エネルギーだけって意味です。
相対論の場合の質量の捉え方が解りませんが。
言い換えれば、「非相対論におけるエネルギーだけ」という意味で
取って頂いても結構です。つまり質量は無いという。。。
多分質問に答えて下さった方も、そのように取って貰ったと思います。
その点に関しては何の問題も無いと考えてますが。
558ご冗談でしょう?名無しさん:02/03/19 14:10 ID:joHwcGWZ
すみません。
再び>>555の修正。
全部載せます。

僕の解釈を整理して書けば非相対論の場合、エネルギーというと
運動エネルギーとポテンシャルです。それらと質量は一応 別個で、
質量エネルギーなんて概念は無い。
僕が知りたいのは、とにかく古典で言うその質量というものが、
相対論の場合何であるかでは無くて、 質量が0と言うのがどういう事
で、なぜそんな事になってしまうのか? なんです。
普通の感覚だと、質量が無いのに粒子(物)として扱う
というのは、どうも違和感があって、またそのような「物質」という
ものが、一体どんな性質を持つのか?って事です。
559ご冗談でしょう?名無しさん:02/03/19 14:14 ID:???
>>558
質問の内容とは関係無いが、

>とにかく古典で言うその質量というものが、
>相対論の場合何であるかでは無くて

古典論と対になるのは量子論だ。
相対論と対比させたいなら古典といわずに非相対論と言ったほうがイイな。

以上、細かい突っ込み終わり。
構わず議論を続けてくれ。
560ご冗談でしょう?名無しさん:02/03/19 14:41 ID:???
>>554
同じ。
確かに仮想ゲージ粒子なら力を媒介するでしょう。でも仮想ミュー粒子とか
仮想パイ中間子とかいろいろあるわけで。勿論物理的性質は一緒ですが。
(除く不変質量…エネルギー保存則を満たす必要がないので)
561ご冗談でしょう?名無しさん:02/03/19 15:31 ID:???
>>558
特殊相対論での質量の定義は (mc^2)^2=E^2-(pc)^2
質量が0とはE=pcであること。

Maxwell方程式から、光のエネルギーと運動量にまさにE=pcの関係があることが
わかっていたので、光子の質量は0ということになる。

なぜそんなことになってしまうかって? 知るか! 光子はそういう性質だとしか
いいようがない。

エネルギーも運動量もあるんだから、質量がなくたって、ものとして
扱っていいじゃない。あるいは、質量はあるけどその値が0だと思えば
少しは納得?

562ご冗談でしょう?名無しさん:02/03/19 16:31 ID:???
>>558 質量が無いのに粒子(物)として扱う

量子論だからです。
563ご冗談でしょう?名無しさん:02/03/19 18:10 ID:???
>>558
素粒子物理学の標準理論によれば、
むしろ全ての素粒子において質量は0であるべきで、質量がある方が
異常なわけです。
で、その質量を与えるのがヒッグス機構(粒子)で、質量はその結び付きの強度を与える
単なるパラメータに過ぎないと。
564ご冗談でしょう?名無しさん:02/03/19 23:10 ID:LShFNC/b
初めましてこんにちわ。
私は、某大学の数学科の人間です、物理の知識は0であります。
で、問題と言うよりも、質問なのですが。
よく、「原子力発電所が事故になり放射能が漏れると、その周辺は1000年ちかく住めない」っていいますよね。
ところが、原爆が落ちた広島、及び長崎には、原爆が落ちて50年しかたってないのに人が住んでますよね。
これってなんでなんですか?
原爆の放射能は、一時的に爆発しても、長期間には残らないものなのですか?
物理初心者にも解りやすいように教えてください。
(煽りではありません)
565ご冗談でしょう?名無しさん:02/03/19 23:13 ID:???
566A.T:02/03/19 23:40 ID:???
ものすごく馬鹿らしくてかつおおざっぱな質問なんですけど、誰か教えてください。

壁によっかかっている棒(壁は摩擦なしで床は摩擦あり)が、すべらない条件って何ですか。
ちなみに問題では、棒と壁の角度θがどういう条件を持つかってなってます。
いや、ほんと基本ですいません。
おおざっぱに教えてください。

567ご冗談でしょう?名無しさん:02/03/19 23:59 ID:???
全部書けよ。それじゃ条件が足らんよ。
568A.T:02/03/20 00:06 ID:???
水平で摩擦のある床から鉛直で摩擦のない壁に重さwのはしごABを立てかける。
はしごと鉛直下向きとのなす角をθとし、はしごの全長は2Lで、その重心Gははしごの中央にあるとする。
また、下端Aと床との間の静止摩擦係数はμであり、はしごのあつさは無視するものとする。

そして、下端Aで受ける垂直抗力の大きさをN1、静止摩擦力の大きさをF、上端Bで受ける垂直抗力をN2とおきます。
で、問題が、
はしごがすべらないためのθの条件
です。よろしくお願いします。
569A.T:02/03/20 00:08 ID:???
↑答えだけでも結構です。
後は自分で考えますんで。
570ご冗談でしょう?名無しさん:02/03/20 00:21 ID:KFGItDdG
>>560-563
ありがとうございます。
何だか解ったつもりになれました。
今後は精進しますです。
因みに僕は数学科の院生なんです(苦藁)
571ご冗談でしょう?名無しさん:02/03/20 00:25 ID:???
最大静止摩擦力は
572ご冗談でしょう?名無しさん:02/03/20 00:26 ID:???
>>570
物理量が虚数云々のスレにも書いてないか?
573A.T:02/03/20 00:27 ID:???
571>>問題文中に書いてありません。
574A.T:02/03/20 00:37 ID:???
自分は、Aを中心とした壁方向の力のモーメントが、壁と逆方向のモーメントよりも小さいとき、
と考えたのですが、間違ってるでしょうか。
ってか全然見当違いだったらはずかしい。
575ご冗談でしょう?名無しさん:02/03/20 02:40 ID:NszbWhIM
シュヴァルツシルト計量のデカルト座標での表示を教えてください。計算するのがめんどくさいです。
576575:02/03/20 02:40 ID:NszbWhIM
あと接続係数も。
577ご冗談でしょう?名無しさん:02/03/20 09:11 ID:???
>>575
(x,y,z)
578Who:02/03/20 10:43 ID:AdAzQbrd
間違っていたらごめんな。
釣り合いの式:
水平方向:F=N2*cosθ
垂直方向:N1+N2*sinθ=W
下端を中心としたときのモーメント:  
   w*L=N2*2*L

どう?
Fはちなみに F=μ*N1 だね?
これらからFとN1とN2を消去したら
いいんとちがうかー?
ほんと違ってたらすんまそん。
579ご冗談でしょう?名無しさん:02/03/20 17:11 ID:9ehBOOJI
可聴域を超えた周波数の音でも音量が多ければ鼓膜が破れたりするの
でしょうか?
580ご冗談でしょう?名無しさん:02/03/20 17:12 ID:9ehBOOJI
可聴域を超えた周波数の音でも音量が多ければ鼓膜が破れたりするの
でしょうか?
581ご冗談でしょう?名無しさん:02/03/20 17:14 ID:WrYM+8YV
>>580
当たり前です。
582ご冗談でしょう?名無しさん:02/03/20 17:14 ID:???
>>580 当然。
583のび犬:02/03/20 17:22 ID:aqUPjmFs


 のび太 「ドラえもん、耳がないのに、よく僕の声が聞こえるよね?」
584A.T:02/03/20 18:03 ID:???
>>578
ありがとうございます!
僕は
鉛直方向のつりあい:F=N2
水平方向のつりあい:N1=w
と考えたのですが・・・上端Bから棒に働く垂直抗力が、
壁に対して垂直に働くと思ったからなんですが・・・
これは間違えなんでしょうか〜?
それにしても返信ありがとうございました。
585ご冗談でしょう?名無しさん:02/03/20 21:42 ID:AUHhcXxT
光速の99.5%で物体が移動するとその長さが10分の1になる。
ここに長さ10光年の巨大な宇宙船があって一日で光速の
99.5%まで加速する。
宇宙船の真ん中を中心にして縮むとすると、普通に考えると船首は後退して
船尾は光速より速く進む。
これがなんで矛盾しないのかおしえてください。
586ご冗談でしょう?名無しさん:02/03/20 22:02 ID:???
>宇宙船の真ん中を中心にして縮むとすると、

この仮定が妥当かどうか考えろ。
特殊相対論でいう”長さ”がどう定義されてるか勉強してから帰ってこい。
587585:02/03/20 22:07 ID:AUHhcXxT
>>586
漏れは物理をやっていないシロートなので
もちっとわかりやすく説明してくりゃれ。
588ご冗談でしょう?名無しさん:02/03/20 22:31 ID:???
まず「いつ加速を始めたか」という情報自体、
実際には見る系によって異なってくる。
船尾と船首では10光年も離れているのだから、
もし船尾から何らかの物質を放出して加速するタイプの
宇宙船ならば船首が加速を始めるのは最低でも10年後のことになる。


ある部分で加速を始めたらなんとなく全体が同時に
加速し始めると思ってはいけない。物体は必ず変形する。
589Who:02/03/21 00:14 ID:IcEzN9DX
>>584
式だけだと考え方が分からないと思うので、力学の本を
読んでみてください。たぶんよく似た問題が載っている
と思います。
590A.T:02/03/21 00:25 ID:???
>>589
あ、はいそれじゃそういう問題探して見ます。
ってかほんとありがとうございました!
591かげお:02/03/21 01:25 ID:???
>>585
動いている宇宙船にとっての同時刻は、外の観測者にとっては同時刻では
ありません。(同時刻の相対性)
たとえ宇宙船の各部分に動力をつけてまったく同じ加速をしたとしても、
外の観測者から見れば、船首の加速が終わるのは船尾の加速が終わってから
10年以上も先になります。具体的には、宇宙船は船尾から船首に向けて
潰れていくように観測されるでしょう。従って、宇宙船のどの部分も光速を
超えません。
592ゆうすけ:02/03/21 01:58 ID:is8N1q9/
はじめまして。
伺いたいのですが、エネルギーの次元 とはなんでしょうか?
今、浪人しているのですが、大学入試に必要ですか?
物理はまだ、最後まで終わらせいませんが・・・
よろしくおねがいします。
一応、検索でしらべましたが、エネルギーの次元を***とし
のような使い方をしていて、エネルギーの次元という言葉の
意味にはふれていませんでした。
ヒット数も少なく・・・
本当にわかりません・・・。
場違いでしたらすみません。
593ご冗談でしょう?名無しさん:02/03/21 02:00 ID:???
ヒント:単位
594ゆうすけ:02/03/21 02:15 ID:is8N1q9/
全然わかりません・・・・ヒントをいただいても
まったくです・・・・
595ご冗談でしょう?名無しさん:02/03/21 02:17 ID:???
エネルギーの単位は?
596ゆうすけ:02/03/21 02:30 ID:is8N1q9/
Jですか?僕ばかです・・・すみません。
597ご冗談でしょう?名無しさん:02/03/21 02:34 ID:???
次元解析って高校の物理ではやらないんだっけ?
もしやるとしたら、そんなこと浪人になってから
知ろうとするのは、ちょっと、なんだかな、というか。
独学ですかな。
598ゆうすけ:02/03/21 02:45 ID:is8N1q9/
そうですか・・ありがとうございました。
599595:02/03/21 04:22 ID:???
>>598
たとえば運動エネルギーはmv^2/2ですが、この単位は何になるでしょう?

確かに「J」も一つの答えですが、もっと基本的な単位(メートルとかキログラム
とか)だけで組み立てて表現できるはずです。
ネムイのでこんだけ。
600ご冗談でしょう?名無しさん:02/03/21 04:37 ID:tJVXSUyw
ちょっとスレ見ていて気になったことがあったんだが、
粘度の単位はkg/m s で、これってPs・sともあらわせるけど、
なぜ圧力かける時間で粘度になるのでしょうか?
601a-b*mb:02/03/21 21:30 ID:iVZUZVYV
いま巷で話題のエシュロンのことなのですが、
新聞によると、電波情報の将来的な減少を見越して、
英語文化圏同盟は、光ファイバーの海底ケーブルから、情報を盗む算段を始めたとありました。
電線からの盗聴は理解できますが、こんな事が可能なのでしょうか。
602Who:02/03/21 23:48 ID:M6NJrYl2
もし、初めて物理を習う人に質量とは何ですか?
と尋ねられたらどう教えますか?
603ご冗談でしょう?名無しさん:02/03/22 00:07 ID:???
>>600
それは圧力とは呼ばないぞ
まあ定性的に考えると、同じ力で速く動くかゆっくり動くかって事でしょ
604ご冗談でしょう?名無しさん:02/03/22 00:10 ID:???
>>602
同じ力を与えた時の動かしずらさ
605犬(文系):02/03/22 04:30 ID:???
宇宙空間に五万トンの物体が浮いています。
コイツは静止しています。
これに一度の打撃を加えて、15m/sの速度を与えるためにはどれだけのエネルギーが
必要なのでしょうか?
必要なエネルギーをジュールとニュートンで求める式を教えてください。
606Who:02/03/22 10:17 ID:MwHa6gKc
>>604
その説明で相手が理解できるでしょうか?
実際にはなかなか相手に伝わらないと思うんですが。
初めての人に説明するって、難しいですね。
607ご冗談でしょう?名無しさん:02/03/22 12:30 ID:1G7tL/QL
>>606
初めての人というだけではなくて、相手が小学生か中学生か、
あるいは大学生か、老人か大人か、恋人か、まったく関係ない人か
それによって説明の仕方は異なると思うので、掲示板では限界があるのでは?

ま 普通は重さから説明をはじめて、じゃ、月の上では、って持っていって質量を定義して、
それから慣性質量の話をするんじゃないかな。どのくらい突っ込んだ話をするかは、
相手によると思う。
608ご冗談でしょう?名無しさん:02/03/22 12:35 ID:1G7tL/QL
>>581>>582
可聴域を超える音には鼓膜は追従しなくなってるのでは?
それでも破れるのかな?
破れるとして可聴域の音とどのくらいの音量の比率になるのかな?

>>580
医者とか生物関係の板のほうが良くない?
609ご冗談でしょう?名無しさん:02/03/22 12:55 ID:???
>>580

鼓膜が共振しないから聞こえない → 鼓膜は動かない と思われ。
超音波と共振する部分が体にあれば、その部分がぶるぶる震えるけど、
超音波検診やってるんだから、そういう部分はないのでは?
610ご冗談でしょう?名無しさん:02/03/22 13:13 ID:???
>>608,609
鼓膜が追従しないのではなく、追従しにくいだけ。
音量を無制限に大きくすれば、必ず対象の破壊を招く。

> 医者とか生物関係の板のほうが良くない?
それは正しい意見。

あと、超音波検診に使うようなMHzの音波でも、
音量次第で細胞死を招く。
611ご冗談でしょう?名無しさん:02/03/22 13:33 ID:hANGI4/n
>>601
分配器でもつけるんだろ
612ご冗談でしょう?名無しさん:02/03/22 15:21 ID:???
>580

http://www.kanazawa-med.ac.jp/~hiromu/new_page_17.htm
>体外衝撃波胆石破砕法(Extracorporeal shock wave lithotripsy: ESWL)

http://www.daido-it.ac.jp/~doboku/kuwa/suna/suna.htm
>超音波とは、音波の一種で周波数が20kHz(20000波長)以上で音として
>耳に聞こえない音波と一般的に定義されています。また、指向性が強く強度が
>大きいため、水中に超音波を照射するとキャビテーション現象と呼ばれる独特
>の現象が生じます。

http://www.cho-onpa.co.jp/jprdct_top.htm
>超音波洗浄装置

http://www.telshop.co.jp/sale/onpa.html
>超音波歯ブラシ
613ご冗談でしょう@名無しさん:02/03/22 15:42 ID:???
>>580

超音波発生機器の安全基準を調べるといいのでは?
人体に傷害が及ぶ可能性があれば、規制や届出・型式認定など
がなされているはずだよ。
614ご冗談でしょう?エッチマンさん:02/03/22 19:37 ID:nMgyoBVi
ソモサン!
例えばここに、小型加速器があるとする。その中では粒子が超高速
で運動している。加速器の質量を0とすると、地上でこの加速器の重
さを測ると、粒子の静止質量に等しくなる。
 いま、地上の重力加速度と等しい加速度系に、この小型加速器を
持ってきて、重さを量った場合、等価原理により、地上と同様の観測
結果が出るのだろうか?慣性質量が高速粒子により莫大な物になって
いる場合、本当に一致するんだろうか?

物理に卓越された皆さんのセッパを、お待ちいたす。
615ご冗談でしょう?名無しさん:02/03/22 19:40 ID:???
>例えばここに、小型加速器があるとする。その中では粒子が超高速
>で運動している。加速器の質量を0とすると、地上でこの加速器の重
>さを測ると、粒子の静止質量に等しくなる。

なんでやねん!!
616ご冗談でしょう?名無しさん:02/03/22 21:59 ID:quLkd0vI
今、金スマで陰陽師やっているんですけど、これ見ていてどう思いますか。
私、ちょっと耐えきれないんですけど・・・
617  :02/03/22 22:10 ID:MS/MTTAY
>>616
折れも見てるよ。
こういうの見てると、自分の子供にはちゃんとした教育をさせねばと思うね。
こんなの自分の親が真剣になって見てると思うだけで
自分の人生終わった気がするねよ。
618ご冗談でしょう?名無しさん:02/03/22 23:09 ID:???
ブーメランはどうして還って来るのでしょうか?
息子にうまく(ぜんぜん)説明出来ないもので。
619ご冗談でしょう?名無しさん:02/03/22 23:33 ID:???
>>618
帰巣本能
620ご冗談でしょう?名無しさん:02/03/23 02:44 ID:???
621ご冗談でしょう?名無しさん:02/03/23 03:02 ID:ntzDclqL
剛体の角運動量方程式から物体固定座標系のオイラーの運動方程式↓というものが導かれますが
d(ωx)/dt = Tx -( Iy - Iz ) * ωy * ωz / Ix
d(ωy)/dt = Ty -( Iz - Ix ) * ωz * ωx / Iy
d(ωz)/dt = Tz -( Ix - Iy ) * ωx * ωy / Iz
※ω:角速度,I:主慣性モーメント,T:外力

外力がゼロでも角加速度がゼロではないですよね
角速度ベクトルがもし見えたら,角速度ベクトルは時間と共に回転しているのでしょうか?

で,相対角運動量 L は
L = ωI
Iは時間に関係無く一定だから,角運動量が保存されない???

回転エネルギ K は
K = ( Ix * ωx^2 + Iy * ωy^2 + Iz * ωz^2 ) / 2

これもまずい・・・
おしえてまんこ
622ご冗談でしょう?名無しさん:02/03/23 08:12 ID:99hYiPXK
院試って普通は何校受けるもん?3くらい?
623ご冗談でしょう?名無しさん:02/03/23 08:23 ID:???
行きたくないとこ受けてもしょーがねーべ?
624622:02/03/23 08:55 ID:99hYiPXK
>>623
複数校の研究室でやりたい分野の研究ができるときは複数校うけるでしょ
でも数の目安ってあるじゃん
625ご冗談でしょう?名無しさん:02/03/23 08:56 ID:99hYiPXK
>>623
Youは難航受けた?
626ご冗談でしょう?名無しさん:02/03/23 09:21 ID:???
まだ受けてないけど・・・ tかk大いっぽんでいこうかと。
目下比較検討中。

行きたいとこは全部受けりゃいいと思うんだけど。
受かった諸先輩方の意見はどーなんだろ。
627ご冗談でしょう?名無しさん:02/03/23 12:45 ID:???
D-T反応におけるローソンの点火条件について教えて下さい
特に温度、圧力、時間の関係式がありましたらよろしく
628ご冗談でしょう?エッチマンさん:02/03/23 14:04 ID:ThKYQDpU
はあ・・・仕方ない、新しいスレ立てよう。
629ご冗談でしょう?名無しさん:02/03/23 20:10 ID:PHEv6NjW
>>622
今どこら辺の大学でどこら辺の事をやっていて
将来的にどこら辺の事をやりたいのか書け

t逝ってる奴がt院に逝くんだったら一つでいいだろ

あとなー、将来研究で食っていくつもりがないんだったら
院なんか逝くなよ。
それか、趣味と割り切れよ。
630ご冗談でしょう?エッチマンさん:02/03/23 21:16 ID:rlis22cj
物理だったらK大だね!それはそうと、「614」の質問には
ホントに答えてくれないの?
631ご冗談でしょう?名無しさん:02/03/23 21:29 ID:???
>>630
ていうか、意味わかんねーもん
632ご冗談でしょう?名無しさん:02/03/23 21:56 ID:roFe6Qvj
NHKで、地球は大きな磁石で、北極はN極、南極はS極って
イラストが出たのですが、
北極はS極
南極はN極
と主張するデンパが出現していました。

http://live.2ch.net/test/read.cgi/weekly/1016853781/640-1001

なんとか説明してやってください。。。
633ご冗談でしょう?名無しさん:02/03/23 22:06 ID:???
ハァ?
しねよ
634ご冗談でしょう?名無しさん:02/03/23 22:14 ID:???
>>632
NHKに抗議しる!!!
635632:02/03/23 22:39 ID:roFe6Qvj
NHKに電話で質問したら、

北極はN極
南極はS極
方位磁石の赤はS極、白はN極

だそうです。。。

納得!

みなさま、ご迷惑をおかけしました。。。
636ご冗談でしょう?名無しさん:02/03/23 23:39 ID:???
 ネタですか?
 北を指すからN極と名づけたんでしょうが・・・。
637ご冗談でしょう?名無しさん:02/03/24 00:28 ID:???
>>632
つーかそのスレの640がちゃんと説明してるサイトにリンクはってるだろ
ttp://qa.jst.go.jp/qadata7/JST-Q1444.htm
638ご冗談でしょう?名無しさん:02/03/24 00:41 ID:3j2AUlQK
物理学で対称性ってどれくらい重要なの?
639名無し ◆TLe2H2No :02/03/24 02:44 ID:uesdx5Ir
ポテンシャルについて質問です。
ある質点が原点からの距離に比例する力(比例定数をkとする)を受け、
その方向が原点を向くような運動を考えるとき、力Fの成分は
 Fx=-kx , Fy=-ky , Fz=-kz
と書けるとありました。

一方ポテンシャルが
 U=C/√(x^2+y^2+z^2) (Cは比例定数)
となる運動を考えたとき,r=√(x^2+y^2+z^2)とおけば
 Fx=-C/r^2*x/r , Fy=-C/r^2*y/r , Fz=-C/r^2*z/r
となるので,この場合質点は原点からの距離の二乗に逆比例し,原点方向に
向かう力を受けるとありました。ここで質問ですが,この場合は成分表示したとき
にx/√(x^2+y^2+z^2) のように書き表すのに、なぜ最初の例のときは
ただ単にxと書き表すのでしょうか?最初の例のように書けば最後の例は
 Fx=-C/r^2*x
のようには書けないのでしょうか?
640ご冗談でしょう?名無しさん:02/03/24 06:17 ID:???
>>639
今から寝る所なので激しく勘違いなことを言うかもしれないが
つまりUは球対称ポテンシャルということが言いたいわけでしてそれなら
xとかyとかはどうでも良くて原点からの距離rが重要なわけです
あとUを偏微分するとFってのは良いですか。

では寝ます。お休み。うわーもう空が明るいよ。
641ご冗談でしょう?名無しさん:02/03/24 09:08 ID:???
>>639
 結局同じ値なら別にどんな書き方しようがかまないんじゃない?
ただ最後の式は
  Fx=-C/r^2*x (×) >> Fx=-C/r^3*x (○)
だよね?
 後はまぁ強いて言えば式の見易さを気にするくらいか。
>>640はこのことも言ってるんだと思うけど。
642Who:02/03/24 10:05 ID:AzDYP0l1
>>639
Fx=-k*x , Fy=-k*y , Fz=-k*z の場合、Fの2乗を計算すると
F^2=k^2(x^2+y^2+z^2) となり、r=√(x^2+y^2+z^2)とおけば、Fは
F=-k*rとなって距離 r に比例したしきにちゃんとなっている。
だからFx=-k*x , Fy=-k*y , Fz=-k*zと書くのは正しい。

次にFx=-C/r^2*x/r , Fy=-C/r^2*y/r , Fz=-C/r^2*z/r の場合も
同じように考えてみると
F^2=C^2*(x^2+y^2+z^2)/r^6 = C^2/r^4
だから F=ーC/r^2となり rの2乗に逆比例した形になっている。
だからrの2乗に逆比例した力を
Fx=-C/r^2*x/r , Fy=-C/r^2*y/r , Fz=-C/r^2*z/r
に書くのは正しい。

じゃあもし639の言う形で力を表せばどうなるか?
Fx=-C/r^2*x のように書くとFの2乗はF^2=C^2/r^2となって
F=-C/r となってrの2乗に反比例しなくなる。
じゃあ何がおかしいかなんだけど、Fx=-C/r^2*xとかくなら
このときのxは単に座標のxではなくて力のx方向の「成分」の
意味合いを持たさなければならない。
だからx方法の成分ということで639のxを x=> x/r と書く。
643ご冗談でしょう?名無しさん:02/03/24 15:16 ID:5oPdkrpG
重いものと軽いものはやっぱ一緒に落ちるものなんでしょうか?
学校ではそう習いました。
でも、重いものはやっぱり早く落ちる気がします。
重いものは早く落ちますよね?
644犬(文系):02/03/24 17:03 ID:???
>>643
空気抵抗のせいでそう思えるんですよ。
一度真空中で試して御覧なさい。同じ速度で落下しますから。

ところで、だれか>>605の質問には答えてくれぬのでしょうか
…ヒントでいいんでどなたか是非。
645ご冗談でしょう?名無しさん:02/03/24 17:24 ID:UrSHOyFk
10kgの石を2個落とす。
10kgの石を糸でつないで同時に落とす。
10kgの石を棒でつないで同時に落とす。
10kgの石を短い棒でつないで同時に落とす。
10kgの石を接着して落とす。

646ご冗談でしょう?名無しさん:02/03/24 17:34 ID:???
>>644
打撃では普通熱や火花が出るから、正確には出せないと思う。
はなはだ非現実的ですが、打撃のエネルギーが全て運動エネルギーになる
と考えると? E=(mv^2)/2 だよね。
647ご冗談でしょう?名無しさん:02/03/24 17:37 ID:UrSHOyFk
そーなん?静止してる時よりちと重くなるから。。。とか言われるんかと思った。
648643です。:02/03/24 18:04 ID:5oPdkrpG
レスありがとうございます。

10kgの鉄の玉が10mの高さから落ちるとします。
それを、お月さんと地球でやると地球でやったほうが
早く落ちますよね?

極端なたとえになりますが、
地球に太陽が落ちるのと、地球にお月さんが落ちるのも同じになるんでしょうか?
649ご冗談でしょう?名無しさん:02/03/24 18:17 ID:UrSHOyFk
どやろかなあ。
太陽が地球に落ちるには、少なくとも太陽の半径以上の高さからじゃないと
落とせないからなあ。
そんなに高いところだと地球の重力はかなり弱くなりそうだしなあ。
10mの高さなら、落下中に受ける重力はほとんど同じとして考えて良いだろうけど。
650:02/03/24 18:34 ID:???
興味本意でお聞きします。現在古典電磁気学を学んでいるのですが、
電荷系のポテンシャルエネルギーの計算を一つの電荷に適用すると、
和が発散してしまいます。
本によると(バークレー)、古典電磁気学の範囲を超える内容とのこと。
一体、どうやって現在は点電荷を存在させるのに必要なエネルギーを
求めているのでしょうか?
651ご冗談でしょう?名無しさん:02/03/24 19:43 ID:???
>>648
落下速度が同じ、というときの速度とは
2物体の共通重心に対する速度のこと。
物体表面同士の相対速度ではない。
652643です。:02/03/24 20:09 ID:5oPdkrpG
ふむふむ。
では、太陽とお月さんがそのままの質量で、野球ボールの
大きさになったとします。
その場合、太陽とお月さんを10mの高さから
落としたらそれも同じ時間で地面にたどり着く?
653ご冗談でしょう?名無しさん:02/03/24 20:18 ID:???
651から自分で考えてみなせい
654643です。:02/03/24 20:26 ID:5oPdkrpG
重いものだと重心が地球の中心から落とそうとするもの
にずれるから落ちる時間も早くなる?
地球の地面を基準に考えてしまうとやっぱり重いものが
先に落ちるってことかな・・。
655ご冗談でしょう?名無しさん:02/03/24 20:40 ID:WfWogTaE
ラグランジュの未定定数決定法について
かかれているページ教えて
656ご冗談でしょう?名無しさん:02/03/25 01:03 ID:uj1gFl7l
超紐の概要教えて。
俺の頭では、かいつまんで理解する事も不可能だった(藁)
ま、物理屋さんじゃないから、知らなくても良いけど興味本位で聞いてみた。
657な ◆P/Pe9sxI :02/03/25 03:36 ID:5RIzbKvt
>>654
厳密に言えば、同じ高さから自由落下させる時には、
質量の大きい物体の方が小さい物体よりも早く落ちます。
万有引力が2物の質量の積に比例するからです。

100gの物体と1000gの物体とで実験して、
差を測定できる技術は現在あるのでしょうか。
658 :02/03/25 03:59 ID:???
ままままじですか?
659ご冗談でしょう?名無しさん:02/03/25 04:28 ID:Ezlz1E+T
F=GMm/r^2
ma=F=GMm/r^2
a=GM/r^2
加速度に己の質量は関係ないぴょ
660な ◆P/Pe9sxI :02/03/25 05:22 ID:5RIzbKvt
間違えた。てへ。
何だ、単なる勘違いか。
等価原理を否定する大物が現れたかと期待したのに(藁
662ご冗談でしょう?名無しさん:02/03/25 13:07 ID:???
>>654
正解。次は実際に運動方程式計算してみようか。
2物体の相対速度と、重心に対する速度の両方を
調べてみよう。
663ご冗談でしょう?名無しさん:02/03/25 14:30 ID:toG1Jf9i
>>650
全くの素人なんで、答えるっつぅのもアレなんですが
最後の一文の意味が理解しにくいんで、想像で書きますと

電荷によるポテンシャルが発散するって電荷に非常に近い地域ですよね、
バークレーという本は知りませんが、
古典電磁気学では、静電ポテンシャルは点電荷の中心部で発散するが、
そのような小さなスケールではどうせ古典電磁気学は適用できないので、
発散していても気にするな。ってことじゃないんですか?
だから気にしないのが一番かと...
気になるなら量子力学を勉強すればよいんじゃないかな?

専門家の人どうですか?
664ご冗談でしょう?名無しさん:02/03/25 14:52 ID:Oog2dX97
>>663
点電荷は理想化されたひとつのモデルに過ぎないわけで、
そもそも電気量は1点に凝縮していないのでは?
量子力学では、電荷は常に仮想光子を出していると考えるので、
その近傍での積分は、もっと厳密に計算しなければいけないでしょう。
665ご冗談でしょう?名無しさん:02/03/25 15:03 ID:???
くりこみくりこみ
666132人目の素数さん:02/03/25 15:15 ID:JKpPYeu1
春休みの復習で、高校の物理(力学)と
大学の力学を並行して勉強したら
余計わかんなくなってきた・・・。
やっぱ、高校の物理は勉強する価値無し?
667ご冗談でしょう?名無しさん:02/03/25 15:28 ID:???
大学に入ったらやる価値なし。
668132人目の素数さん:02/03/25 15:57 ID:BlGP2qYH
でもさ、高校物理の問題、久しぶりに見たら解けないんだよ。
微積を使わないから解かりにくくてさ。
669ご冗談でしょう?名無しさん:02/03/25 22:12 ID:DIcf/0yA
この板のバナー何種類あるか知りたい・・・
やっぱり、ファインマンのもあるよね?
670ご冗談でしょう?名無しさん:02/03/25 22:29 ID:???
>>669
物理板でもロゴをつくってみませんか? http://cheese.2ch.net/test/read.cgi/sci/992696475/l50
671な ◆P/Pe9sxI :02/03/26 01:47 ID:lIf0oNq1
>>659
「てへ」のすぐ後で寝て、昼過ぎに起きて、床屋行って、
鏡の中の自分を見つめながらふと疑問が浮かびました。なんで刈り上げに失敗してんだよ、って。

で、地球の加速度AはGm/r^2ですよね。質量が大きい物ほど地球に大きな加速度を
与えませんか? そうなると、早く落ちませんか?
672ご冗談でしょう?名無しさん:02/03/26 02:09 ID:???
>>671
要するに、重い物体のほうが地球を強く引っ張るから、「地球と物体の相対速度」
の意味では重い物体のほうが (ごくわずか) 速く落ちる、ってことでしょ。
重さが羽毛と月くらい違えば無視できない違いが出るだろうね。
7〜8年くらい前に Nifty の FSCI あたりで論争になってた、古いネタだ。
673651,662:02/03/26 09:55 ID:???
>>671
ごたごた御託を並べる前に実際に速度計算しろ。
他人が書いてること少しは読め。
674i:02/03/26 12:46 ID:DuWJn7pS
<厳密> に計算するなら、物体の運動と同時に
地球に働く力から地球の運動も議論しておこうね。
まずは適切な慣性座標系で考えなくちゃ!
結果出たら教えてよ。
675ハートのクイーン:02/03/26 15:34 ID:IZ91H5Lp
なんだかスレ違いな感じがしますが。。

今アルフケンの5th Editionを読んでいますが、ここの5ページ目

(2)Vector A may be defined over some extended region, that is, A and
its components may be functions of position, for example, Ax=Ax(x,y,z),
and so on. Examples of this sort include the velocity of a fluid varying
from point to point over a given volume and electric and magnetic fields.

の部分を訳してみたら

(2)ベクトルAが範囲を拡張されて定義される、つまり、ベクトルAとその成分が
位置の関数、例えばAx=Ax(x,y,z)とその軸上(?)となる。
この種の例は点から与えられた体積や電場、磁場を越えた点へ変化する流体の速度を含む。

と、いう頭の弱そうな訳になってしまい意味が分からなくなってしまいました。
ここの部分の意味を教えていただけないでしょうか。
676ご冗談でしょう?名無しさん:02/03/26 15:49 ID:???
... and so on も知らないやつが英語で専門書を読むなよ
677ご冗談でしょう?名無しさん:02/03/26 16:16 ID:???
ほんとに、数学なんかやる前に英語勉強しろ。
ようするに、
世の中には(位置に依存しない定ベクトルだけでなく)位置に依存するベクトル場なるものがある。
それは各点ごとに異なった成分をもつベクトルが張り付いてる。
例えば、電場や磁場とか速度場。
678ハートのクイーン:02/03/26 18:05 ID:TNbZVwon
>676
and so on の意味は「...など」でした。
辞書見たら載ってました。(笑
>677
ご説明の方有難う御座いました。

私は英語が人並みはずれてできないので、
この本では英語の勉強も併せてやろうと欲張ったのですが
無理がありすぎたみたです。
英語を一から勉強しなおしますです。
679ご冗談でしょう?名無しさん:02/03/26 18:10 ID:???
>>678
とりあえず高校英語ぐらいまではできないとね…
ゆっくりでも着実に力をつけたほうが良いですよ。
680ご冗談でしょう?名無しさん:02/03/26 19:10 ID:???
わからない部分が、内容がわからないのか、英語自体がわかってないのか
判別できないでしょ。とっても非効率だよ
681ご冗談でしょう?名無しさん:02/03/26 20:31 ID:zyWXg6d8
今度の看板はファラデーか…
あれを面積分すると、磁束の時間変化が
起電力の原因になるっていう受験のときからの同じみの式に
なるんだよね。ストークスの定理を使うのだが…

さって、次はなんだろう。ディラック方程式か、うーん
なんだろうな?
682ご冗談でしょう?名無しさん:02/03/26 21:11 ID:???
Diracはこの前見た気がするが。
683ご冗談でしょう?名無しさん:02/03/26 21:16 ID:???
>>682
俺もこの前、Dirac見た。吉野家で並食ってたよ。
以外と日本食が好きらしい。
684ご冗談でしょう?名無しさん:02/03/26 22:04 ID:c2Q5KRsE
Diracの膿
685ご冗談でしょう?名無しさん:02/03/26 22:16 ID:iVqgrBvY
>>683

ディラックを見ましたか…?あうそう…
ディラックってけっこう庶民的なんだね。
今度見かけたら、あっちでは何して生活しているのか
聞いといて下さいね。
686ご冗談でしょう?名無しさん:02/03/26 23:18 ID:???
Diracは最近便秘気味らしいよ
687ご冗談でしょう?名無しさん:02/03/26 23:59 ID:FQKykFY7
>>671
あのねー
重い物質は確かに強く地球に引かれるんだけどー
でも、重いから加速しにくいんだよねー

だからー、結局重くても軽くても差し引き同じなんだよねー

等価原理って言うんだけどー、聞いた事あるかなー?
688ご冗談でしょう?名無しさん:02/03/27 00:28 ID:xMNuAxRO
真空中って温度はあるんですか?
689672:02/03/27 02:00 ID:???
>>687
あのね〜
宇宙に地球と落下物だけがあるとしたとき、重心系で考えると、

物体の運動
 mx'' = - mMG/r^2

地球の運動
 MX'' = mMG/r^2
 (m, M : 落下物と地球の質量、x, X : 落下物と地球の位置、r : 落下物と地球の距離)

だよね。

だから、落下物の加速度 x'' は m によらないけど、地球の加速度 X'' は m に
依存するよね。

だから、「落下速度」を「地球と落下物が近づく速度」と定義するなら (そんなに変な
定義じゃないでしょ)、落下速度は m が大きいほうが速いよね。

さらに、地球と落下物がより速く近づくってことは、より早く近くなるってことで、重力
は逆自乗則にしたがって強くなるから、落下物自体の速度も速くなるよね。

…ってことを 671 はず〜っと言ってるの。くだらない、枝葉末節な話だけどね。

これくらいのことは読み取れるようになってからレスつけようね〜
馬鹿丸出しだよ〜
690ご冗談でしょう?名無しさん:02/03/27 02:27 ID:qtavTgjt
どなたかの3次元箱型ポテンシャルの固有状態,波動関数を求める問題
について教えてください.サイエンス社の「演習 量子力学」持っている方は,
P46,2章例題12です.
初めの方に「E<0では束縛状態が,E>0では非束縛状態が存在する」
ってありますよね?まだシュレディンガー方程式解いてもいないのに,
どうしてそんなことがわかるのでしょうか?
ポテンシャルを見ればわかるのですか?また,E<0ってどういうこと?
エネルギーが負?よくわかりません.
どなたか教えてください!馬鹿ですみません.
691ご冗談でしょう?名無しさん:02/03/27 04:14 ID:Cz2txAoY
エネルギーの最低値は任意だからねー…
1年のとき、惑星の古典力学やらなかった?
その時E<0とかあったと思うけど…
そんなことくらい一度勉強したら覚えとけよ、まーいいけど
692ご冗談でしょう?名無しさん:02/03/27 04:21 ID:UcRct/tt
↑の続き

そういえば書いてて思い出したけど、力学の演習問題で
ロケットの惑星からの脱出速度を計算する演習問題があったなー
E≧0、E<0 が判定の基準になるんだったような…つまり
それが束縛状態に対応するのではないかいな。

693ご冗談でしょう?名無しさん:02/03/27 05:30 ID:8hoi13L0
>>682

あんたの述べてる式はマクスウェル方程式の
rotE = - ∂B/∂t
でしょ。看板はこれをすでに面積分しちゃってるよ
694ハートのクイーン:02/03/27 09:50 ID:???
>679
高校の時の参考書見つけたので、
それをやろうかと思うです。

>680
>675はちょうど、その状態になっていたですね。
仰るとおり非効率でした。。

レス下さった方々、有難う御座いました。
695ご冗談でしょう?名無しさん:02/03/27 17:19 ID:???
今年京理に入るんすけど、今、何をしておくべきですか?
696ご冗談でしょう?名無しさん:02/03/27 17:33 ID:c9ftgjht
直角に曲がった管の中に水を流すと、
どのような力が水から管にかかるんですか?
697ご冗談でしょう?名無しさん:02/03/27 17:41 ID:yGqNErPO
>>696 水が管を押す力がかかります
698ご冗談でしょう?名無しさん:02/03/27 17:41 ID:KVFySEcE
==2==C==H======================================================

         2ちゃんねるのお勧めな話題と
     ネットでの面白い出来事を配送したいと思ってます。。。
===============================読者数:102115人 発行日:2002/03/12
どもども、人格障害者のひろゆきですー。
今日は重大なお知らせがありますですー。
これまで2chでは、企業に対する真偽の曖昧な書き込みは削除しませんでしたが、これからは削除依頼が出たら一旦全て削除して、その真偽をおいらが責任を持って調査し、真実であると確信した時点で改めて復帰させ公開することにしましたですー。。。
これまでは企業に対して、書き込みの内容が嘘であることを「証明してみろ」という傲慢な態度を示してきましたが、これからはおいらが書き込みの内容が真実であることを証明してから大衆の目に触れさせることにしますですー、、、
いやぁ、よく考えると始めからこうするのが当然だったのかも知れませんねー。。。

ちなみに最近はDHCの裁判の模様をお伝えしてますが、それも話半分で聞いていてくださいー、、、
おいらは相手側の主張の極一部を抜き取って、自分の都合の良いように歪曲して報告してますが、その前後の話も含めれば本当は全く違う状況なんですよー、、、えぇえぇ、、、
おいらってば本当に卑劣な人間なんですよー。。。
向こうの弁護士さんも相当に怒っていると思いますですー。。。

ついでに言っておきますが、DHCの件もこれまでと同様に旗色が悪くなったら報告するのを止めますので予めご了承くださいですー、、、
本当は結果まで全て真実を報告して、もし削除するよう決定がくだれば、相手側の企業に対する謝罪のひとつでも掲載するべきなんですよねー。。。
おいらってば人格障害者なもんで許して欲しいですー。。。

んじゃ!
699ご冗談でしょう?名無しさん:02/03/27 17:48 ID:c9ftgjht
>>697
やっぱりそうなりますよね。
ありがとうございます
700ご冗談でしょう?名無しさん :02/03/27 18:44 ID:vqaxY5Et
>>695
わからないことがあれば人に聞く前に、自分で調べる
習慣をつけること。
701ご冗談でしょう?名無しさん:02/03/27 21:08 ID:H8vYYRrq
>>695
今何をすべきかって?
それだったら車の免許取っとくことを薦めるよ。まじで。
702ご冗談でしょう?名無しさん:02/03/27 23:06 ID:8t1YH6Z9
あと、適当に物理数学の公式を丸暗記しとくといいよ
微分演算子を使った微分方程式の公式とかテーラー展開の公式
積分公式、うーん、で余力があれば留数定理とか複素数関係、線形代数
とかまーとにかくそこらへんは暗記だね

703ご冗談でしょう?名無しさん:02/03/27 23:44 ID:FSk5sxiP
エネルギー密度
てイメージしにくいけど、どうやったら理解できる?
704:02/03/27 23:46 ID:???
<664
レスありがとうございます。
「仮想光子」というのは初めて聞きました。
何だか良く分からないのですが、量子力学によって説明されそう
ということが分かりました。
どんな概念があるのかワクワクします。
705ご冗談でしょう?名無しさん:02/03/28 00:06 ID:???
>>704
まじめにやろうと思えば、場の理論を学ばなくてはなりません。
ブルーバックスあたりの啓蒙書を読めば概念はわかると思いますが。
(ていうか電荷の計算における繰り込みうんぬんも書いてるはず)
706ご冗談でしょう?名無しさん:02/03/28 00:08 ID:RvwNsco4
>>703
例えば、調和振動子1個のエネルギーをEとしたとするでしょ
で、その調和振動子が10個あったとするでしょ
したらその系の全エネルギーΣE=10Eとなるでしょ

したら、今度は逆にその系を構成する調和振動子1個あたりの
エネルギーは?って聞かれたらE(=ΣE/10)ってなるじゃん

でね、まー早い話、その1個あたりのエネルギーていうのが
エネルギー密度っていうんだよね。だから単位は一般的には
[J/M^3]となるんだよ。

まー場の理論なんだけどね。だからその1個あたりのエネルギー
に伴って、ラグラジアン密度とかハミルトニアン密度とかも
あるんだよ。

ついでにいうと、相互作用のラグラジアンとか場のラグラジアン
とか粒子のラグラジアンとかもあるんだけどね

わかったかな?
707:02/03/28 00:15 ID:???
<705
ありがとうございます。
四月から二年生になります。
まずは古典電磁気学の基礎を固めたいと思います
708ご冗談でしょう?名無しさん:02/03/28 01:31 ID:7zaRwH4/
ラグランジュの未定定数法ってなに?
709ご冗談でしょう?名無しさん:02/03/28 02:25 ID:UiZvVCaJ
>>708
演習問題2,3問やればわかるよ
まー、途中計算で行列の対角化の方法とか知ってるといいけど
710ご冗談でしょう?名無しさん:02/03/28 04:34 ID:/ae+gON/
回したゆで卵なぜ立ち上がる?世紀の難問を証明
http://headlines.yahoo.co.jp/hl?a=20020328-00000502-yom-soci

解説と歴史、御願いします。
711ご冗談でしょう?名無しさん:02/03/28 05:31 ID:???
世紀の難問?バカじゃないの。
712ご冗談でしょう?名無しさん:02/03/28 12:42 ID:???
バカにはその難しさすら理解できない
713ご冗談でしょう?名無しさん:02/03/28 14:41 ID:z2fEEr8U
質問スレ 下げてどうするの? 10位以内をキープさせようよ
714ご冗談でしょう?名無しさん:02/03/28 15:59 ID:???
ラグランジュポイントの中には中心から離れるほど遠ざかる力
が働く不安定なポイントもあります
あれって斥力って言っていいんでしょうか?
それとも反重力ポイント?
715ご冗談でしょう?名無しさん:02/03/28 23:17 ID:???
>>714
ただの遠心力を斥力や反重力と呼びたいならどうぞ
たぶんアフォ呼ばわりされるだけだけど
716ご冗談でしょう?名無しさん:02/03/28 23:18 ID:qRSbBsOv
ラグランジュポイントなんですか?
717ご冗談でしょう?名無しさん:02/03/28 23:20 ID:qRSbBsOv
時間の逆行する空間がある、
ときいたのですがほんとにあるんですか
718ご冗談でしょう?名無しさん:02/03/28 23:22 ID:???
719ご冗談でしょう?名無しさん:02/03/28 23:25 ID:???
>>716
Googleで調べろ。
>>717
無い。
>>718
いるけど、何か?
720ご冗談でしょう?名無しさん :02/03/28 23:37 ID:Pcx9CPG4
ねえねえ
なんで電池って人差し指と親指で極同士を持っても電流流れないの?
指を電線代わりにどうして電流は流れないの?
721ご冗談でしょう?名無しさん:02/03/28 23:42 ID:OgYScYM0
ラグランジュの未定定数決定法って
たんなる局地を求めるだけなんじゃない?
722ご冗談でしょう?名無しさん:02/03/28 23:42 ID:???
>>720
微少に流れています
723ご冗談でしょう?名無しさん:02/03/28 23:42 ID:???
>>720
>なんで電池って人差し指と親指で極同士を持っても電流流れないの?
抵抗が大きいから。

>指を電線代わりにどうして電流は流れないの?
流れるだろ。電線触って感電死ってニュース見たこと無いか?
724ご冗談でしょう?名無しさん:02/03/28 23:43 ID:KK+jctqW
はー、みんななめてんの?あんまし下らない質問すんなよ

もし、その指に電流が流れたら、空気中に置いてても
それだけで放電するようになるかもしれん

そこらへんのところは比抵抗率ρ ( = 比導電率σ^−1 )
と抵抗の長さd、面積S で決まります。
 i(電流密度) = σ(比導電率)× E(電場)
ですね。(なんか自分の知識の確認のためのカキコ)
725ご冗談でしょう?名無しさん:02/03/28 23:54 ID:???
>>718
東大物理だけとはレベルが高すぎ。
726720:02/03/29 00:09 ID:WUw62gdg
722>>
あ、やっぱ流れてんだ
単に、神経が反応するほど流れてないって事なんだ
じゃあさ、電池を直列にいっぱいつなげれば両手で持てば流れるの?

723>>
じゃあさ、じゃあさ
電池に豆電球を電線でつなげて
その電線を切って、その端々を両手で持ったら体に電流流れるよね
その時、体の抵抗がすっごい大きいから、電圧も上がると思うんだけど
そうなると、電池はどうなるの?煙ふくの?
それと、町中の電線を切らずにそのまま両手で持ってぶら下がっても
体に電流流れないじゃんか。で、体が地面につけばばりばり流れると思うんだけど
地面ギリギリで浮いててもやっぱ空中放電したりして体に電流ながれんの?
あと、両手で電線持てば、その両手の幅の分の距離ができるよね
その両手の幅の距離分電位差が生じると思うんだけど
その電位差では、体の抵抗が大きすぎるから電流は流れないの?
727ご冗談でしょう?名無しさん:02/03/29 00:15 ID:???
>その時、体の抵抗がすっごい大きいから、電圧も上がると思うんだけど
はぁ?
>地面ギリギリで浮いててもやっぱ空中放電したりして体に電流ながれんの?
ぜひ、おまえ自身で実験しろ。
728ご冗談でしょう?名無しさん :02/03/29 00:37 ID:WUw62gdg
>>727
わからないなら、無理して答えなくて良いです
729ご冗談でしょう?名無しさん:02/03/29 00:52 ID:2gkIVMbV
> じゃあさ、電池を直列にいっぱいつなげれば両手で持てば流れるの?
流れるでしょう。

> その時、体の抵抗がすっごい大きいから、電圧も上がると思うんだけど
電池は定電圧源なので大きな抵抗が入ると電流が減少するだけです。

> 地面ギリギリで浮いててもやっぱ空中放電したりして体に電流ながれんの?
流れるでしょう。

> その両手の幅の距離分電位差が生じると思うんだけど
交流伝送なのでヒトの手の幅程度ではほとんど電位差は生じません。

> その電位差では、体の抵抗が大きすぎるから電流は流れないの?
流れません。
730完全文系人間:02/03/29 00:58 ID:uKjpOxJL
すみません、高校で物理選択してなかったのですが、どなたか
教えてください。

今度、家を建て直すことになりました。コジェネで湯を作り、
床下に密閉水槽(ステンレス製)を入れて暖房しようかな、と
考えています。本屋で公式を立ち読みして試算してみました。
この机上計算はどこが間違っているのでしょうか?↓

熱損失をとりあえず無視し、完全に密閉された空間が240m3。
ここに容量200lのお湯を入れる水槽をおくとします。

70℃のお湯が20℃まで冷えるとき、この空間に放出する熱
の総量は、

 Q=mct=200000g×4.2(比熱)×(70−20)
  =42,000,000J

空気の比熱を0.24と仮定すると、温度上昇は、

  K=42,000,000÷0.24÷240000÷1.5(比重)
   =486℃

たったこれだけのお湯で!

なあほな! わかりません! どなたかHELP!!!

お答えいただく方には先回りして、「ありがとう!」
731ご冗談でしょう?名無しさん:02/03/29 00:58 ID:WUw62gdg
>>729
ありがと
732ご冗談でしょう?名無しさん:02/03/29 01:19 ID:Yf/RNmaJ
厨房レベルの質問でごめんなさい。
月は自転しているから、
いつも同じ面が見えると
いうことが理解できません。
自転してたら違う面も見えるような気もするんだけど。

それと、惑星はなんで円盤状に並んでいるのですか?
そういえば銀河も円盤だ。
それと公転の方向はみな同じなんですか?
その方向性はどうやってきまるのですか?

本と厨房でごめんなさい。
733ご冗談でしょう?名無しさん:02/03/29 01:51 ID:???
>>730
>熱損失をとりあえず無視し、
これが死ぬほど非現実的です
734ご冗談でしょう?名無しさん:02/03/29 01:58 ID:???
>>730
水温と気温が等しくなったところで熱の流れは止まる。

735ご冗談でしょう?名無しさん :02/03/29 03:15 ID:WUw62gdg
>>732
簡単に言えば月の自転周期と公転周期が同比率のため
例えば、月が自転しないとすると、地球が90度回れば、月も90度地球の周りを動く
しかし、自転しないならば、月の見える部分も90度分増える。
その見える部分が見えない理由が自転するためである。
そして、自転周期と公転周期が同じなので、月が地球の周りを90度回ったとき
月自身も90度回る。
よって見えない。

>それと、惑星はなんで円盤状に並んでいるのですか?
まず、太陽があるとする。
そこに、太陽の引力に引き寄せられて動いていた水星が太陽の周りを回りだしたとする。
そして、金星が飛んできたとする。
もし、太陽と水星の回る方向と垂直に飛んできたなら、太陽と水星の引力が、それぞれからかかる。
しかし、太陽と水星の回る方向と水平に飛んできたなら、太陽と水星の引力が一直線でむすばれて
金星にかかる引力が太陽と水星の引力を足した力が金星にかかり、こちらの方が垂直方向に進入した
金星よりも大きな力が加わる。
結果として、水平方向に入ってくる星の方が軌道に乗りやすい為円盤状になる。
736ご冗談でしょう?名無しさん :02/03/29 03:21 ID:WUw62gdg
>>732
>それと公転の方向はみな同じなんですか?

水金地火木土天冥海の海王星と冥王星の順番がたまに変わるが
それは、公転方向が水平でないためであり
公転の方向は同じではない。
737ご冗談でしょう?名無しさん:02/03/29 05:54 ID:WaNpQgX7
回転しているコマの軸の上に
同じ速度で逆回転するコマをのせたらどーなりますか?
738ご冗談でしょう?名無しさん:02/03/29 06:15 ID:???
>>730
733,734両氏が回答していますが、
貴兄の家の風呂にお湯を一杯いれて風呂場を密閉して放置しておいたらどうなります?
次の日の朝には風呂場の温度は金星並になってますか。

外へ逃げる熱量は無視できません。
輻射と対流と伝導によって熱は逃げていきます。これらは、家の外壁面積、材質、外気温等々の関数です。
ついでに、家の外壁と熱源(つまり加熱された水)との間に何があるかも問題です。

がんばってください。
739ご冗談でしょう?名無しさん:02/03/29 10:00 ID:HbMncMM8
テレビで低周波による心霊現象を解明してました。
面白いので実験したいのですが
低周波ってどうすれば発生させられますか?
740ご冗談でしょう?名無しさん:02/03/29 10:11 ID:???
低周波によ”り”ならわかるが
741ご冗談でしょう?名無しさん:02/03/29 10:47 ID:tlYzzk5/
磁石でもグルグルまわしとけば?
742ご冗談でしょう?名無しさん:02/03/29 12:27 ID:???
>>735
惑星のほうの説明はなんだかなぁ。
http://cheese.2ch.net/test/read.cgi/sci/1010255685/でも読んだら>>732
743ご冗談でしょう?名無しさん:02/03/29 12:49 ID:???
>>736
順番が変わる理由は冥王星の軌道が大きく潰れた楕円になってることで、
軌道面の傾きは関係ない。完全に同一平面でも順番は変わりうるでしょ。

744732:02/03/29 13:30 ID:Yf/RNmaJ
>>735
月の説明はなんとなくわかりました。
要するに地球に対していつも同じ面を向けているには、
月も自転が必要ということなんでしょうか?

惑星の公転軌道については
よくわかりません。

ところで太陽は自転しているのですか?
745ご冗談でしょう?名無しさん:02/03/29 15:01 ID:ReSnKfSv
>>744
月がまったく自転していなかったら裏側が見えるはずです
ボールでも持って自分の周りを回してみれば一瞬で分かります

>ところで太陽は自転しているのですか?
太陽表面上の黒点が動いているので、自転していると考えられています
746親切な人:02/03/29 15:32 ID:???

ヤフーオークションで、凄い人気商品、発見!!!

「高性能ビデオスタビライザー」↓
http://user.auctions.yahoo.co.jp/jp/user/NEO_UURONNTYA

ヤフーオークション内では、現在、このオークション
の話題で、持ちきりです。
747完全文系人間:02/03/29 19:50 ID:uKjpOxJL
>>733,734,738

どうもありがとうございました! 逃げるのはわかっていたのですが、
とりあえずはどのくらいの熱をお湯が持っているのかが知りたかったの
でした。計算はあっていたみたいですね。それにしてもすごい熱量。

最近は「Q値」という宣伝文句をよく目にしますし、断熱サッシなんか
首都圏でも当たり前ですので。断熱性能がいいに越したことはないです
けど、そちらに金をかけた分、どのくらいの熱源を確保したらいいのか
難しいですね。
748チンコ:02/03/29 20:10 ID:zPctLQjc
749ご冗談でしょう?名無しさん:02/03/29 20:12 ID:zPctLQjc
>>748テロ?
750ご冗談でしょう?名無しさん:02/03/29 20:48 ID:UiStS6ek
スイング・バイってどういう理論なのですか?
サルにもわかるように説明してください。
751ご冗談でしょう?名無しさん:02/03/29 20:51 ID:???
ぶらんこ
752750:02/03/29 21:06 ID:UiStS6ek
>>751
ごめんなさい、よく・・・わからない・・・
753ご冗談でしょう?名無しさん:02/03/29 21:08 ID:GBABT373
duty cycleってなんですか?
754ご冗談でしょう?名無しさん:02/03/29 21:13 ID:???
>>750
その名の通り「振り逃げ」のことだYO
755750:02/03/29 21:42 ID:UiStS6ek
・・・検索してみます。スレ汚してすみませんでした。
756ご冗談でしょう?名無しさん:02/03/29 22:24 ID:???
汚したらきれいにしる!
757ご冗談でしょう?名無しさん :02/03/29 23:55 ID:BF8GRubk
さるに説明する気にはなれんわな
758落ちこぼれ生徒:02/03/30 02:02 ID:lbhSnILA
こんばんは。
突然で本当に恐縮なんですが、この一問だけ教えて下さい。叩かれるのは覚悟しております・・・。すいませんι

私は化学と言う言葉自体にアレルギー反応起こしてしまうくらいこれまで触れてこなかったので皆様のおしゃっている事もハッキリ言ってチンプンカンプンですι
そんな奴ですが、ちょっとでも教えて下さると光栄です。

問題なんですが、
「冥王星の外側、太陽から約60天文単位の距離に地球の3倍の質量を持つ太陽系10番目の惑星が発見されました。
 第10惑星は、地球と同じ公転面をほぼ円軌道で公転しています。
 地球の質量を5.974×10の24乗kg、公転周期を365.2422日として、この惑星の公転周期を求めなさい。」

ってやつです・・・。
切実にヨロシクお願いしますです・・・(T_T)

 

759親切な人:02/03/30 02:10 ID:???

ヤフーオークションで、凄い人気商品、発見!!!

「高性能ビデオスタビライザー」↓
http://user.auctions.yahoo.co.jp/jp/user/NEO_UURONNTYA

ヤフーオークション内では、現在、このオークション
の話題で、持ちきりです。
760ご冗談でしょう?名無しさん:02/03/30 02:22 ID:???
>>758
「切実」にしちゃぁずいぶん以前から悠長に何度も同じ問題を質問してないか?
1度ならず解説レスもあっただろーが。えーかげんにせぇ

761ご冗談でしょう?名無しさん:02/03/30 02:23 ID:f7aAleA+
762京都人間 ◆FGgWeDPU :02/03/30 02:44 ID:aDZr6n+B
ガラスなんかで、
暗いほうから明るいほうを見ると見えるのに
明るいほうから暗いほうを見ると鏡みたいになるのはなぜか
763ご冗談でしょう?名無しさん:02/03/30 02:59 ID:???
>>762
透過光と反射光の強さの違い
764ご冗談でしょう?名無しさん:02/03/30 02:59 ID:???
ルべーグ積分って、知っとくといいことありますか?
765ご冗談でしょう?名無しさん:02/03/30 03:00 ID:???
 反射光と透過光のどちらが明るいかで決まる。
766ご冗談でしょう?名無しさん:02/03/30 03:02 ID:???
>>764
知っといて悪いことはなかろう。たぶん
767ご冗談でしょう?名無しさん:02/03/30 06:16 ID:qfLMIaoM
 ルベーグ積分って早い話、リーマン積分の細長い短冊を合わせる
ことでグラフを近似して面積(測度)を求めるのに対して
測度を求めるグラフに小さな正方形を合わせていってそれで近似
して測度を求める、という考え方なんだよね。
 つまり、縦横両方から攻めるという考え方。
 そうすることで、異常なグラフの測度も求められるようになったんだよ

 例えば、長さ1の正方形内にx座標、y座標ともに有理数な点にaという
値を与えてその他の点(無理数な点)、これを面積分したらどうなるかと
いうと、以外にも面積(測度)S=0となるんだよ

 でもまー最後になったけど、物理屋にとって数理やる以外はルベーグ積分
なんか知っててもなんの役にもたちません。こんなの勉強するんだったら
もっと他のことやった方がいい。(繰り返し言うが数理をやる人以外は)
あえて役に立つときといえば、一仕事終えて、打ち上げの席の2次会で
ちょっと物理学とか数学とかの垣根を越えて理学を語ってみるときくらい
しか役にはたたない。

768ご冗談でしょう?名無しさん:02/03/30 06:26 ID:???
>>767
ワケワカ。ルベーグ積分だって短冊で考えるでしょうが。
その短冊の幅について測度の概念を導入するわけだ

769i:02/03/30 11:10 ID:xIKEDpJw
そうそう。ルベーグ積分では測度の概念が重要。
測度0の集合における関数の値は気にしない!
サポートコンパクトの連続関数を <拡張> した
L^2 空間やフーリエ変換では本質的な役割を果たすよね。
770ご冗談でしょう?名無しさん:02/03/30 14:03 ID:IigcDSPy
るベー具積分の簡単な本ってない?
771ご冗談でしょう?名無しさん:02/03/30 14:58 ID:3yRklrJ7
宇宙空間って、真空なのに何故温度があるの?
日陰はとても寒いらしいけど、空気と云う媒体が熱を放熱させる大気中
ならともかく宇宙で凍りつくと云う事は、放熱した熱量は一体何を
媒体にしているのですか?

それとも、宇宙空間に熱湯(仮に80度)を放つと、80度の固体(氷)
になってるんでしょうか?
772771:02/03/30 15:01 ID:3yRklrJ7
日向は、当然太陽光線という強烈なエネルギーが降り注ぐ訳だから、
温度があがることはあっても下がる事はないと思うのですが・・
773ご冗談でしょう?名無しさん:02/03/30 15:56 ID:???
「太陽光線という強烈なエネルギーが降り注ぐ訳」
ここんところをよーく考えてみよう
774ご冗談でしょう?名無しさん:02/03/30 18:27 ID:I9hnpQaq
>>769  767 です

ルベーグ積分を物理屋が簡単にざっと学ぶのに適している本は
 「ルベーグ積分30講」、志賀浩二、朝倉書店
がいいと思う。ぼくはこれで勉強しました。ただ手抜きして勉強
したせいか >>768 に突っ込まれてしまいましたが、とほほ…

でも、突っ込まれても別にどうでもいいや、と思えるほど物理屋
にとってルベーグ積分って無意味だよ
775771:02/03/30 21:47 ID:kXSsmSHQ
一応私はエーテル理論派ではないのですが・・・、太陽光線のエネルギーは
極論で云うと物質が飛んでくると云う風に理解しているのですが・・・

と、云う事は宇宙空間に放り出された80度の水はエーテルを媒体にして熱量
が奪われていき冷やされて個体(氷)になるということなのでしょうか?
776ご冗談でしょう?名無しさん:02/03/30 21:56 ID:mDcR1uDN
宇宙空間では気圧が0なので水は瞬時に沸騰し,凝固します。

>>775
熱量は真空中でも飛散するとすると、80度の水は熱量を飛散させ、0kに限りなく近づこうとするのでは?
周りからエネルギーを与えられなければ、周りにはエネルギーの無い真空状態なので、
高い所から落ちる水の如く、どんどん熱量は失われていくと思う。
最終的には水は土星の輪のように氷になるでしょう。
777771:02/03/30 22:01 ID:kXSsmSHQ
>>776 (・_・)......ン?

良く分からないのですが、80度の水は80度のまま、沸騰して凝固
するのですか、それとも、熱量も減って80度よりも温度が下がって
凝固しているのですか? もしも、温度が下がっているのでしたら、
真空中で熱量が飛散していく媒体は何でしょうか?
778771:02/03/30 22:02 ID:kXSsmSHQ
>>776 それと、沸騰=気化なのでは?

気化して凝固するというのは!? これは??
779ご冗談でしょう?名無しさん:02/03/30 22:40 ID:AdMF9QAx
>>774
本紹介してくれてありがとう
780ご冗談でしょう?名無しさん:02/03/30 23:15 ID:???
>>777
 主に気化熱によって温度が下がります。

 また、低温低圧では、水は液体の状態では(普通は)
存在しません。
781ご冗談でしょう?名無しさん:02/03/30 23:31 ID:rpLuSSD+
ルベーグ積分、物理数学を勉強するための数学…かな…
782ご冗談でしょう?名無しさん:02/03/30 23:48 ID:5r/Gq3ur
リーマン積分の完備化→ルベーグ積分
知らなかったとしても、実際は無意識に使っているのだ。
783ご冗談でしょう?名無しさん:02/03/30 23:50 ID:???
>>777
仕方ないなー。自分でも書いてるじゃん。
「太陽光線」がエネルギーを与えてるって。
エネルギーを与えられるものは、エネルギーを奪えるでしょ?
黒体輻射とか検索しる!
784ご冗談でしょう?名無しさん :02/03/31 00:10 ID:eFKsAbWx
>>771
熱を持つ物質は、赤外線がでます。
もちろん赤外線は、エネルギーがでます。
熱を持てば、分子が振動してるわけですが、そのうち分子同士で
振動をうち消しあうので、さめてきます。
ちなみに、水が沸騰するのは、圧の問題です
785784:02/03/31 00:14 ID:eFKsAbWx
言い方間違えた

赤外線というエネルギーが出ます。
786ご冗談でしょう?名無しさん:02/03/31 00:18 ID:JD0Objq2
>>777
このイシタコ! (ちょっと使ってみたかった。)
787771:02/03/31 00:20 ID:QUbudzKL
>>784 なるほど、赤外線か!!

で、気化した水が凝固するのは?
788初学者:02/03/31 02:11 ID:/ECCV/Xg
>>782
完備ってどういう意味だっけ?
789ご冗談でしょう?名無しさん:02/03/31 02:22 ID:rSHiqrvG
>>788
荒っぽく言えば、極限操作で閉じているってことかな。
ルベーグ積分できる函数は、リーマン積分できる関数列の
極限みたいなものだってこと。
790 :02/03/31 12:42 ID:???
質問です!
物理の問題で計算してって最後に桁をあわせるんですけど、足し算引き算と
かけ算わり算の時とじゃやり方が違うって聞いたんですけど、どうやって桁
を合わせるのか教えて下さい。ちなみにこれが分かんなくて浪人です。
791ご冗談でしょう?名無しさん:02/03/31 12:45 ID:???
 実際に誤差を足して計算してみれば分かるでしょ。
792790:02/03/31 12:51 ID:???
>>791
誤差を足すって意味がよく分からないんですが?
馬鹿ですいません
793ご冗談でしょう?名無しさん:02/03/31 12:55 ID:1ZyVbB9V
>>790
解らなくても大学受験程度では関係ないと思うが?
それよりも、計算問題で値を代入するのを最後にする
習慣を付けるべき。
この位の知識なら、大学入ってから実験の授業のときに教えてくれるよ。
794791:02/03/31 13:00 ID:???
 たとえば(3000+−10)と(250+−20)を
掛け合わせるのだったら、
(3000+10)*(250+10)
(3000+10)*(250−10)
などを計算してみろ、ってこと。
795790:02/03/31 13:01 ID:???
>>793
関係ないですか・・・・
たとえば 回答が2,40を2,4にして減点されるんですが?

796791:02/03/31 13:04 ID:???
 失礼、
(250+10)>>>(250+20)
(250−10)>>>(250−20)
の間違いです。
797ご冗談でしょう?名無しさん:02/03/31 13:10 ID:1ZyVbB9V
>>795
減点しても大きくてマイナス一点だろ。
まともな大学だったら、最後の計算が合ってることより
回答のロジックを重要視するはずだよ。
そうでない大学だったら入らない方がまし。
798ご冗談でしょう?名無しさん:02/03/31 13:33 ID:???
棒を回転させたとき軸付近の点より先っちょの点の方が
移動距離が大きいから速いと考えていいんでしょうか?
いいとしたら軸付近の点が光速だったら先っちょはどうなりますか?
799ご冗談でしょう?名無しさん:02/03/31 14:53 ID:IS2f0641
>>790
有効数字について調べるがよろし。
800ご冗談でしょう?名無しさん:02/03/31 14:54 ID:???
全ての条件が理想的だったとしても
先っちょが光速に到達する直前で回せなくなります

801ご冗談でしょう?名無しさん:02/03/31 14:58 ID:???
>>800
ちょっと不正確では無いか?
先っちょを光速に到達させようといくら加速しても、光速には到達できない、では。
802ご冗談でしょう?名無しさん:02/03/31 15:10 ID:???
株式会社ニュートンプレス発行、ニュートン別冊シリーズ
ってどんな本なのでしょうか?
803ご冗談でしょう?名無しさん:02/03/31 15:25 ID:EnF13+n4
>>798
 いくつか答えがあるね。
1)まわすのに無限大のトルクが必要になる。
2)棒が渦巻状に変形し、どの部分も光速に達しない。

 普通は2)の答えが採用されると思うけど。
804790:02/03/31 15:47 ID:???
>>797
有効数字ですね、ネットで調べてきます。
805ご冗談でしょう?名無しさん:02/03/31 16:54 ID:???
>>803
なるほど。どーもっす。
やっぱ質問としては既出だったみたいですね。
806ご冗談でしょう?名無しさん:02/03/31 20:21 ID:9CcD4SVw
あれ?
807ご冗談でしょう?名無しさん:02/03/31 22:38 ID:VtlJ1/pU
最近電子レンジにセンサーがついていて、
ぬくもったら自動的にとまるようになっているが、
この仕組みを物理化学的におしえてください。
808ご冗談でしょう?名無しさん:02/03/31 22:39 ID:???
底辺a、高さhの二等辺三角形の重心の位置はどこにあるか教えてください。
809ご冗談でしょう?名無しさん:02/03/31 23:40 ID:4GO/tErv
>>897
中のセンサーが「熱いよ」って言うと、止まる仕掛け

>>808
辺の1/2の点と頂点に線を引いて、交わるところが重心
810ご冗談でしょう?名無しさん:02/04/01 00:49 ID:???
陽子崩壊が観測されたってテレビのニュースでやってましたが
そんなに大事件なんですか?
811ご冗談でしょう?名無しさん:02/04/01 00:52 ID:???
>>810
ソースきぼんぬ

ノーベル賞だわさ
812811:02/04/01 00:53 ID:???
×ノーベル賞
○ノーベル賞級
ついでにage
813ご冗談でしょう?名無しさん:02/04/01 01:21 ID:???
>>810
そうでもないよ。
814ご冗談でしょう?名無しさん:02/04/01 01:29 ID:???
>>810
なお今日は何日?とか言ったらぶっ殺す
815810:02/04/01 02:09 ID:5YVkv/cP
>>814 お前みたいにメール欄見る事も知らない厨房に俺は殺せないよハゲ
816814:02/04/01 02:15 ID:???
>>815
ハァ?
縦読みくらい覚えとけヴォケ
817ご冗談でしょう?名無しさん:02/04/01 09:59 ID:4n5xb+08
>>810
「πに神のメッセージ」くらい凝ってほしかった。
818ご冗談でしょう?名無しさん:02/04/01 10:24 ID:???
こんな凄い、液晶表示板が開発されていたの?
http://dailynews.yahoo.co.jp/fc/science/april_fools/
819ご冗談でしょう?名無しさん:02/04/01 10:52 ID:???
↑URLにapril_foolsって書いてあるのが気になるところだけど。
820ご冗談でしょう?名無しさん:02/04/01 10:58 ID:???
>>818 のウソニュース結構おもしろい
>Maccrosoft、X8oxの模造品の見分け方を公開
とか。
821ご冗談でしょう?名無しさん:02/04/01 11:13 ID:???
「真っ黒ソフト」にちとワラタ
822新3厨房:02/04/01 12:26 ID:???
教えてくれたら幸いです。

時刻10-4秒後ってどういうことなんですか?
823ご冗談でしょう?名無しさん:02/04/01 12:32 ID:???
6秒後
824ご冗談でしょう?名無しさん:02/04/01 12:36 ID:???
10の−4乗秒後の時刻、と言う意味かな?
=0.0001秒
間違ってるかもしれん。前後の文章から判断しる。
825新3厨房:02/04/01 12:36 ID:???
>>823
本当ですか?
ただ単に10−4をやればよかったんですね。
ありがとうございました。
826824:02/04/01 12:48 ID:???
配慮が足らんかった。すまん。<4月ヴァカ
827ご冗談でしょう?名無しさん:02/04/01 12:53 ID:MXGqhgVy
>>822-826
ええっ!?
単純すぎて逆にわけわからん
6秒後を(10−4)秒後
なんて表記する機会なんて
あるか?
>824が正しいんじゃないか?
828ご冗談でしょう?名無しさん:02/04/01 18:57 ID:1wRY5HQs
特許の本見てたらNECが超伝導磁石で空間をゆがめ推進する装置とかいう奴の案を
登録していたが理論的に可能ですか?
鑑定してもらってる博士がDr.赤松なので…(w

形がUFOそっくりなんですよね 
829ご冗談でしょう?名無しさん:02/04/01 19:24 ID:BbupigJ4
屋上から同じ大きさの軟式のテニスボールと鉄球を同時に落としたところ、
思い鉄球が先に落下した。それらの落下運動の違いについて述べよ。

どうなるの? 教えて
830ご冗談でしょう?名無しさん:02/04/01 19:27 ID:???
>>829
空気抵抗の影響が出やすいか出にくいかって違いだろ。
831ご冗談でしょう?名無しさん:02/04/01 19:30 ID:???
>>830
空気抵抗は出やすいとか出にくいとかではないとおもわれ
832ご冗談でしょう?名無しさん:02/04/01 21:48 ID:???
あのなぜ正極=陰極になるのですか?これらの語源は何ですか?
わざわざ2つを区別して使う理由も教えてください。
833 :02/04/01 23:28 ID:Mw6D9YfT
テレビ番組のオカルト者はよく好んで見るのですが、色んな番組に
出て超能力やら気やらを肯定している人がいるのです。
町好雄教授です。
彼は何故科学者なのに超能力を信じてしまったのでしょう?どう
いう過程でオカルトにはまってしまったんでしょう?

http://www.satokiko.jp/mas/mas_post01209.html
(佐藤気孔センター)
834ご冗談でしょう?名無しさん:02/04/02 00:00 ID:???
>>829
テニスボールは空気抵抗で縦方向に縮むから。
835ご冗談でしょう?名無しさん:02/04/02 00:59 ID:???
>>829
空気抵抗が同じで、重力による力が質量分だけ違う。
よって、重い方が先に落ちる。
836ご冗談でしょう?名無しさん :02/04/02 01:59 ID:sEK3ZLEZ
>>829
地球上の一般的な建物の屋上から落としたなら
突風などの影響で、テニスボールが遅く落ちるかもね

837ご冗談でしょう?名無しさん:02/04/02 10:00 ID:uHQOcpId
>>832

A:電流が流れ出す極と流れ込む極
B:電位が高い極と低い極

1:正極と負極
2:陽極と陰極

A,B と 1,2 の正しい組み合わせを線で結べ
838ご冗談でしょう?名無しさん:02/04/02 14:41 ID:1U8Vu2CZ
物理に関しては全く素人の看板屋です。
塩ビのラミネートを施したインクジェットシートを貼ったアクリル板に
透明なアクリル板を重ねて設置しました。
すると部分的に濡れたようにキラキラした感じになりました。
コレってニュートンリングってヤツでしょうか?
コレを防ぐ方法ってありますか?
839ご冗談でしょう?名無しさん:02/04/02 18:06 ID:RKmOL+16
スス板の「ターンで加速は可能か?(スキー)」
http://sports.2ch.net/test/read.cgi/ski/1014633324/l50
というスレで、インラインスケートのバート(ハーフパイプ)で
はRのついたところで伸び上がることで、角運動量保存則により
加速が出来るが、伸び上がるかわりに軌道のほうを変えること
でも加速できるんじゃない?という話が出ました。
が、それが出来てしまうと永久機関が作れちゃう事になるので、
出来ないと思うのですが、理論的になぜ軌道を変えた場合には加
速できないのかわかりません。

運動エネルギー的には、伸び上がって加速する場合は、その伸び上
がる運動エネルギーを加速に使ってるからで、軌道を変える方は
どこからも運動エネルギーが入ってこないから、だと思うのですが、
どういう場合に角運動量保存則を使ってよくて、どういう場合には
使えないのか、よくわかってません。

高校物理レベルでわかるように教えてください。
840 :02/04/02 18:53 ID:bEGVpqhD
放射能とは何ですか?
841ご冗談でしょう?名無しさん:02/04/02 19:14 ID:???
842名無しさん@お腹いっぱい。:02/04/02 19:29 ID:???
重力の塊を拳に発生させたら手はどうなりますか
よく漫画とかで見かけるけど、リアルの場合どうなるかなと思って
843ご冗談でしょう?名無しさん:02/04/02 22:13 ID:GJe5hFgB
すいませんがE=hνというのは運動エネルギーなんですか?
844ご冗談でしょう?名無しさん:02/04/02 22:20 ID:NfZFnPNX
光子の振動エネルギー
845ご冗談でしょう?名無しさん:02/04/02 22:22 ID:???
>>843
光子のエネルギーは全部運動エネルギー
846ご冗談でしょう?名無しさん:02/04/02 22:41 ID:???
>>838
もう少し状況を詳しく書いてもらえませんか?
アクリル板2枚で塩ビシートをサンドイッチしたのですか?
847ご冗談でしょう?名無しさん:02/04/03 00:02 ID:???
>>839
ちょっと話は変わるが、紐の先に錘をつけて振り回したとき、途中で紐を引っ張って
回転半径を短くしていって、螺旋を描くように運動させると、角運動量保存則により、
錘は加速する。
一方、紐を棒に巻きつけたりして (巻きついていくことで半径が縮む) 同じような
螺旋を描かせても錘は加速しない (棒に与えるモーメントまで含めれば角運動量は
保存する)。

同じ螺旋を描かせているのになぜ違いが出るのか。
紐の張力と錘の運動方向の角度を考えればわかる。

それと同様に考えてはどうか。

…図とか描けない (AA は苦手だ) ので却ってわかりにくかったかも知れぬ。
848ご冗談でしょう?名無しさん:02/04/03 00:22 ID:???
>>839
角運動量がどうたらこうたら言う前に
床面との摩擦、反発、それらの速度依存性とか
そゆことについて考えるべきなんじゃないの?
人だけ見てたって角運動量なんて保存してないんだからさ。

該当スレ読むのメンドクサくて読んでないので
既出の意見だったらごめんなさい。
849ご冗談でしょう?名無しさん:02/04/03 00:24 ID:???
あ、なんか変なこと書いちゃった。ごめん忘れて。
850ご冗談でしょう?名無しさん :02/04/03 00:51 ID:106NJE/0
842>>
グーにした手の中の、ある1点から1gの重力場が発生するなら
手は地球方向に引っ張られる。それが右手とするなら右手方向に2gかかる
右のこぶしは、地面についたままとしたなら、それに対して踏ん張らないとだめで
なおかつ、一点に向かって力がかかるので、手をちょうど又の下の地面につけ
ふんばればなんとか持ちこたえれる。そして無理して腕を上げたら、足が手の方向に
引っ張られるので、足払いをかけられたように転んで・・・
空気も・・・・・・

まあ、えらい事になるってこった
851ご冗談でしょう?名無しさん:02/04/03 08:30 ID:+g4uOcox
空気中をマッハ2で直進して近づいて、マイクや耳など(別に何でもよいのですが)の
すぐ横を通り過ぎてそのまま直進して離れて行く音源があるとします。
その音源から聞こえてくる音は、
音源が通り過ぎた瞬間から、逆再生された音と半分の速さで再生された音が
重なって聞こえると思うのですが、皆さんはどう思われますか?
852ご冗談でしょう?名無しさん:02/04/03 09:11 ID:qwu6kZw1
>>846
下側のアクリル板に塩ビシートを貼り、その上にもう1枚の透明なアクリル板を重ねます。
ですから、アクリル板2枚で塩ビシートをサンドイッチしたような状態です。
よろしくお願いします。
853839:02/04/03 11:14 ID:r7yY6FDL
>>847
紐が棒に巻き付く場合には加速しない、というのは知らな
かったのですが、そうだとするとまさにこの問題と同じ
ことが疑問なのです。ありがとうございます >847
そう、同じ螺旋を描かせているのになぜ違いが出るの?と
いうことなのです。

ヒントを書いていただいてますが、やっぱりわかりません。
同じ軌跡を描いているということは、どちらも同じ方向へ
同じだけの力が働いている、というわけではないのでしょう
か。
854ご冗談でしょう?名無しさん:02/04/03 13:03 ID:???
>>851
なぜそう思うの?
855受験生:02/04/03 17:10 ID:1tRHA2m3
質問です。。
同容量の2つのコンデンサーが並列に繋がれていて充電されている状況を考えます。
また、終始電荷の出入りはないものとします。このとき、片方のコンデンサーだけに
誘電体を完全に挿入する際の仕事を求めよ、というものです。
これって後の静電エネルギーから最初の静電エネルギーを引けばいいんですよね。
このとき、挿入した方のコンデンサーだけの計算じゃなくて、2つの合成容量で計算
しないと正答が得られないのは何故ですか?
他方のコンデンサーは全く関与してないと考えるのは間違えなんでしょうか?
856受験生:02/04/03 17:13 ID:1tRHA2m3
あ、すでに充電は完了しているということです>>855
857ご冗談でしょう?名無しさん:02/04/03 17:40 ID:???
>>855
水槽で考えてみれ。二つの水槽がホースでつながれてて、
そこに風船を沈めるときに必要な仕事とかかんがえてみ。
858初学者:02/04/03 18:31 ID:ixRZVl7Y
物理と工学の論文はどう違うんだ?
859ひげ ◆4.Spygo2 :02/04/03 18:37 ID:7ORHAzlB
ヘビが、自分の尻尾を飲み込んでいくと最後はどうなるのですか?

ラウンジのメンバーでは明確な答えは出ませんでした。
860ご冗談でしょう?名無しさん:02/04/03 18:39 ID:Meqw/Nc+
ヘビが、自分の尻尾を飲み込んでいくと最後はどうなるの?
http://ex.2ch.net/test/read.cgi/entrance/1017821108/
861ちたま:02/04/03 19:17 ID:IG8oreIW
地球に中心を貫くように穴をあけ、
その穴に物体を通したらどうなりますか?
862ご冗談でしょう?名無しさん:02/04/03 19:18 ID:???
空気抵抗で停止。
>>859
死にます。
864ひげ ◆4.Spygo2 :02/04/03 19:51 ID:???
>>863
回答ありがとうございます。
「決して死なず、限り無く体の柔らかいヘビならどうなるだろう」
という条件下ではどうでしょうか。ラウンジでは質量保存のどーたら
こーたらで止まっています。
馬鹿げた質問でお気を悪くされたらすみません。
865847:02/04/04 00:21 ID:UCzyKuTA
>>853
錘の運動を変化させる力はどちらの場合も紐の張力だが、張力は紐の方向に
しか働かない。

紐を引っ張って回転半径が小さくなる場合と、紐が巻き付いて半径が小さく
なる場合とで、錘の軌道と紐の関係を実際に図に書いて見るとよいと思うよ。
866ご冗談でしょう?名無しさん:02/04/04 01:11 ID:???
>>864
どういう文脈で質量保存がでてきたのか
さっぱりわかりませんがラウンジには理系の学生
一人もいないのですか?
867某ラウンジャー:02/04/04 01:13 ID:???
ラウンジ的には

>その愛が宇宙になって・・・
>そして私たちは生まれますた。
>お父さん蛇二人に感謝

ということでファイナルアンサーでましたので

どうもお騒がせしました
868ご冗談でしょう?名無しさん:02/04/04 02:09 ID:???
>>867
先にウンコしたほうの勝ち
http://cheese.2ch.net/test/read.cgi/sci/1003418838/
869ご冗談でしょう?名無しさん:02/04/04 05:18 ID:OZhn+KP5
すみません、この電波を引き取ってください

心理学って人間を機械とみなしますよね... ver.2
http://yasai.2ch.net/test/read.cgi/psycho/1014468917/
870ご冗談でしょう?名無しさん:02/04/04 05:56 ID:J2W7xZcR
871ご冗談でしょう?名無しさん:02/04/04 10:56 ID:fSaM0OvN
赤い紙が赤く見えるのは、その紙が赤以外の光を吸収するから?
872ご冗談でしょう?名無しさん:02/04/04 11:06 ID:Y9MPXRPz
>871
それか、緑(赤の補色)の光だけを吸収するから。
873ご冗談でしょう?名無しさん:02/04/04 11:12 ID:fSaM0OvN
他の色の紙でも同じことが言えるんですか?
874ご冗談でしょう?名無しさん:02/04/04 11:35 ID:???
>>868
859がくわえてるのは自分のシッポだよ?

875ご冗談でしょう?名無しさん:02/04/04 11:39 ID:fSaM0OvN
レーザください。お願いします。
876839:02/04/04 12:35 ID:nBxDZ8yS
>>865
絵を描いて考えてみたんだけど、やっぱダメです。
ほんとに自分の疑問と同じ事なのかも不安になって
きました。
で、自分なりに物理の問題っぽくまとめてみました
ので、こちらを見てつっこみ入れていただけないで
しょうか。お願いします。

【実験(A)】
半径Rの半円から直線になるレール[A]があり、
レール上をなめらかに動ける軽い滑車がある。
滑車の中に重さmの重りがあり、半円を半分
進んだところから半円終了まで徐々に中心に
向かって、長さLだけmの重りが内側に飛び
出すしかけになっている。
これを初速vで動かしたところ、直線のレール
部分では速度v'となった。

【レール[A]を上から見た図】
  >─ ̄
 /  ↑
∨   R
│   ↓
| ←R→
│   ↑
∧   R
 \  ↓
  >─______

  >─ ̄■
 /  ←v




∧\_  v'→
 \ ──■↑L
  >─_□↓___

この場合、角運動量保存の法則により、
mRv=m(R-L)v1 から v1=vR/(R-L)
となるのではないかと考えました。
877839:02/04/04 12:36 ID:nBxDZ8yS
>>876

【実験(B)】
次に、重りの軌跡を調べると、半円の半分の
位置から、半径(R-L)の円弧となっていたため、
この重りの軌跡と同じ、つまり前半は半径Rで
後半は半径(R-L)のレール[B]を作り、そこを
実験(A)で使った仕掛けを作動させずに、滑車を
初速vで動かしたところ直線のレール部分では
速度v''となった。

【レール[B]を上から見た図】
  >─ ̄
 /  ↑
∨   R
│   ↓
| ←→
∧  ↑R−L
∧\_↓
 \ ───────↑L
  >─______↓

  >─ ̄■
 /  ←v


|

 \_  v''→
   ──■────

この場合は、ただ単に運動量保存則が働いて、
v''=v
となるのではないかと考えました。

【疑問】
これは正しいでしょうか?
正しいとすると、なぜ(A)は角運動量保存則が
使えて、(B)は使えないのでしょうか。
878ご冗談でしょう?名無しさん:02/04/04 12:43 ID:???
(B)だって角運動量保存則
Rmv=Rmv''
だろ。
879839:02/04/04 12:50 ID:nBxDZ8yS
>>878
それだと、なんのエネルギーも加えてないのに速度が
あがっていることになりますよね?
880ご冗談でしょう?名無しさん:02/04/04 12:55 ID:???
ハァ?
Lだけ押し上げるエネルギーはどこからくるの?
881839:02/04/04 14:23 ID:nBxDZ8yS
>>880
(A)のほうではバネとかで仕掛けが動いている、と考えて
ください。(B)はなにもおこらず、レールに従って動くだ
けです。
882ご冗談でしょう?名無しさん:02/04/04 14:28 ID:???
だからよー
そのバネが失ったエネルギーはどこにいくんだよ
883839:02/04/04 15:10 ID:nBxDZ8yS
>>882
いやあの、だから(A)のほうは、そのバネのエネルギーでもって
加速してるんだと思います。が、(B)のほうはそれがないので
加速できないのでは、と思うのです。
が、なんで同じ軌跡を描いているのに、片方は角運動量保存則
を当てはめることが出来て、一方は出来ない、のがいまいち理由
を理解できないので、それを説明して欲しいな、と。
884ご冗談でしょう?名無しさん:02/04/04 15:37 ID:???
v'=v'' だろ。違うのか?
885884:02/04/04 15:39 ID:???
そっか。違うな。納得。
886ちょっとだけ教えて:02/04/04 16:14 ID:qAf+jPxb
でさ、光って回折するじゃん。
そしたらさ、壁の向こう側がちょっとだけ見えてるってこと?
887ご冗談でしょう?名無しさん:02/04/04 16:21 ID:???
>>883
なんで同じ軌跡ってだけで全て同じになっちゃうんだ?
例えば等速度直線運動と等加速度直線運動は同じ直線の軌跡だけど
話が全然違うだろ
888ご冗談でしょう?名無しさん:02/04/04 16:36 ID:???
>>886
ちょっとだけな
889ご冗談でしょう?名無しさん:02/04/04 17:12 ID:???
そもそも(A)の場合にだって角運動量保存は一般には当てはめられないでしょ。
レールの動かし方で変わっちゃうと思うのだが。

890ご冗談でしょう?名無しさん:02/04/04 19:21 ID:QVP08ojf
Newton's timeってうまく日本語に訳すと何?
891i:02/04/04 19:41 ID:g9tVezJG
>>890
ニュートンの絶対時間
892ご冗談でしょう?名無しさん:02/04/04 22:33 ID:???
光は質量が0なのに、
なぜ運動量が0でないのですか?
また、なぜ重力に引き寄せられたり、
するのですか?

誰か、一般人にもわかるように説明
してください。お願いします。
893132人目の素数さん:02/04/04 22:36 ID:???
>>892
波だからです。
894中間:02/04/04 22:39 ID:QiYL/itD
そうそう。そのとおり。
波って難しいよね。
895ご冗談でしょう?名無しさん:02/04/04 23:50 ID:???
>>892
逆に質問します。
なぜ運動量がゼロでないと質量もゼロでないと思うのですか?
893は無視しておいて構いません。
896892:02/04/04 23:57 ID:???
894は無視しなくていいのですか?
897i:02/04/05 00:39 ID:8CmFmx3G
質量がゼロだと運動量をもたないというのは
粒子特有の性質です。波ではこう簡単には行きません。
# って書いたら、無視されるか?
898 ◆956Xb4qA :02/04/05 01:23 ID:???
>839
876で【レール[A]を上から見た図】 とあるが横から見た図の間違いでは?
でもって鉛直下向きに重力が働いているのでは?
それなら角運動量は保存されません。
太陽を公転する地球の円運動のように重力が向心力となっている場合とわけが違う。
そもそも重りが滑車から距離Lを保って浮いたまま円運動するというのは物理的に不自然。
突然バネが出てきたりと876-877の設問が曖昧で意味不明だから答えようがない。
899質問です。:02/04/05 02:00 ID:WFhtFTVs
0次元の点の大きさを教えて下さい。
1次元の線の太さを教えて下さい。
2次元の面の厚みを教えて下さい。
問題はちょっとでも大きさ、太さ、厚みがあるとそれは次元が違います。
900ご冗談でしょう?名無しさん:02/04/05 02:07 ID:???
>>897
>質量がゼロだと運動量をもたないというのは
>粒子特有の性質です。波ではこう簡単には行きません。

じゃあ質量mの粒子の運動量はmをゼロに
持っていったらゼロになるんですか?
なるなら数式で説明してくれませんか?
901ご冗談でしょう?名無しさん:02/04/05 02:24 ID:ITMF+OZP
わたしの持ってる参考書で、
回転を表す行列R
cosA -sinA 0
sinA cosA 0
0 0 1
と、3×3反対称行列Iの関係
R=exp(A・I)
から、空間回転の演算子として全角運動量(軌道+スピン)の
交換関係などを導いていましたが、
このような幾何学的な変換になぜ
スピンのような座標以外の自由度が
含まれるんでしょうか。
902ご冗談でしょう?名無しさん:02/04/05 02:57 ID:bQCzBIIp
量子ってなんですか?
903ご冗談でしょう?名無しさん:02/04/05 03:02 ID:???
>>902 武の嫁です。
904ご冗談でしょう?名無しさん:02/04/05 03:06 ID:???
>>897
光子も粒子ですが。

>>900
「持っていく」って粒子を具体的にどうするのですか? 例えば電子で。
数式すら立てられませんが。
905ご冗談でしょう?名無しさん:02/04/05 06:33 ID:???
>>898
上から見た図でいいでしょ。
横から見た図だと思うと確かに意味不明になるけど
906839:02/04/05 08:50 ID:Kqrx6gxn
>>905
そう、上から見た図です。
水平のモデルじゃないと、重力の加速を計算に入れないと
いけなくなるので、問題点がはっきりしてこなくなると
思ったので。
スス板のほうではインラインのバートを例に出してるので
すが、バートだとこの図を横から見たような感じになりま
すね。

実験(A)のほうの図で、□が滑車、■が滑車に乗ってる重り
で、最初■は滑車上にありますが、途中からバネ等を使って
円弧内側に入っていきます。その軌跡を書いたんですが、
それがちょっとわかりにくいでしょうか?

実験(B)のほうでは、その■が通った軌跡と同じように
レールをつくり、そこを滑車を転がした場合、というもの
です。
907839:02/04/05 08:54 ID:Kqrx6gxn
>>889
(A)の場合でも角運動量保存則は当てはめられないです?
そこが不可だと、議論が根底からかわってきてしまうので...

(A)も(B)もレールは動かしません。(A)の場合に、滑車に
乗っている重りが回転運動内向きに飛び出るだけです。
908889:02/04/05 09:17 ID:???
>>907
889はレールが動くと勘違いした故のザレゴトです
あと、滑車というよりは台車とでも言ったほうが誤解がないですね。
滑車というと糸がひっかかってくるくる回るものを思い浮かべますから

ともあれ、角運動量保存則がどういう仮定の元に成り立つか、(A)(B)で
その仮定は成り立っているかを考えれば自ずと答えは出ます。
909889:02/04/05 09:20 ID:???
書き忘れました。角運動量は回転の中心を指定しないと定義できないことに注意。
910839:02/04/05 10:03 ID:Kqrx6gxn
>>909
ありがとうございます〜
つまり、>877で書いた疑問への答えは、
「角運動量保存則は、回転中心が変化しないで回転半径が
かわる場合にしか成り立たないから」というまとめでよい
でしょうか。
911ご冗談でしょう?名無しさん:02/04/05 10:32 ID:???
>>910
それだと軌道が同じなら全て同じになっちゃわない?
バネの伸ばし具合をうまく調整すれば、(A)の軌道は(B)と同じにすることも
できますよ?


912ご冗談でしょう?名無しさん:02/04/05 10:49 ID:???
>>910
角運動量保存則がどういう場合に成り立つのか理解できてないのに、
角運動量保存則が使えるのかどうかを考えてもしょうがないでしょう。

角運動量はトルクが0のとき変化しない。これを元に改めて考えれ
913898 ◆956Xb4qA :02/04/05 10:50 ID:???
>>839
角運動量保存則が成り立つのはトルクが0になる場合。
半径Rの半円と半径(R-L)の円弧の中心は一致しないよね。
だから半径Rの半円の中心を座標系の原点にした場合、
半径(R-L)の円弧上で重りに働く力は半径(R-L)の円弧の中心を
向いて座標系の原点方向には向かないと。
よって位置ベクトルと力が平行にならずにトルクも0にならない。
914ご冗談でしょう?名無しさん:02/04/05 14:13 ID:???
900越えたので次スレの準備ヨロ
915ご冗談でしょう?名無しさん:02/04/05 14:53 ID:???
あの、ここで聞いていい質問かわかりませんが、今年のLT23(低温国際会議)に
Abstractを出したのですが、その審査って通りやすいものなのですか?
自分の学位に関わってくるので是非知りたいのですが。
ベルの不等式は成立しないものだと思っていましたが、
まだ、疑義があるのですか?

917ご冗談でしょう?名無しさん :02/04/05 23:19 ID:90/cBS0B
>>839
で、その飛び出したmってやつも、回転運動してんの?
918ご冗談でしょう?名無しさん:02/04/05 23:31 ID:4OfNL+6u
http://www.ed.kagu.sut.ac.jp/~j2200213/
このサイトについて、どう思いますか?
919工学生:02/04/05 23:32 ID:LIcvi4dw
工学部のマスターを取ったのですが
本当は物理も勉強(研究)してみたいです
どういうところから手をつけたら
良いのでしょうか?
まず,分野から決めたいのですが,
本などを読んでいても数式が複雑で
どうも勝手が違います.
どうやって勉強していけば良いのでしょうか?
920工学生:02/04/05 23:33 ID:LIcvi4dw
ちなみに専門は複雑系の応用です
921ご冗談でしょう?名無しさん:02/04/06 02:40 ID:???
>>919-920
立場がよく分からないのですが、修士を出て今は就職してるんですか?
922休憩時間:02/04/06 09:57 ID:7gGyu0g0
漏れは、この板の住人ではありませんがちょっと質問します。
アニメの「ドラえもん」知ってます? それに出てくる「タケコプター」。
あれが実在したとします、(2枚羽根・半径10p・超軽量で硬い未来の金属)
40kgの子供を空中へ浮遊させるには、ペラの回転数がどれくらい必要でしょうか?
又、浮遊した場合子供は猛烈な勢いでペラとは逆の方向に回るような気がする
のですが・・・。無理やり理論をつけて下さい。
923839:02/04/06 10:13 ID:JaKE7jG4
>>917
むろん重りmは台車とつながっているという前提で考えてました
ので、回転運動しています。
924ご冗談でしょう?名無しさん:02/04/06 10:33 ID:PonEi4cj
>>923
 >>913で答え出てますよね?一応もうひとつ補足しておけば
「角運動量は原点を決めなきゃ値は不確定」
ってとこですか。これで解決!
925ご冗談でしょう?名無しさん:02/04/06 12:14 ID:/2slD4d3
広島に投下された原子爆弾は20キロt(TNT高性能火薬2万トン)
の爆発力があったそうです。
一方、ビキニ環礁で使用された水素爆弾は14メガt(広島型の700倍)
の爆発力があったそうなのですが、この700倍というのはどういう事
なのでしょう???

広島に投下された原爆は中心地から1.5キロ以内の
建物や人に壊滅的な被害を与えたそうなのですが、
700倍の威力ということは被害範囲も700倍増しになるのでしょうか?
926ご冗談でしょう?名無しさん:02/04/06 12:20 ID:???
発生する熱量が700倍ということ。
爆発の影響は、概ね、距離の2乗に反比例する。
927839:02/04/06 12:24 ID:JaKE7jG4
>>924
です。>>912,913あたりのレスが答えですね。
自分なりの理解としては「角運動量保存則は、回転運動系の
原点がかわらないときに、トルクが0の場合に成り立つ。」
928兵器ヲタ:02/04/06 12:27 ID:/2slD4d3
>>926
概ね、とはなんでしょうか?
929ご冗談でしょう?名無しさん:02/04/06 12:42 ID:???
>>928
途中で熱や放射線の吸収があったりするから、
必ずしも、逆2乗に従うとは限らないとか色々。
930兵器ヲタ:02/04/06 12:49 ID:???
発生するエネルギーが700倍ってことで700倍の距離に被害が
拡大するってことではないのですね。

>>626&629算、
レスさんくすこです
931ご冗談でしょう?名無しさん:02/04/06 13:55 ID:+dF0rnsD
大学の物理学科と応用物理学科ってどこがちがうの?
932ご冗談でしょう?名無しさん:02/04/06 14:42 ID:U12ilYzB
物理の1組と2組。。つうことで。
933ご冗談でしょう?名無しさん:02/04/06 15:13 ID:PRISee9H
f=-dV/dx
f:力
V:ポテンシャルエネルギー
x:変位
ってどういう考えで導きだされたんですか?
本読んでも,「こう定義した!」としか書いてないんですけど。。。
934ご冗談でしょう?名無しさん:02/04/06 15:15 ID:pSUmbRRN
マジですごいこと考えたよ!これを学会に提出したらノーベル賞間違い無しだね。
よく聞けよ、あのさ、地球って自転するよな?それを利用するんだよ。
1日で一回転するってことは12時間で現在の日本の場所にブラジルがくるってこ
とになる。てことはいまこの場所で浮いていれば勝手に地球が回ってくれるから
浮いてるだけでブラジルまで行けるってわけだ。海の上も通過するのであったかく
していったほうがいいかと思われる。しかし公転のことも考えると宇宙空間に出てしまう
おそれもあるので注意していただきたい。4月5日現在、この原理についての論文を作成中
であり史上最年少でのノーベル賞受賞も時間の問題である。


935ご冗談でしょう?名無しさん:02/04/06 15:32 ID:WCLHYLbx
>>934
すごい!面白くなさ過ぎてかえって面白いぞ。
936ご冗談でしょう?名無しさん:02/04/06 21:14 ID:DT+810LM
>>935
フカクニモワラタ
937ご冗談でしょう?名無しさん:02/04/06 23:41 ID:???
雪だるまをレンジでチンしたら、
溶けますか?蒸発しますか?
938ご冗談でしょう?名無しさん:02/04/06 23:43 ID:V4ScbzSU
>>937
温まります。

ごめんおもしろくないね。氏にます
939ご冗談でしょう?名無しさん:02/04/07 05:48 ID:???
>>937
シナクテイイ
940ご冗談でしょう?名無しさん:02/04/07 15:31 ID:eM4vP5mR
>>933
xの距離でのvの変化率
941ご冗談でしょう?名無しさん:02/04/07 21:56 ID:???
>>933
 ポテンシャルエネルギーの定義、ただそれだけだな。
 ただ、xは変位じゃなくて座標だと思うが。
 何か問題が?

942水無瀬:02/04/08 01:19 ID:???
超弦定理っていうのは、どういった定理なのですか?

慨出ならスミマセン。
943ご冗談でしょう?名無しさん :02/04/08 02:07 ID:Hy1MH6F/
>>942
万物は小さな弦からなるていう定理
944H imajin@留学:02/04/08 02:11 ID:Y7qHYEa7
 重力遮断技術


  重力遮蔽とは、重力の影響を遮る、つまり重力が働かないような状況を意味する。
  普通ならできない、それで終わりのところ、NASAでは研究を続けている。
   # 現在の学会では驚愕のできごと。まあ、日本では絶対やらないでしょうね。

  ことの発端は、92年のロシアの研究者が2%の重力遮蔽に成功したという報道
  が最初である。もし本当なら、ノーベル賞をいくつももらえるほど、画期的だ。

  この実験を確かめようと、NASAは研究を開始したようだ。その費用、8千万
  円ほどが、試験装置に費やされた。

  その装置は、超伝導材料による、直径15cm・厚さ1cmほどの円盤だ。これ
  をマイナス233度で、毎分5千回転の速度で回転すると、円盤から上の領域は
  重力が遮られ、モノの重さが軽くなると言う。
   # 超伝導材料は、非常に脆いのが特徴だ。果たして、この回転数で無事?
     なお、円盤を造ったのは、超伝導コンポーネント 社

  既に、昨年実験がなされているが、それはいい結果ではなかった。この度、また
  あらためて実験が行われるようだが、果たして・・
   # こういう不可能へのチャレンジが、世の中を変えていくことを信じたい
     ものです。

こぴぺですんません、こんな事ってあるんでしょうか?
945ご冗談でしょう?名無しさん:02/04/08 02:13 ID:???
もうちょい回した方がいいんでは。
◆麻呂は政治死相板の原住民でチンケなヤマト(=中卒)で娯JALダニ!!!

★その政治死相板で【神とも皇子とも】尊称されておられる菊地ちゃ〜んが、日記を憑けておりますので、遊びに診に来てくれマンコで娯JALダニダニ!!!

★以下は、麻呂の菊地ちゃ〜んの妄想でなく【日記】のアドレスなので娯JALダニダニ!!! ヨロチンコ!!!
菊地ドットコム(真面目な人が報われる日本に!)
http://www2.diary.ne.jp/user/138138/

◆追伸!
 ご意見やクレームなどは、以下に書き込みをしてくれマンコで娯JALダニダニ!!!
http://tmp.2ch.net/sisou/
http://tmp.2ch.net/test/read.cgi/sisou/1018113469/
菊地奮戦記 社会人編 part2
947ご冗談でしょう?名無しさん:02/04/08 13:10 ID:???
今度はガウスの発散定理か…、しかし毎回どっから肖像が画
みつけてくるんだろ?
948物理厨房:02/04/08 13:30 ID:hfhVdeRU
この問題の(b)の解き方を御教示頂けませんか?

http://cgi.members.interq.or.jp/leo/sinki/cgi-bin/photo/img-box/img20020407122025.jpg
949ご冗談でしょう?名無しさん:02/04/08 17:20 ID:8pZvgHzo
>>948
英語ヤダ
950 :02/04/08 19:32 ID:???
>>948
運動方程式を解けばいい。
951質問です:02/04/08 21:12 ID:Qsk9XKK/
次の様な3つの偏向板を用意します。

A:水平方向に偏向面を持つ偏向板(角度0°)
B:垂直方向に偏向面を持つ偏向板(角度90°)
C:斜め45°に偏向面を持つ偏向板

このとき、3つの偏向板を、ABC や CAB という具合に、
AとBが隣り合うように並べた場合は、光は通過しません。
これは、AとBが互いに90°の角度をなすので、縦偏光と横偏光が遮断されるので
光は通過できないという具合に、直感的に分かりやすく理解できるかと思います。
ところが、CをAとBの間にはさんで、ACB の順番で並べると、
光は通過するという話を読んだことがあります。
絵にするとこんな感じです。

 光→ABC・・・(光は通過しない)
 光→CAB・・・(光は通過しない)
 光→ACB→  (光は通過する!)

でも、何故、ACBの場合だけ光が通過するのか、よく分かりません。
ACBの順に並べても、AとBが入ってるのだから、
結局は、光は遮断されるようにも思えるのですが、実際に光は通過するようです。
そして、量子力学を適用すると、このことがきちんと説明できるのだそうです。
そこで質問なのですが、量子力学的にこの状況を具体的に記述すると、
どの様に説明されるのでしょうか?
952ご冗談でしょう?名無しさん:02/04/08 22:19 ID:pTMcEaNQ
>>951
 偏光板は、偏光の向きを選択するんじゃなくて変化させるの。
古典論の範囲内だぞ。
 量子力学云々は、スピンの問題のアナロジーだろ。
953ご冗談でしょう?名無しさん:02/04/08 22:30 ID:???
>>951
偏光版は「スピンの観測」のアナロジですね。
この場合、観測の順序は重要な意味を持ちます。
非可換な観測をうまいこと繰り返すと、
状態を殆んど任意の状態に持っていったりできます。
量子ゼノン効果とか名前が付いていたような…
954ご冗談でしょう?名無しさん:02/04/08 22:47 ID:???
最近ギター始めて思ったんですが
1オクターブ(低いドから高いドの間)
って物理的にはどんな関係があるんでしょうか?
955ご冗談でしょう?名無しさん:02/04/08 22:54 ID:???
>>954
周波数が2倍
956ご冗談でしょう?名無しさん:02/04/08 23:21 ID:???
>>954
弦の長さが1/2
957ご冗談でしょう?名無しさん:02/04/09 00:20 ID:???
>>956
弦の線密度と張力が同じ場合ですね。
958ご冗談でしょう?名無しさん:02/04/09 00:21 ID:???
>>953
>>951の偏光板の話は、北野先生の量子ゼノン効果のレビュー記事
でしょう。
この話は、また、量子暗号にも深く関連しています。
959954:02/04/09 00:26 ID:???
周波数違うのになんで同じドなんでしょう?
960955:02/04/09 00:40 ID:gLTZGqFj
>>959
人がそう決めたかられすね。
アルキメデスかなんか忘れたけど、ハーモニーって言いまして、
いくつかの周波数が簡単な整数比になるものを特別視したわけです。
で、ドと1オクターブ高いドは1:2、とか色々あります。
こう言う音階を純正律、って言うんですが、色々不都合があるってことで、
バッハが平均律、って言う音階を作った。これは1:2の比以外は
皆近似なんです。

板違いですまぬ。
961ご冗談でしょう?名無しさん:02/04/09 00:41 ID:xgaji60H
物理的に僕を馬鹿にせよ」
962三村(物理的):02/04/09 00:51 ID:???
>961
(始まってないのに)閉じちゃったよ!
963953:02/04/09 01:13 ID:???
>>958 さんくす!
その「北野先生の量子ゼノン効果のレビュー記事」って何処に
載っているのか教えて貰えませんか?
964ご冗談でしょう?名無しさん :02/04/09 01:36 ID:MxCq05HG
961>>
君は、あれだな
965ご冗談でしょう?名無しさん:02/04/09 08:45 ID:xgaji60H
まぞ
966ご冗談でしょう?名無しさん:02/04/09 10:15 ID:???
誰か新スレたてろ
967ご冗談でしょう?名無しさん:02/04/09 11:06 ID:xgaji60H
俺は最近立てたばかりだ

お役に立てずすまん
968ご冗談でしょう?名無しさん:02/04/09 16:37 ID:g1sbmsdf
恐れ入りますが、

BB弾の重量とエネルギー
http://caramel.2ch.net/test/read.cgi/gun/1018085930/

これをちょっとだけ見てやって、正しい解説をしてくださる方おられませんか?
969ご冗談でしょう?名無しさん:02/04/09 16:58 ID:???
音響磁気効果って何ですか?
フォノンが関係してるからこんな名前なんですか?
万引き防止タグに興味をもって調べていたらこの効果を利用してるらしい
ことまでわかりました。
970ご冗談でしょう?名無しさん:02/04/09 20:36 ID:jWF51hQN
ペットボトルからお茶を一気に出す時
ドッドッドッドッドっとでるのは何故?
971951:02/04/09 20:49 ID:???
レス有難うございます。

>>952
偏光のことも、正直、詳しくないです(汗

>>953
952さんも指摘している通り、「スピンの観測」と関係が深いようですね。
「スピンの観測」に関する分かり易い参考書があったら教えて欲しいです。

>>958
私が>>951の話を読んだのは、一般向けの啓蒙書でした。
それには、証明が全然載ってなくて、謎だけが残りました(w
972ご冗談でしょう?名無しさん:02/04/09 22:15 ID:???
>>963
記憶によると、「光学」っていう学会誌だったと思う。
2年ぐらい前だろうか?
http://www.kuee.kyoto-u.ac.jp/~kitano/zeno/
も見てミソ。
973ご冗談でしょう?名無しさん:02/04/09 22:29 ID:???
>>971
サクライの「現代の量子力学」とか、高林武彦の「量子力学とは何か」
とかはいかがですか?
974ご冗談でしょう?名無しさん:02/04/10 00:33 ID:5CtK7Wcd
質問はここに書けばいいのかな?

4次元とか11次元とか36次元がイメージできません。
どうすればイメージできますか?
マジで教えて。
975ご冗談でしょう?名無しさん:02/04/10 00:40 ID:9MVCVcGo
>>974
凡人には無理。
それに、せいぜい(イメージ)出来る人でも
5次元まで。
976ご冗談でしょう?名無しさん:02/04/10 00:46 ID:5wL/btO0
>974
定義からきちんと勉強するとそーゆー悩みはなくなる。
977ご冗談でしょう?名無しさん:02/04/10 01:11 ID:5CtK7Wcd
>>795
5次元でもいいからイメージしたいのですが。
何かイラストとかないでしょうか。
勉強しかないのでしょうか。

>>976
なるほど。がんばってみます。
978ご冗談でしょう?名無しさん:02/04/10 01:20 ID:???
>>977絵でりかいしようとしてる時点でむずかちいぞ。
絵なんて2D,せいぜい3Dだ。イメージだ!
979ご冗談でしょう?名無しさん:02/04/10 01:32 ID:2kD6YI3a
統計力学について質問です!系TUを接触させ,
それを一つの孤立系と見て,結合系の状態密度を求める問題です.
系T,Uの状態密度をΩ1,Ω2,TUの結合系の状態密度をΩ12とします.
またT,UのエネルギーはそれぞれE1,E2で,E1+E2=Eです.
TのエネルギーがE1とE1+dE1,UのエネルギーがE2とE2+dE2の間に
あるような微視的量子状態の数は
     Ω1(E1)Ω2(E2)dE1dE2
となります.これをE-ΔE<E1+E2<Eの範囲で合計すると,
積分変数をE=E1+E2とE2に変更すると,
     Ω12=∫Ω1(E-E')Ω2(E')dE'   (積分区間0→E)
となるとあります.(久保リョウゴの統計力学より)

質問1:最後の式になる理由がわかりません.
だって,dE1=d(E-E2)=−dE2となるから,積分変数を変更すると,
dE1dE2=−(dE2)^2となって,最後の式は,
     Ω12=∫Ω1(E-E')Ω2(E')(dE')^2
とならないでしょうか?おかしな式ですけど(汗)
つまりなんでdE1dE2だったものが,一重積分になってしまうんですか?

質問2:E-ΔE<E1+E2<Eの範囲でってありますよね?
なんでΔを使ってるんですか?E1,E2については,
E1とE1+dE1,E2とE2+dE2というように微分のdを使っているのに.

すみません,教えて下さい.

     
980訂正:02/04/10 01:39 ID:2kD6YI3a
Ω12=∫Ω1(E-E')Ω2(E')(dE')^2
→ Ω12=−∫Ω1(E-E')Ω2(E')(dE')^2
すみません,マイナス付け忘れました.

E1とE1+dE1,E2とE2+dE2というように微分のdを使っているのに
→E1とE1+dE1の間,E2とE2+dE2の間,というように微分のdを使っているのに
981ご冗談でしょう?名無しさん:02/04/10 02:01 ID:+EG2GypF
地面から垂直に立った棒をそのまま倒すのでは無く、
棒の重心から倒したやり方、すなわち棒の底辺部を
前に引いたり、後に押したりして、棒の重心部が垂直
落下させた方が地面に激突させたときのエネルギーが
大きいと聞いたことがあります。
本当でしょう?
もし本当なら、その理論の正式名(物理用語)などはありますか?
よろしくお願いします。
982ご冗談でしょう?名無しさん:02/04/10 02:50 ID:Q6hbamnG
>>981
そんなことねぇだろ
重心の位置はどっちの場合も同じだろ
回転のエネルギーは・・・
加わるか??
983ご冗談でしょう?名無しさん:02/04/10 04:44 ID:???
そんなこともこんなことも、
何をいっているのかわけわからんよ981は
984 :02/04/10 06:32 ID:???


ペットボトルからお茶を一気に出す時
ドッドッドッドッドっとでるのは何故?



985ご冗談でしょう?名無しさん:02/04/10 09:41 ID:???
>>978
そっか俺には無理そうです。ありがとう。
986困った時の名無しさん:02/04/10 15:03 ID:gP7Zg1mE
ペイアウトの利用
ビギナーのための攻略法です。
ここでは説明どおりにすればほぼ確実に
150ドル(約19,500円)〜210ドル(約27,300円)を手にする方法です。
詳しく説明しています。
あなたも賞金を手にして下さい。
http://www.medianetjapan.com/10/entertainment/casinoboy/martin.html
987ご冗談でしょう?名無しさん:02/04/10 16:00 ID:???








           ねずみ講には気をつけろ








                         >>986
988951:02/04/10 19:05 ID:???
>>973
有難うございます。ちょっと調べてみたいと思います。
989oo:02/04/10 22:18 ID:???
何故タバコの火を水につけると「ジュッ」という音がするのですか?
990ご冗談でしょう?名無しさん:02/04/11 20:29 ID:???
重力(引力?)って何なのか。と。
空間の歪みですか。粒子ですか。
どっちですか。
教えてください。
991ご冗談でしょう?名無しさん:02/04/11 20:51 ID:???
>>990
質問に答える前に

あなたは光を波だと理解しますか?
それとも粒子と理解しますか?
それとも別の理解をしていますか?

答えて貰えるとありがたい。
992天帝ユタ様 ◆UEhy5aKQ :02/04/11 21:05 ID:AECJXurg
重力とは次元の閉鎖なのだ。
物質が誕生し、周期を加速させることによって、次元は閉鎖していく。
次元の閉鎖によって、一つの塊になるのだ。
http://ime.nu/219.23.28.73:8080/atom.html
993天帝ユタ様 ◆UEhy5aKQ :02/04/11 21:10 ID:AECJXurg
だから重力はいわば物質を極めて物質化すると発生する。
水素とへリウム(あるいは元素周期)、この周期のズレが重力になるのではないか。
http://ime.nu/219.23.28.73:8080/atom.html
994ご冗談でしょう?名無しさん:02/04/11 21:13 ID:m9X/H2iE
今年から高Vになります。数学は得意なのですが物理が少しばかり苦手で。
物理は好きなのですが思うように習得できません。
物理でお勧めの参考書とか問題集があれば教えていただきたいのですが。
995ご冗談でしょう?名無しさん:02/04/11 21:13 ID:hZktLoyI
ろうそくの炎がろうそくを傾けてもいつも上向きになるのは何故ですか?
996天帝ユタ様 ◆UEhy5aKQ :02/04/11 21:17 ID:AECJXurg
電子密度+元素周期+重力=100%
重力(C%)=1-((1−電子密度(E%))+元素周期(S%))
http://219.23.28.73:8080/atom.html
997天帝ユタ様 ◆UEhy5aKQ :02/04/11 21:19 ID:AECJXurg
アー間違えた。人工知能用の論理なのにな。
((1−電子密度(E%))+元素周期(S%)+重力(C%)=100%
998ご冗談でしょう?名無しさん:02/04/11 21:23 ID:???
>>991
人間ごときには粒子にしか見えないけど、
妖精さんには波のように見えているのかも。

いや、ブルーバックスの多世界理論とやらをちょっと読んで、
勝手に妄想しただけなんですけど。
妖精を信じてるわけではないので、一応。
つまり、波かと思う。ヒトには見えないだけで。

>>992-993
よくわからないけど、まじっすか?
999天帝ユタ様 ◆UEhy5aKQ :02/04/11 21:26 ID:AECJXurg
((1−電子密度(E%))+元素周期(S%)+重力(C%)=原子次元(E:100%)
天帝ユタ様のいうことが
よくわからない僕はドキュンですか?
10011001
このスレッドは1000を超えました。
もう書けないので、新しいスレッドを立ててくださいです。。。